lexnik XVIII f . 4- .v Vsebina Uvodnik Jerneja Bone 02 Sobota - je kaj matematike v njej ? Vrtec Andreja Nudl, Dejan Brezočnik, dr. Alenka Lipovec, Darja Antolin 05 Struktura zastopanosti matematičnih dejavnosti v slovenskih vrtcih Osnovna šola Natalija Čerček 015 Obdelava podatkov v začetnem šolskem obdobju Lucija Gregorič 024 Matematika v podaljšanem bivanju Anja Janežič 030 Nič pomeni nič - nečesa Tadeja Možina 045 Stili zaznavanja in diferenciacija Srednja šola Nevenka Križman 057 Povezovanje matematičnih in strokovnih znanj v programu lesarski tehnik Rosana Vrh Makarovič 068 Matematika - pomemben pripomoček dobremu slaščičarju Simona Vreš 073 Naravoslovje kot medpredmetno zasnovan izbirni predmet v drugem letniku gimnazije Kristijan Breznik, Janko Marovt 090 Nekaj več o praštevilih Samo Repolusk 097 Priprava bodočih učiteljev matematike na povezovanje znanj pri pouku matematike V Contents Editorial Jerneja Bone Saturday - What is mathematics in it? 2 Kindergarten Andreja Nudl, Dejan Brezocnik, dr. Alenka Lipovec, Darja Antolin The Structure of mathematical activities in Slovenian kindergartens 5 Primary school Natalija Cercek Data processing in the initial school period 15 Lucija Gregoric Mathematics in after school care 24 Anja Janezic Nothing means nothing - of something 30 Tadeja Mozina Types of perception and differentiation 45 Secondary school Nevenka Krizman Linkage of mathematical and professional knowledge in program for woodworking technician 57 Rosana Vrh Makarovic Mathematics - important help to confectioner 68 Simona Vres Science as an intercurricularly based optional subject in 2nd grade of Gimnasia 73 Kristijan Breznik, Janko Marovt More about the prime numbers 90 Samo Repolusk Preparation of future mathematics teachers for integration of knowledge at mathematical lessons 97 01 Sobota - je kaj matematike v njej ? Uvodnik Jerneja Bone Sobota je dan, ko nisem v službi (se pa tudi zgodi) in se moje odgovorna urednica dogajanje ne vrti okoli matematike. Se morda motim? Poskušam se sprehoditi skozi pretekli dan in si odgovoriti, koliko in katero matematiko potrebujem in kako jo povezujem v vsakdanjem življenju. Jutro Sobotno jutro je drugačno od običajnih juter med tednom. Umirjeno prebujanje. Potem dobra in dišeča jutranja kavica v lepi skodelici. Jutranje primorsko sonce (ja v Ljubljani ga te dni ni) zariše zanimivo svetlobo na gladini kave - to je kavstika. Ob tej zanimivi igri svetlobe se mi obudi spomin na predavanje dr. Marka Razpeta na strokovnem srečanju in občnem zboru DMFA konec oktobra 2011 v Portorožu. Če povem preprosto, je kavstika ovojnica žarkov, ki so se odbili ali so bili lomljeni. Kavstike je prvi uvedel in proučeval Ehrenfried Walther von Tschirnhaus. Tudi svetloba, ki se odbija od prstana, naredi tako obliko. Ja, spet matematika. Pa saj nisem v službi! 02 MATEMATIKA V ŠOLI, letnik 18, številka 1-2, marec 2012 | ISSN 1318-010X | Izdal in založil: Zavod RS za šolstvo, Ljubljana, Poljanska 28 | Predstavnik: mag. Gregor Mohorčič | Uredniški odbor: | Jerneja Bone, ZRSŠ, OE Nova Gorica; jerneja.bone@zrss.si; | dr. Darjo Felda, Univerza na Primorskem, Pedagoška fakulteta Koper, darjo.felda@pef.upr.si; | dr. Marjan Jerman, Univerza v Ljubljani, Fakulteta za matematiko in fiziko, marjan.jerman@fmf.uni-lj.si; | Darja Antolin, Univerza v Mariboru, Pedagoška fakulteta Maribor, darja.antolin@uni-mb.si; | dr. Zlatan Magajna, Univerza v Ljubljani, Pedagoška fakulteta Ljubljana, Zlatan.Magajna@pef.uni-lj.si; | mag. Mojca Suban Ambrož, ZRSŠ, OE Novo Mesto, mojca.suban@zrss.si; | Simona Vreš, Gimnazija Ravne na Koroškem, simona.vres@guest.arnes.si; | Sabina Kumer, Šolski center Krško - Sevnica, kumer.sabina@gmail.com; | dr. Lucija Željko, OŠ Sostro, Lucija.Zeljko@guest.arnes.si; | dr. Šefket Arslanagic, Univerza v Sarajevu, Prirodno -matematički fakultet, Bosna in Hercegovina, | dr. Vladimir Kadum, Visoka učiteljska škola u Puli, Hrvaška, | Herremans Adriaan, Universiteit Antwerpen | Jezikovni pregled: Tatjana Ličen | Izvlečki v angleščini: mag. Gregor Adlešič | Oblikovanje: Anže Škerjanec | Urednica založbe: Simona Vozelj | Naslov uredništva: Zavod RS za šolstvo, OE Nova Gorica (za revijo Matematika v šoli), Erjavčeva 2, 5000 Nova Gorica | Prelom in tisk: Littera picta d.o.o. | Naklada: 700 izvodov | Letna naročnina (4 številke oziroma 2 dvojni): 20,86 EUR za šole in ustanove, 14,19 EUR za posameznike in 13,35 EUR za dijake, študente in upokojence. | Cena posamezne dvojne številke v prosti prodaji je 13,35 EUR. | Naročila: ZRSŠ - Založba, Poljanska cesta 28, 1000 Ljubljana, faks: 01/30 05 199, e-pošta: zalozba@zrss.si | Revija je vpisana v razvid medijev, ki ga vodi Ministrstvo za kulturo pod zaporedno številko 568. | Revija Matematika v šoli je indeksirana in vključena v mednarodne baze podatkov: | MathEduc - Mathematics Education Database, ZDM - The International Journal on Mathematics Education, Co-operative Online Bibliographic System and Serveces (COBISS) | Poštnina plačana pri pošti 1102 Ljubljana. | © Zavod Republike Slovenije za šolstvo, 2012 | Vse pravice pridržane. Brez založnikovega pisnega dovoljenja ni dovoljeno nobenega dela revije na kakršenkoli način reproducirati, kopirati ali kako drugače razširjati. Ta prepoved se nanaša tako na mehanske oblike reprodukcije (fotokopiranje) kot na elektronske (snemanje ali prepisovanje na kakršenkoli pomnilniški medij) ter medijske oblike reprodukcije. | - kolofon- Ob sobotah smo naročeni na enega izmed slovenskih dnevnikov. Z veseljem ga prelistam in preberem, kar me zanima, največkrat samo naslove, potem pa si rečem, popoldne pa še to in ono. A kaj, ko pri branju različnih prikazov (krožni, stolpčni, linijski...), ki jih je v tem času res veliko, zopet nehote uporabljam znanje matematike. Dopoldne Dopoldansko opravilo je pospravljanje. Najmlajša se igra v svoji kuhinji, pretaka, preliva in poliva. Vse je mokro. Kaj je že rekla dr. Tina Bregant v Olimju na strokovnem posvetu Fleksibilni predmetnik in aktualni izzivi osnovne šole, ki je bil 29. in 30. septembra 2011 v Podčetrtku? »... To, kako bo otrok dobro reševal odvode v srednji šoli, se pravzaprav začne že v starosti dveh let; ko on preliva vodo iz enega lončka, primerja prostornino . to je osnova matematičnega mišljenja ...« Potem pa naj preliva in poliva. Bomo že pospravili in pobrisali, samo da bodo odvodi šli v glavo. Še nujno dopoldansko opravilo: obisk trgovine. Izbiram izdelke, vsak je označen s črtno kodo, Kaj vse je zapisano s črtnimi kodami? Vse, razen cene. Pomislim, kaj bi lahko naredili s črtnimi kodami v šoli pri pouku matematike. Kaj vse bi se morda vprašali učenci, če bi jim dali možnost, da se vprašajo in razvozlajo. Ideja, ki jo moram še natančneje premisliti, domisliti, si rečem in grem dalje po trgovini. Opoldne Še kuhanje. Tudi štedilniki so že pametni: ko voda v posodi zavre, vtipkam, koliko časa naj vre, ko preteče določen čas, pa mi zapiska. Še na uro mi ni treba pogledati. Malo drugače je, ko v teh jesenskih dneh kuham na štedilnik na drva. Tu pa je vse drugače. Ogenj, dovolj drv; zdaj je premalo, nato preveč vroče; poglej na uro in si zapomni, kdaj bo kuhano. Še dobro, da lahko kot kuharica hrano prej poskusim. Pa ocena količine vode v posodi! Ni problema, saj je na notranjosti posode označeno 0,5 l vode, 1 l vode in tako dalje. Če znam sama določiti približno liter vode, se bojim, da moji otroci tega ne bodo znali, če jih ne bomo tega učili. Priprava mize za kosilo je delo naše triin-polletnice. Vsakemu en krožnik, ena žlica . Prirejanje. Preslikave. Štetje. Zadnje leto štejem vse od ena do deset. Vse morava prešteti, znova in znova. Popoldne Popoldne je bolj umirjeno. Ne dogaja se veliko. Pospraviti je treba sinove kaktuse. Resda imam delo z njimi tudi jaz, a so predvsem njegova skrb. Tako lepi so, urejeni, ra- znoliki. Filotaksa - urejenost listov, mu razložim. Obisk velikih trgovin je pri nas redek. Sem pa bila prijetno presenečena, ko sem zagledala našo naslovnico v grobi obliki na reklami ene izmed naših trgovin. Lepo! Mimogrede še reklama za revijo Matematika v šoli. Večer Sobotni dan je hitro minil. Čeprav na videz nepovezan dan z matematiko, je bila vseskozi nekje v ozadju matematika. Še marsikaj zanimivega se je zgodilo. Morda še kdaj v obliki dnevnika v uvodniku revije. 04 Uvodnik Struktura zastopanosti matematičnih dejavnosti v slovenskih vrtcih The Structure of mathematical activities in Slovenian kindergartens I Povzetek Matematiko v predšolskem obdobju lahko razdelimo na tri vsebinska področja: aritmetiko in algebro, geometrijo in merjenje in druge vsebine. Zanimalo nas je, koliko matematičnih dejavnosti izvajajo vzgojitelji slovenskih vrtcev in kako uravnotežena je zastopanost po vsebinah. Pregledali smo 130 poročil študentk vzgojiteljic in v njih zasledili 800 dejavnosti. V prispevku podajamo analizo zastopanosti po področjih in predloge za izboljšanje predšolske prakse. Ključne besede: matematika, predšolska vzgoja, aritmetika, geometrija, merjenje. Andreja Nudl, Dejan Brezočnik, Alenka Lipovec, Darja Antolin Pedagoška fakulteta Maribor I Abstract Preschool mathematics can be divided into three different content areas: arithmetic and algebra, geometry and measurement, and other contents. We were interested in the quantity of mathematical activities in which children are engaged in Slovenian kindergartens. We have also described the ratio in which specific contents were observed. We analysed 130 portfolios of future preschool teachers and discovered 800 different types of activities. In the paper we analysed the distribution of different content areas and offered some suggestions for improvement of practice in preschool institutions. Keywords: mathematics, preschool education, arithmetic, geometry, measurement. Matematika v šoli ~ XVIII. [2012] ~ 005-014 a Uvod Področje predšolske matematike je vedno bolj raziskovano. Za razmah tega področja se Navaja več razlogov: večja vključenost otrok v vrtce, pomembnost matematike kot temeljne znanosti, problematika zagotavljanja principa enakosti in pomembnost zgodnjega matematičnega izobraževanja za nadaljnjo poklicno pot (Clements, 2006). Otrok lahko pri mnogih situacijah najde rešitev s pomočjo matematike (Kurikulum za vrtce, 1999, str. 64). Predšolski otroci se torej srečujejo z matematiko, prav zato se pojavljajo vprašanja o tem, zakaj imajo v šoli težave s šolsko matematiko. Za to obstaja več razlag. Piaget npr. meni, da otroci potrebujejo več praktičnih izkušenj, in poudarja pomen dejavnosti, prilagojenih razvojni stopnji. Morda je težava v tem, da se v šolski matematiki (čeprav imajo že precej izkušenj) otroci ne znajo izraziti v matematičnem jeziku in s simboli (Huges, 1986, Donaldson, 1978, povzeto po Manfreda Kolar, 2006). Veliko je matematičnih konceptov, ki jih otroci lahko usvojijo že v predšolskem obdobju. Primere matematičnih dejavnosti, ki pokrivajo cilje, s katerimi usvojijo te koncepte, najdemo v Kurikulumu za vrtce (1999). Matematične dejavnosti so razdeljene v dve skupini, in sicer za otroke od 1. do 3. leta in otroke od 3. do 6. leta. Že na prvi pogled lahko vidimo, da so dejavnosti za otroke od 3. do 6. leta obširnejše in številčnejše. Kljub temu je presenetljivo veliko matematičnih dejavnosti predlaganih že v prvem starostnem obdobju. Že v tem obdobju otroci npr. štejejo, vendar na preprostejši način. V drugem obdobju napredujejo v aritmetiki in cilji so precej zahtevnejši. Pri štetju se že pojavi štetje nazaj,, spoznajo število nič, igrajo se z žepnim računalom, odštevajo, seštevajo itd. Viden preskok se kaže tudi na drugih področjih, predvsem pri geometriji. Otroci v drugem obdobju spoznajo kopico novih pojmov, ki so zanje kar zahtevni (nagnjeno, poševno, krivo, rob, vogal ...). Prav tako spoznajo imena za like in telesa ter opazujejo še mnogo drugih oblik (Kurikulum za vrtce, 1999, str. 65-72). b Aritmetika in algebra Povzetek pomembnejših raziskovalnih ugotovitev s področja razvoja pojma število najdemo v delu Manfreda Kolar (2006). Ugotovljeno je bilo, da so otroci zmožni opredeliti moč množice, če je ta dovolj majhna. Otroci v prvem starostnem obdobju po navadi štejejo glasno in počasi. Šele pri štirih letih lahko število opazovanih predmetov povedo že po eni sekundi (2-3 predmeti). Tuje raziskave potrjujejo, da predšolski otroci že štejejo z razumevanjem. Pri tem sledijo trem principom štetja: princip povratno enoličnega prirejanja (bijekcija), princip urejenosti in princip kardinalnosti. Prvi pomeni, da otrok vsakemu od preštevanih predmetov priredi natanko eno ime oz. besedo za število. Po drugem principu otrok pri štetju uporablja besede v vedno enakem vrstnem redu. Pri tretjem principu je lahko otrok uspešen le, če razume prva dva. Otrok, ki ima usvojen princip kardinalnosti, se zaveda, da zadnja izrečena beseda (ime za število) pomeni moč preštevane množice. Princip kardinalnosti naj bi otroci usvojili nekoliko pozneje. Pri predšolskih otrocih pogosto ne vemo, ali sploh vedo, kaj vse lahko štejejo. Nekaj raziskav je pokazalo prav to, da otroci ne prepoznajo vseh stvari kot števne. Kot števne najbolj prepoznavajo predmete, ki so si po- 06 Struktura zastopanosti matematičnih dejavnosti v slovenskih vrtcih dobni. Podobni so si npr. po barvi ali obliki. To so predmeti, ki imajo neke skupne lastnosti. Starejši so otroci, bolj raznoliki so lahko predmeti, ki jih prepoznavajo kot števne (Manfreda Kolar, 2006). Otroci spoznavajo števila skozi različne igre. Za občutek opišimo igro, ki jo bomo imenovali Štej v krogu. Igramo jo v manjši skupini (4-5 otrok). Potrebujemo približno 40 drobnih predmetov, ki so si podobni (npr. link kock, storžev, gumbov ...). Razporedimo jih drugo zraven drugega v obliki kro-žnice. Vmes vrinemo približno 5 drugačnih predmetov (npr. predmetov druge barve ali oblike), ki jih bomo imenovali vsiljivci (Slika 1). Otrok vrže običajno igralno kocko, glasno pove število pik, ki je padlo, in se odloči, kje bo začel šteti. Šteti lahko začne kjer koli, razen na vsiljivcih. Z dotikanjem šteje predmete. Predmet, pri katerem se štetje ustavi, vzame s krožnice in ga obdrži, če se pri štetju ni dotaknil vsiljivca. V nasprotnem primeru štetja ne more nadaljevati in je na vrsti nasle- dnji otrok, ki ponovno vrže kocko in se postopek ponovi. Zmaga otrok, ki zbere največ predmetov. Po nekaj igrah otroci ugotovijo, da je treba štetje vedno začeti na primernem mestu daljšega krožnega loka, saj tako ne pridemo do vsiljivca. Področje aritmetike in algebre, kamor sodijo tudi vzorci, ponuja v predšolskem obdobju zelo raznolike dejavnosti. Predloge lahko najdemo v Golob (2007). Vzorce lahko prikažemo na različnih predstavitvah, vendar v predšolskem obdobju poudarjamo konkretno izkušnjo, ki je najpogosteje gibalna ali glasovna. Otroci lahko oblikujejo vzorec s ploskanjem in s tleskanjem ali postavitvijo v vrsto (npr. vzorec z osnovnim gradnikom AABC lahko predstavijo s postavitvijo stoj--stoj-sedi-most, stoj-stoj-sedi-most (Slika 2). Mogoča je tudi glasovna predstavitev z različnimi »naravnimi« zvoki (ko-ko-dak, ko-ko-dak je glasovna predstavitev vzorca z osnovnim gradnikom AAB) ali z uporabo instrumentov. [Slika 1] Začetna postavitev in vmesni položaj pri igri Štej v krogu. Osnovni gradnik Konkretno/enaktivno Slikovno/ikonično AB Živi vzorec (deček-deklica-deček-deklica) ABB Zvok: Tra-la-la tra-la-la Gibanje: Počep-poskok-poskok počep-po-skok-poskok AABC [Slika 2] Vzorci Y Geometrija in merjenje Raziskav s področja predšolske geometrije in merjenja je bistveno manj kot raziskav s področja razvoja aritmetičnih pojmov. Pri razvoju geometrijskih pojmov sledimo Van Hiele-jevim stopnjam (Van de Walle, 2004), pri čemer je predšolski otrok na vizualizacij-ski stopnji, kar pomeni, da geometrijske oblike dojema kot ločene enote in jih opisuje po njihovih zunanjih značilnostih (npr. rdeč in moder pravokotnik sta zanj popolnoma različna lika). Objekt misli so torej oblike in opis tega, »kako so videti«. Rezultat razmišljanja otrok v tem obdobju pa so ugotovitve o razredih oblik, ki so si podobne. Otrok prepozna in poimenuje like na podlagi celostnega pristopa. Za otroke je na tej stopnji kvadrat poimenovan kvadrat zato, ker je »videti kot kvadrat«. Ker je videz na tej stopnji dominanten, lahko preglasi lastnosti, ki določajo obliko. Npr., za 45° rotiran kvadrat za otroke ni več kvadrat, ampak se spremeni v »karo«. Otroci na tej stopnji klasificirajo in urejajo oblike glede na njihov videz. Npr.: »Ta dva bom položil skupaj, ker sta koničasta/debela/ videti kot hiša.« Z osredinjanjem na videz so otroci sposobni iskati podobnosti in razlike med oblikami. Kot rezultat lahko ustvarijo in razumejo razrede oblik. Predvideni dejavnosti na tej stopnji sta urejanje in klasificiranje ter sestavljanje in razstavljanje oblik. Kot pripomočka sta za sestavljanje in razstavljanje najbolj znana tangram in ploščice za vzorč-ke. Tangram (slika 3), ki je v osnovni obliki za predšolsko obdobje pretežak, najdemo tudi v prirejeni obliki za zgodnje matematično obdobje (Lipovec in Štukl, 2010). 08 Struktura zastopanosti matematičnih dejavnosti v slovenskih vrtcih Ploščic za vzorčke (ang. pattern blocks), žal v naših vrtcih ne najdemo dovolj pogosto. Poleg proste igre sestavljanja, oblikovanja in nadaljevanja vzorcev ta pripomoček ponuja tudi možnost ponazorila za usvajanje prvih konceptov ulomkov (slika 4). S tem posegajo tudi v aritmetiko. Clements in Sarama (2007) v evalvaci-ji projekta Building Blocks ugotavljata, da sestavljanje dvodimenzionalnih oblik pomembno vpliva na razvoj geometrijskih pojmov. Ugotovljeno je bilo, da je vpliv na razvoj geometrijskih konceptov močnejši pri sestavljanju dvodimenzionalnih oblik kot pri 9 drugih dejavnostih te stopnje (npr. opisovanje oblik in njihovih lastnosti, klasifikacija, nadaljevanje geometrijskih vzorcev, merjenje). 5 Druge vsebine V slovenskih vrtcih so, poleg dejavnosti s področja aritmetike in algebre ter geometrije in merjenja, tudi dejavnosti s področja drugih vsebin, najpogosteje s področja obdelave podatkov oz. prikazovanja z različnimi diagrami. Rozmanova (2009) podrobneje opisuje to področje in opozarja na terminološko zmedo ter ugotavlja, da se mnogi prikazi podatkov, ki so znani po svetu, v Sloveniji še niso dovolj uveljavili. Ugotavlja tudi, da lahko »grafične prikaze bogato in vsestransko uporabljamo in vpletamo ne le v izključno matematične dejavnosti, ampak tudi na vsa druga področja dejavnosti v predšolski vzgoji.« (Rozman, 2009, str. 228). Čeprav je znanih nekaj raziskav v matematiki v predšolskem obdobju, se te običajno osredotočijo na eno vsebinsko področje. Redko gre za pregledno raziskavo, ki bi pokazala, katero matematično področje prevladuje. Prevladuje mnenje, da je v osnovni šoli geometrija zapostavljena na račun aritmetike. Kako pa je v predšolskem obdobju? Z našo raziskavo smo želeli osvetliti to področje in vsaj deloma odgovoriti na zastavljeno vprašanje. Namen naše raziskave je torej ugotoviti matematične dejavnosti v slovenskih vrtcih in raziskati, katera matematična področja so najpogosteje zastopana. e Metodologija Kot vir podatkov smo vzeli poročila, ki so jih v okviru integrirane prakse oddali študentke in študenti tretjega letnika programa Predšolske vzgoje na Pedagoški fakulteti Maribor, v študijskih letih 2009/2010 in 2010/2011. Njihova poročila so poleg drugih sestavin vsebovala tudi dnevnik matematičnih dejavnosti. Gre za dnevnike, ki so jih študenti pisali med opravljanjem prakse v vrtcu. Dnevniki so nastajali, ko so študenti in študentke obiskovali predmet Metodika matematike, ki poteka v zimskem semestru. Prav takrat so študenti enkrat na teden preživljali čas v vrtcu, v okviru integrirane prakse. Namen prakse je, da študent spozna vrtec kot ustanovo, delo vzgojitelja in težave pri tem delu ter da preizkuša svoje sposobnosti in pridobljena znanja med študijem. Pri praksi študent bolj ali manj samostojno (pod mentorstvom) vodi oddelek in s tem preveri svojo usposobljenost za delo. Za oblikovanje zapisa v dnevnik matematičnih dejavnosti si je vsak študent v vrtcu izbral enega otroka. Opazoval ga je pri vseh dejavnostih z matematičnimi cilji in vsa svoja opažanja, ugotovitve in vtise zapisal. Dnevnik je vseboval spol in starost opazovanega otroka, zapise o datumu (kdaj je katera matematična dejavnost potekala), kratek opis poteka določene matematične dejavnosti in okviren čas njenega trajanja. Prav tako so v dnevnik zapisali tudi svoja opažanja o zanimanju otroka za dejavnost (ali otrok uživa v njej, se dolgočasi ...), spretnosti (ali otrok ob dejavnosti razvija določene spretnosti, veščine, ali je pri dejavnosti uspešen .) ter otrokovo razmišljanje in izražanje. Ob prebiranju dnevnikov matematičnih dejavnosti smo ugotovili, da gre za bogat vir podatkov o matematičnih dejavnostih v predšolski vzgoji, zato smo se odločili, da jih podrobneje raziščemo. V raziskavi smo se osredotočili na zapise, ki so se nanašali 010 Struktura zastopanosti matematičnih dejavnosti v slovenskih vrtcih na opis matematičnih dejavnosti. Pregledali smo 130 matematičnih dnevnikov in izpisali vse matematične dejavnosti ter jih klasificirali po matematičnih področjih. Podatke smo obdelali z uporabo programskega svežnja SPSS za obdelavo in analizo podatkov. Z Rezultati in interpretacija Izkazalo se je, da v vrtcih poteka veliko matematičnih dejavnosti. Nnašteli smo skupaj 800 matematičnih dejavnosti v 130 dnevnikih. Nekatere so preproste (npr. preštevanje), druge pa so relativno zapletene (npr. zbiranje podatkov in grafični prikaz). Analizirali smo tudi razdelitev dejavnosti po sklopih. Vse smo klasificirali po matematičnih področjih, ki jih predlaga osnovnošolski učni načrt (aritmetika in algebra, geometrija in merjenje ter druge vsebine). Znotraj aritmetike in algebre smo oblikovali podrazrede: štetje, števila, računske operacije in vzorci. Dejavnosti verbalnega preštevanja (npr. koliko otrok je danes prišlo v vrtec) smo ločili od dejavnosti, ki števila predstavljajo z relacijami (več - manj) ali prikazujejo s številko (simbolnim zapisom). Pri geometriji in merjenju smo razen različnih geometrijskih oblik (liki in telesa) ter različnih količin (masa, dolžina, prostornina ...), oblikovali tudi skupino relacij (orientacija) in transformacij (skladnost in zrcaljenje). Znotraj drugih vsebin smo ločili tri velike sklope, in sicer klasifikacijske in seriacijske dejavnosti, ki ne posegajo neposredno v razvoj koncepta števila, kombinatorične situacije in različne grafične prikaze (npr. Carrollov, Euler Ven-nov, drevesni prikaz ...). Področje Vsebina f f % Aritmetika in algebra Štetje 253 31,6 % Števila in številke 103 12,9 % Računske operacije 29 3,6 % Vzorci 38 4,8 % Skupaj 423 52,9 % Geometrija in merjenje Liki 69 8,6 % Telesa 17 2,1 % Skladnost 4 0,5 % Zrcaljenje 0 0 % Orientacija 32 4,0 % Dolžina 10 1,3 % Prostornina 2 0,3 % Masa 11 1,4 % Čas 17 2,1 % Denar 15 1,9 % Temperatura 1 0,1 % Skupaj 178 22,3 % Druge vsebine Kombinatorika 0 0 % Seriacija 63 7,8 % Klasifikacija 118 14,7 % Stolpčni diagram 9 1,1 % Vrstični diagram 3 0,4 % Carollov diagram 0 0 % Euler-Vennov diagram 3 0,4 % Drevesni diagram 0 0 % Puščični diagram 3 0,4 % Skupaj 199 24,8 % Skupaj 800 100 % [Tabela 1] Število (f) in strukturni odstotki (f %) matematičnih aktivnosti Na področju aritmetike in algebre smo zaznali 423 dejavnosti (52,9 %), od tega je 385 (48,1 %) zaznanih dejavnosti v aritmetiki in le 38 (4,8 %) v algebri. Na področju geometrije in merjenja smo zaznali 178 dejavnosti (22,3 %), od tega 112 (14,0 %) dejavnosti v geometriji in 66 dejavnosti (8,3 %) na področju merjenja. S področja drugih vsebin je bilo zaznanih 199 dejavnosti (24,8 %), od tega so zelo prevladovale dejavnosti seriacije in klasifikacije, ki jih je bilo skupaj kar 181 (22,5 %). Dejavnosti s področja kombinato-ričnih situacij ni bilo zaznati, vsi prikazi pa so bili zastopani le s 15 dejavnostmi (2,3 %). Čeprav je bilo pričakovati, da bo največ dejavnosti na področju aritmetike, pa je skrb vzbujajoče, da je geometrija zastopana le s 14 odstotki. Podobno velja za merjenje in grafične prikaze, na račun večjega deleža seria-cijskih in klasifikacijskih dejavnosti. Kot je že omenjeno, največji delež predstavljata aritmetika in algebra, več kot polovico vseh dejavnosti. Štetju je namenjenih največ dejavnosti (31,6 %), sledijo števila (12,9 %). Že pri analizi poročil smo zaznali, 50 4D 10 Hi C Arilmetikd L promrtniii C nnofjenlf ■ mladji In Hflilflkaclla C pnkdii [Prikaz 1 ] Zastopanost matematičnih dejavnosti po področjih v odstotkih. da se največ dejavnosti nanaša na štetje, kar je seveda razumljivo, saj je štetje začetni koncept, ki ga je pričakovati v zgodnjem obdobju otroka. Presenetljivo malo je bilo zaznati dejavnosti za usvajanje začetnih konceptov računskih operacij (3,6 %) in vzorcev (4,8 %). Ko otroci vstopajo v devetletno osnovno šolo, v 1. razredu računajo do 10 in na konkretni ravni celo do 20. Učitelji gradijo na predhodnih izkušnjah, in pričakovati je, da bodo otroci izkušenjsko že seznanjeni z osnovnimi strukturami seštevanja in odštevanja (del-del-celota, dodajanje oz. odvze- 012 Struktura zastopanosti matematičnih dejavnosti v slovenskih vrtcih manje in primerjanje). Čeprav otroci lahko pridobivajo neformalno znanje tudi zunaj ustanov (npr. doma), pa raziskave kažejo na prednosti vključevanja otrok v predšolske ustanove tudi pri znanju. S tega vidika se od vrtca pričakuje, da otrokom ponudi struktu-rirane dejavnosti s predkoncepti računskih operacij, osnova za nadaljnjo gradnjo znanja računskih operacij, ki jih imamo za eno izmed elementarnih matematičnih znanj. Če se osredotočimo na geometrijo in merjenje, lahko takoj opazimo, da izstopajo dejavnosti, povezane z liki, ki predstavljajo skoraj polovico dejavnosti s tega področja. Podrobnejša analiza je pokazala, da gre veliko primerih samo za štiri like, in sicer krog, kvadrat, pravokotnik in enakostranični trikotnik. Žal se s tem spodbuja slika koncepta skozi prototip, ki je pozneje lahko omejujoča. Dejavnosti, ki jih predlaga Van de Walle (2007), vključujejo približno 20 raznolikih likov (raznostranični trikotnik, pravokotni in topokotni trikotnik, paralelogram, trapez, konkavni večkotniki in krivočrtni liki). Kljub temu smo z dejavnostmi na področju likov lahko zadovoljni v primerjavi s količino dejavnosti, ki so namenjene usvajanju začetnih konceptov geometrijskih teles. Telesa, skladnost, zrcaljenje, dolžina, prostornina, masa, čas, denar in temperatura so v vsakdanjem življenju zelo pomembni , zato bi moral biti po našem mnenju v vrtcih večji poudarek na teh vsebinah. V zgodnjem obdobju je predlagan pristop »od telesa proti točki«, kar bi se v našem primeru moralo pokazati v več dejavnostih, povezanih s telesi. Žal ni tako, saj smo zaznali približno štirikrat manj dejavnosti s telesi kot dejavnosti z liki. Prav tako manjkajo dejavnosti s področja transformacij, zasledili smo jih namreč le štiri, pri čemer je šlo le za razvoj koncepta skladnosti, zrcaljenja oz. kakšne druge simetrije pa ni bilo zaslediti. Merjenje je starodavna veja matematike, ki ima izrazito izkušenjski vidik in je po naravi povezana z vsakdanjim življenjem. Zaradi tega bi pričakovali veliko dejavnosti s tega področja. Izkazalo se je, da je dejavnosti s področja merjenja le 66 (8,3 %), pri čemer prevladujejo dejavnosti, povezane z denarjem, časom in dolžino, prostornina in temperatura pa sta komaj zastopani (npr. samo ena dejavnost je bila povezana s temperaturo). Razporeditev je deloma v skladu z dognanji razvojne psihologije o zaporedju razvoja konzervacijske sposobnosti (Labi-nowicz, 1989, str. 19), kjer je zapisano, da se najprej razvije konzervacija števila (ki ga lahko najdemo znotraj količine denar), nato sledijo dolžina, prostornina in masa. Če se osredotočimo še na tretje področje. Tudi tukaj lahko zaznamo precej večje število nekaterih dejavnosti, druge pa veliko manj. Na tem področju izstopata dve skupini dejavnosti, in sicer klasifikacija (14,7 %) in seriacija (7,8 %). Videti je, da sta klasifikacija in seriacija dejavnosti, ki se zdita vzgojiteljem v vrtcu pomembni in jima zato namenijo precej pozornosti. Potem opazimo, da so področja, ki se sploh ne pojavljajo. Gre za dejavnosti, povezane s kombinatoriko, ter dve posebni vrsti grafičnih prikazov (Carrollov diagram in drevesni diagram). Čeprav nismo pričakovali mnogih dejavnosti s področja kombinatorike, nas je presenetilo, da nismo zaznali niti ene same, čeprav je nabor zelo raznolik in za otroke zanimiv (npr. Na koliko načinov lahko oblečemo medvedka, če ima modro in rdečo majico ter dolge in kratke hlače?). Čeprav se je v zadnjem času grafičnim prikazom namenjlo veliko pozornosti, se žal zdi, da jih je v slovenskih vrtcih še vedno zelo malo; če se že pojavijo, pa gre za stolpični ali vrstični prikaz. Za konec Z rezultati smo lahko le deloma zadovoljni. V vrtcu se izvaja veliko matematičnih dejavnosti, vendar pa je energija enostransko preveč usmerjena na določena področja. Čeprav je aritmetika nesporno temeljna in pomembna, pa se zdi, da sta tudi v vrtcu močno zapostavljena geometrija in merjenje. Poleg tega je premalo povezovanja z vsakdanjim življenjem, tudi medpredmetnosti skorajda ni za- e Literatura slediti. Omenimo še, da nas je presenetilo, da je učenja skozi igro razmeroma malo oz. da se nekatere dejavnosti obravnavajo kot igra, čeprav nimajo značilnosti le-te. Vzgojiteljica npr. meni, da se otroci »igrajo«, čeprav po navodilih odrasle osebe izvajajo razmeroma nezanimivo dejavnost (npr. preštevanje avtomobilčkov). Morda smo zato ponekod v poročilih zasledili tudi, da otroci nočejo izvajati matematičnih dejavnosti ali pa niso pokazali zanimanja za igre z matematičnimi cilji in so se raje igrali s svojimi igračami, kljub veliki motivaciji vzgojiteljice ali študentke/študenta, ki je otroka opazoval. 1. Clements, D. H. in Sarama, J. (2007). Effects of Preschool Mathematics Curriculum: Summative Research on the Building Blocks Project. Journal for Research in Mathematics Education, 38 (2), 136-163. 2. Clements, D. H. (2006). Part one: Major themes and recommendations. V Douglas H. Clements, Julie Sarama in Ann Marie DiBiase (Ur.), Engaging young children in mathematics: Standards for early childhood mathematics education (pp. 1-72). Mahwah, NJ: LEA. 3. Golob, A. (2007). Celostni pristop pri matematiki - vzorci skozi glasbo. Diplomsko delo, Maribor: Univerza v Mariboru, Pedagoška fakulteta. 4. Labinowicz, E. (1989). Izvirni Piaget. Ljubljana: Državna založba Slovenije. 5. Lipovec, A., in Štukl, M. (2010). Uporaba tangrama pri pouku matematike na razredni stopnji. Revija za elementarno izobraževanje 3 (1) 43-52. 6. Manfreda Kolar, V. (2006). Razvoj pojma število pri predšolskem otroku. Ljubljana: Pedagoška fakulteta. 7. Rozman, J. (2009). Grafični prikazi podatkov v vrtcu. Diplomsko delo, Maribor: Univerza v Mariboru, Pedagoška fakulteta. 8. Van de Walle, J.A. (2004). Elementary and Middle School Mathematics. Teaching Developmentally. NY: Pearson. 9. Kurikulum za vrtce. (1999). Ljubljana: MŠŠ in ZRSŠ. 014 Struktura zastopanosti matematičnih dejavnosti v slovenskih vrtcih Obdelava podatkov v začetnem šolskem obdobju Data processing in the initial school period I Povzetek Prispevek zajema obdelavo podatkov v prvih letih osnovnošolskega programa. Vključuje prikaz konkretnega materiala za usvajanje novih vsebin. Opisana je učna priprava in potek učne ure prikaza podatkov. Ključne besede: obdelava podatkov, kombinatorika, verjetnost, statistika. Natalija čerček, OŠ borcev za severno mejo, Maribor I Abstract The article presents data processing in the first few years of primary school program and includes a presentation of concrete material for adoption of new learning contents. It also describes the preparation of a lesson plan and the course of lesson dedicated to the presentation of data. Keywords: data processing, combinatorics, probability, statistics. a Uvod Obdelava podatkov zajema prvine statistike, kombinatorike in verjetnosti. Z obdelavo podatkov se otroci srečajo že pred stopnjo konkretnih operacij, pred 7. letom starosti. Učenci v osnovnošolskem obdobju spoznajo Matematika v šoli ~ XVIII. [2012] ~ 015-023 delo s preglednicami, diagrami in anketami iz vsakdanjega življenja. Znanja s področja obdelave podatkov so odlična osnova za reševanje matematičnih problemov in iskanje strategij reševanja le-teh. Spodbujajo ustvarjalnost, inovativnost, izvirnost in divergentno mišljenje. Področje obdelave podatkov nam odpira svobodo ustvarjanja, kjer lahko uresničimo številne nenavadne, nove zamisli, asociacije. Učenec je znotraj učne ure dejaven subjekt, ki opazuje, ustvarja, raziskuje, daje predloge in logično sklepa. b Obdelava podatkov v začetnem obdobju osnovne šole Učni načrt za matematiko vključuje že v prvih letih osnovnošolskega programa začetne osnove obdelave podatkov. Učenci dosegajo znanja v izkušenjskem svetu, na konkretnem nivoju (Ministrstvo za šolstvo, znanost in šport, 2006, 2011). V prvem razredu razvrščajo predmete, telesa, like glede na eno izbrano lastnost, odkrijejo in ubesedijo lastnost, po kateri so razvrščali, razvrstitev elementov pokažejo z različnimi diagrami (puščičnim, Carrollo-vim, Euler-Vennovim), podatke prikažejo s preglednico, s stolpci in figurnim prikazom. S temi vsebinami učitelj spodbuja otrokov kognitivni razvoj. Ciljem prvega razreda se v drugem pridružijo še preproste kombinato-rične situacije, kjer nastavljajo in preštevajo vse možne izide, v tretjem pa preproste kom-binatorične situacije predstavijo grafično, s preglednico in kombinatoričnim drevesom (Cotič, 1999). Preden se lotimo obdelave nekega problema, se z učenci pogovorimo in tako dobimo njihove ideje, ugotovimo predznanje, ki ga imajo, nato izhajamo iz njihovih izkušenj, njihovega »znanega« sveta. Skupaj naredimo načrt, kaj bo naš problem, katere podatke potrebujemo, kako bomo zbrali potrebne podatke, kako bomo obdelali podatke, kako bomo podatke prikazali in jih na koncu dejavnosti interpretirali. Tako se učenec nauči, da »ubesedi« problem, zna izluščiti podatke, ki ga zanimajo in jih potrebuje, z njimi operira in debatira o dobljenih rezultatih. Z obdelavo podatkov lahko otroci veliko izvejo drug drugem, o družinah in ljubljenčkih, času odhoda v šolo, telesni višini, o tem, kaj imajo radi in česa ne ... Z odgovorom učenca na preprosto vprašanje dobimo pomemben podatek, ki prispeva k celoti. Če je . * - J» rt ;.f> - i I/ W J^ [Slika 1] Stolpčni prikaz (Moj hišni ljubljenček) [Slika 2] Tortni prikaz (Moj hišni ljubljenček) 016 Obdelava podatkov v začetnem šolskem obdobju mogočih več odgovorov, je dobro, da omejimo število možnosti. Ugotavljamo lahko priljubljenost (najljubši sadež, letni čas, barva, ljubljenček, vrsta sladoleda, nogometno moštvo ...), število (ljubljenčkov, družinskih članov, ur uporabe računalnika, ur spanja ...) in izvajamo meritve (višina, dolžina roke, skok v daljino, dnevne temperature, dolžina sence, količina padavin ...). Učenci izvejo, kakšni so kot razred in kako sami kot posamezniki sovpadajo s celoto. Ob spoznavanju okolice šole lahko učenci ugotavljajo število trgovin, gasilskih postaj, stolpnic, parkov ... Učenci ne primerjajo le podatkov znotraj svojega razreda, ampak lahko naredijo tudi primerjave med različnimi razredi. Najprej se moramo z učenci dogovoriti, katere podatke bomo obdelali. Če vzamemo na primer hišne ljubljenčke, se jih lahko odločimo razvrstiti po barvi, številu tačk ali velikosti. Najbolje je, da sedimo na preprogi z učenci v krogu in skupaj razpravljamo o idejah, mnenjih in skupaj določimo kriterije razvrščanja. Učence lahko izzovemo k razmišljanju s preprostim vprašanjem: Kaj bi rad izvedel o hišnih ljubljenčkih? Iz učencev dobimo nevihto predlogov, ki se navezujejo na zunanji videz hišnih ljubljenčkov, oglašanja, njihovih pre- hranjevalnih in bivalnih navad. Iz izkušenj lahko povem, da učenci najraje obdelujejo podatke, ki se nanašajo na to, kar imajo sami radi in kar jim je všeč. Primere najdemo pri hrani (najljubša sladica, jed, napitek), na področju oblačenja (najljubše oblačilo, obutev), iščemo najljubšo barvo, risanko, igro ... Če vzamemo na primer, da učenci izbirajo med najljubšo uporabo športnega pripomočka za prosti čas (kolo, rolerji, skiro) ali najljubšim zimskim športom (smučanje, drsanje, deska-nje, sankanje), po končani učni uri trdijo, kako »lepo« smo se danes »igrali«. Področje obdelave podatkov nam torej odpira svobodno otroško igrišče, kjer se otroci sproščeno igrajo. Prikazanih je nekaj primerov, kako lahko hitro obdelamo podatke in material uporabimo večkrat. Vsakič pripravimo le simbol, učenci pa označijo svoj izbor bodisi s ščipal-ko, zamaškom ali znakcem (Van de Walle, Lovin, 2006). [Slika 3] Stolpčni prikaz (navpično, vodoravno) 17 a) Kombinatorika Kombinatorične situacije predstavimo preko konkretne (enaktivne) ravni, slikovne, grafične (ikonične) do simbolne ravni. V enaktivnem nivoju je zastavljeno izhodišče problemske situacije, ki jo analiziramo in izvedemo konkretno dejavnost. Dejavnost shematiziramo z risbo, s skico in sistematičnimi prikazi v ikoničnem nivoju. V simbolnem nivoju dejavnost prikažemo v abstraktni obliki. Otroci si že v predšolskem in zgodnjem šolskem obdobju izoblikujejo osnovne pojme iz kombinatorike in jih uporabljajo v novih situacijah (Cotič, 1999). Primer kombinatorične situacije, ki sem ga izvedla z učenci tretjega razreda: Učenec si lahko izbere za popoldansko malico napitek in sadež. Na voljo so trije različni napitki (čaj, mleko, sok) in dva sadeža (jabolko, banana). [Slika 4] Kombinatorično drevo [Slika 6] Preglednica 018 Obdelava podatkov v začetnem šolskem obdobju Izbor lahko prikažemo z razpredelnico, puščičnim diagramom ali kombinatoričnim drevesom. Vseh možnosti je m • n, torej 3 • 2 = 6. b) Verjetnost Pri teh vsebinah, tako kot pri kombinatoriki, ostanemo v izkustvenem svetu. Učenec pridobiva izkušnje z naključnimi dogodki, jih zapisuje s preglednico ali histogramom, opiše, kaj je mogoče in kaj nemogoče, loči, kaj je zanesljiv, naključen in nemogoč dogodek. Napovejo lahko svoje mnenje, ali bo danes deževalo, ali bo predmet potonil ali plaval na gladini, ali bodo jutri manjkali štirje učenci pri pouku, ali bo sonce vzšlo naslednji dan itd. Primer meta kocke: zagotovo bo eno-mestno število, nemogoče je, da bo sedmica, mogoče bo štirica. Primer vrečke, v kateri je 30 modrih kock in dve beli: Malo verjetno je, da bomo izvlekli belo kocko, zelo verjetno, a ne zagotovo, je, da bomo izvlekli modro kocko. Učenci ugotavljajo, kje je večja verjetnost točne napovedi dogodka, pri metu kovanca, če sta možnosti števka in slika, ali metu šestih kock in zadeti same šestice (Cotič, 1999). c) Statistika Y Učna ura matematike, obdelava podatkov, 2. razred Sledi prikaz učne ure matematike, kjer smo z učenci drugega razreda prikazovali podatke s stolpčnim, tortnim in Carollovim diagramom. Učna tema: Prikazi podatkov Učna enota: Prikaz odnosov in lastnosti s stolpčnim, tortnim in Carollovim diagramom Vzgojno-izobraževalni cilji: - učenci ugotavljajo odnose in lastnosti s stolpčnim, tortnim in Carollovim diagramom, - izražajo odnose in oblikujejo izjave v naravnem in matematičnem jeziku, - znajo prikazati odnose s stolpčnim, tor-tnim in Carollovim diagramom, - znajo razbrati odnos, ki je prikazan s stolpčnim, tortnim in Carollovim diagramom, - razvrščajo podatke glede na dve lastnosti, - ugotavljajo odnose in lastnosti med ljudmi, - zapisujejo podatke in jih urejajo, - se navajajo na natančnost in sistematič-nost pri delu, - se navajajo na natančno opazovanje in logično sklepanje. Po 019 čem se razlikujemo? Pred tablo sem poklicala deklico in dečka. Iskali smo razlike (spol, lasje, narodnost .) in skupne lastnosti (učenca istega razreda, enake starosti, oba stojita na isti Zemlji, oba pokriva isto nebo . ). 2. Obravnava nove snovi Onkraj resničnega sveta obstaja domišljijski svet. Potopili smo se vanj. Nekoč ... nekje ... globoko v gozdu, kamor ne seže človeška noga, so živele zajčje družine. Kot vaši starši so očki zajci in mamice zajklje hodili v službo. Nabirali so hrano za svoje zajčke. Zajčki pa so obiskovali šolo. V vsaki zajčji družini je bilo različno število zajčkov. Vsako jutro si je učiteljica zajklja zapisovala, kdo je prišel v šolo. To preštevanje ji je vzelo veliko časa. Učence sem izzvala, naj predlagajo, kako bi jih učiteljica lažje preštela in ločila v tej gneči zajčkov, h kateri družini spada kateri zajček. Eden izmed predlogov je bil, da bi bili člani iste družine enako oblečeni. Ta odgovor je bil kot nalašč za nadaljevanje, saj sem pripravila rdeče, modre in rumene kapice. Pred tablo sem poklicala nekaj učencev in jim nadela kapice. Vsaka družina je imela svojo barvo. Učiteljica je zajčke hotela prešteti. Za nasvet je vprašala Zvezdico Zaspanko. Svetovala ji je, da naj za vsakega zajčka v tabelo naredi eno črtico. Učiteljica je zapisovala črtice po vrsti, kakor so zajčki prihajali. Zvezdici Zaspanki se je potožila, da se pogosto zmoti v štetju, saj je toliko črtic. Dobila je nov nasvet: Vsako peto črtico povleči čez štiri pred njo. Tako boš hitreje štela, koliko črtic je. [Slika 7] Stolpčni prikaz a) STOLPIČNI PRIKAZ Ob usmerjanju sem učence vodila, da so se spomnili, kako smo preštevali predmete (npr. telesa) in s čim smo si pomagali pri zapisu. Stolpčni prikaz poznajo že iz prvega razreda. Bi lahko tako zapisovala tudi učiteljica zajklja? Tako kot vi ji je svetovala tudi Zvezdica Zaspanka. Naredimo stolpce. Vsaki družini pripada en stolpec. Višina stolpca pa nam pove, katerih je več. Zajčki so položili svoje kartončke na ustrezen stolpec. b) TORTNI PRIKAZ Ker je učiteljici zajklji postalo zapisovanje podatkov na različne načine zabavno, je prosila Zvezdico Zaspanko, naj jo nauči še česa. Naročila ji je: Zajčki naj se postavijo v krog okoli palice. Vsi zajčki ene družine naj stojijo drug ob drugem. Vsi naj se držijo za roke. Učenci so z levo roko prijeli trak volne, ki visi s palice, z desno pa sosedovo levo roko. Zajčki so počepnili, da so si drugi učenci lahko ogledali nastalo situacijo. Hitro so prišli do spoznanja, da jih spominja na kose razrezane torte. Zajčki so dobili v roke svoj ustrezno obarvani kos torte in ga postavili na tla. 020 Obdelava podatkov v začetnem šolskem obdobju Takšnemu prikazu rečemo tortni prikaz. Vsak zajček ima svoj kos torte. Vsi skupaj predstavljajo celo torto. Ugotovili so, da ima naša torta 11 kosov, saj je 11 zajčkov. Vsak je svoj kos nato prilepil na pripravljeno »prazno« torto na tabli. c) CAROLLOV DIAGRAM Lotili smo se še drugačnega prikaza, Carol-lovega diagrama. Naredili smo dve koloni. V prvo so se postavili dečki in v drugo tisti, ki niso dečki. Brez težav so ugotovili, da so to deklice. Na tleh sem z debelejšim trakom vertikalno razmejila skupini. Znotraj vsake kolone smo dodali še eno lastnost. Vsak zajček je moral razmisliti, ali ima v svojem imenu črko i. Zajčki so se ustrezno postavili in na tleh sem vodoravno razmejila skupini s trakom. Vsak zajček je nato snel trakec s kape, na katerem je bilo napisano njegovo ime. Svoje ime je pritrdil na ustrezno mesto na tabli. 3. Utrjevanje Za utrjevanje znanja so rešili učni list. Na učnem listu so bile različne živali. Izpolnili so tabelo, stolpčni, tortni in Carollov diagram. > ■ f J V 13 r p [Slika 9] Carollov diagram 21 4. Sklep Učence sem po izvedeni učni uri pozvala, naj podelijo z menoj svoje vtise, občutja, naj podajo analizo dela. Za učence je tako izvedena ura, ko so sami glavni akterji obravnave, obdelave podatkov, analize, nekaj posebnega, igrivega, zanimivega in zabavnega hkrati. Učenci so po učni uri doživeto pripovedovali svojo »zajčjo« zgodbo. Pojasnili so, zakaj so imeli čepice, kako so pomagali učiteljici zajklji, kaj je pomenila višina stolpca, razložili so, kako so vrednotili po dva stolpca med seboj (več, manj), vse stolpce med seboj (največ, najmanj), razložili so, kako so razumeli »torto« in kaj je predstavljal kos torte, vlogo barve tortnih kosov ... Še posebej zanimiv se jim je zdel Carollov diagram. Zaradi tega, ker so igrali glavno vlogo v konkretnem nivoju, jim prehod na ikonični ni delal težav in so ga zlahka razumeli. Moje priporočilo učiteljem je, da se podobno lotijo obdelave podatkov, da izhajajo bodisi iz pravljičnega ali dejanskega, realnega sveta in od tu zajemajo podatke, s katerimi v uri operirajo, jih obravnavajo in obdelu- jejo, saj bodo otrokom blizu in bodo zlahka razvili empatijo problema. 5 Sklep Osnovno vodilo učitelja je učni načrt matematike. V njem je jasno zapisano, da je poleg geometrije in merjenja ter aritmetike in algebre namenjeno tudi drugim vsebinam vsako leto določeno število ur. Učenci se v prvem vzgojno-izobraževal-nem obdobju učijo iskanja potrebnih podatkov iz preglednic in prikazov (figurni, črtni, s stolpci), sami predstavljajo podatke v preglednicah in s prikazi, preiskujejo kom-binatorične situacije in jih grafično predstavijo, razporejajo elemente po različnih kriterijih in razporeditev prikažejo s prikazi (Carrollov, Euler-Vennov in drevesni prikaz) ter prikažejo in berejo odnos med elementi dveh skupin s puščičnim diagramom. Na kreativen način zbirajo in urejajo podatke iz vsakdanjega življenja, ugotavljajo odnose med njimi, jih analizirajo ter jih ustrezno vrednotijo (Ministrstvo za šolstvo, znanost in šport, 2006, 2011). 022 Obdelava podatkov v začetnem šolskem obdobju Pri usvajanju novih vsebin matematike so pomembni vsi tipi reprezentacij, enaktivni, ikonični, simbolični in verbalni. Učence je treba voditi skoznje postopoma in v doslednem zaporedju. Vedno bo enaktivna repre-zentacija pred ikonično in simbolno. V njej dobiva učenec izkušnje iz konkretnega, fizičnega sveta. Enaktivna reprezentacija (kon- kretni nivo) je osnova za razumevanje in dojemanje, tu se moramo ustaviti, ji nameniti čas in energijo. Od učitelja je odvisno, ali bo učenec ure matematike doživel kot suhoparno, nemi-kavno usvajanje novega ali pa se mu bo znotraj teh ur odprlo veliko igrišče, kamor bo rad zahajal in se vedno znova vračal. e Literatura 1. Učni načrt, program osnovnošolskega izobraževanja, Matematika, Ministrstvo za šolstvo, znanost in šport, Zavod RS za šolstvo, Ljubljana 2006. 2. Posodobljeni učni načrt za predmet matematika v osnovni šoli, Ministrstvo za šolstvo, znanost in šport, Zavod RS za šolstvo, Ljubljana 2011. 3. M. Cotič, Obdelava podatkov pri pouku matematike 1-5, Zavod Republike Slovenije za šolstvo, Ljubljana 1999. 4. J. A. Van de Walle, L. H. Lovin, Teaching student-centered mathematics: Grades k-3, Pearson/Allyn and Bacon, Boston, 2006. 23 w Matematika v podaljšanem bivanju Mathematics in after school care Lucija Gregorič OŠ Ivana Roba Šempeter pri Gorici I Izvleček Pogosto je mnenje staršev in pedagogov, da je podaljšano bivanje le varstvo otrok in čas za domače naloge. Lahko rečem, da ni tako. Zavzeti učitelji, ki se zavedamo svojega poslanstva, vemo, da je v podaljšanem bivanju (OPB) možno narediti veliko več, tako na vzgojnem kot na izobraževalnem področju. Ravno v tem času je priložnost za krepitev dobrih osebnostnih lastnosti, navezovanje prijateljskih vezi, reševanje raznih kon-fliktnih situacij in nenazadnje utrjevanje znanja, ki ga pridobijo v dopoldanskem času pri pouku. K temu bi rada dodala, da imamo učitelji z današnjimi številčno polnimi oddelki veliko manj možnosti za kakovostno opravljanje svojega dela, saj zaradi velikega števila otrok posvetimo veliko časa disciplini. V članku bom navedla primer utrjevanja matematičnega sklopa pri urah samostojnega učenja v enem tednu v tretjem razredu. Ključne besede: otrok, matematika, utrjevanje, didaktične igre. I Abstract Parents and pedagogues often hold the opinion that afterschool care is just childcare and time for homework. I can safely say that this is not the case. As committed teachers, we are well aware of our mission and know that much more could be done in afterschool care in the field of upbringing, as well as in the field Matematika v šoli ~ XVIII. [2012] ~ 024-029 of education. This is exactly the time, which is appropriate for the strengthening of good personal qualities, forming friendly ties, solving various conflict situations and, last but not least, consolidation of knowledge, gained in the forenoon time during lessons. I would also like to add that, as teachers of overfilled departments, we have less opportunities to perform our work well, since the large numbers of children force us to dedicate a great deal of our time to discipline. In the article, I will present an example of a week-long consolidation of the mathematical unit in third grade at lessons dedicated to self-learning. Keywords: child, mathematics, consolidation, didactical games. a Uvod Učenje je proces, ki poteka neprekinjeno. Vsaka ura je dragocena, še zlasti, če so aktivnosti prilagojene zmožnostim in interesom otrok, v prijetnem okolju in ugodni čustveni klimi. Pedagoški delavec v to vpne šolski kurikulum s splošnimi in operativnimi cilji. Za podaljšano bivanje v šoli so smernice jasne. Biti mora nadaljevanje in nadgradnja vzgojno-izobraževalnih ciljev pouka, vsebovati mora določene 4 dejavnosti, kot so samostojno učenje, sprostitvena dejavnost, prehrana in usmerjeno preživljanje časa. V procesu pa je treba upoštevati novejše poglede na učenje, ki učence pripravljajo na izzive prihodnosti. V času samostojnega učenja so učenci nosilci vzgojno-izobraževalnega procesa, naloga učitelja pa je, da jih motivira za delo, jim daje naloge, usmerja njihovo delo, svetuje ter v določenih primerih pomaga pri reševanju problemov oziroma nalog. Učiteljeva naloga je, da pri izboru nalog za samostojno učenje upošteva razvojne in individualne sposobnosti učencev ter cilje, ki jih želi doseči. Učence je treba v miselno aktivnost, samostojno organiziranje dela in učenje vpeljevati postopno in sistematično. Da jih bomo navadili na samostojnost, moramo poskrbeti za razvijanje učnih navad ob upoštevanju njihovih osebnostnih lastnosti. Pomembno je, da učence postopno navajamo na različne oblike učenja kot npr. individualno, v dvojicah, skupinsko učenje. Pri samostojnem učenju je treba upoštevati naslednja didaktična načela: - načelo miselne aktivnosti, - samostojnosti, - postopnosti, - sistematičnosti, - upoštevanje razvojnih in individualnih razlik med učenci ter socializacijski vidik. (Korenič, 2007) b Primer utrjevanja znanja matematičnega sklopa pri urah samostojnega učenja v podaljšanem bivanju v tretjem razredu 25 V podaljšanem bivanju delam že skoraj drugo desetletje, tako da imam na tem področju že kar precej izkušenj. Nekaj let sem tudi poučevala na podružnični šoli, zato lahko primerjam naravo dela med seboj in ugotovim, da je tudi delo v podaljšanem bivanju zelo specifično in zanimivo. Z učenci lahko ustvarimo marsikaj dobrega in koristnega za njihovo življenje, saj preživijo današnji šolarji večino časa v šoli. Na podlagi teh ugotovitev sem zato veliko razmišljala prav o učenju matematike oziroma utrjevanju znanja pred preverjanjem in ocenjevanjem v podaljšanem bivanju. Zastavila sem si izhodišče - naučiti učence učiti se s pomočjo različnega didaktičnega materiala individualno in v skupini. Zavedam se, da matematika ni že vnaprej dano in sklenjeno znanje, temveč vselej aktivno razmišljanje, zato naloga učitelja ni le prenašanje matematičnih vednosti, pač pa zbujanje in spodbujanje zanimanja, radovednosti in spoznavnih potreb. V prvih treh razredih je zelo pomembna konkretna raven ali konkretno-izkustvena dejavnost, saj je ena od obveznih stopenj v razvoju kognitivnih procesov. Pri oblikovanju matematičnih pojmov moramo zato imeti na voljo mnogo didaktičnega materiala, iger in igrač. Otroku naj bi bila didaktična sredstva vedno dosegljiva, tako, da jih lahko uporabi vedno, kadar sam začuti potrebo po tem. Poglavitne učne metode so predvsem: igra, opazovanje in izkušenjsko učenje. Z njimi bo otrok v prvem triletju usvajal različne matematične vsebine. Pri tem naj bi učitelj upošteval različne sposobnosti otrok ter pri njih spodbujal razvijanje pozitivnega odnosa do matematike in zaupanje v lastne matematične sposobnosti. Zaradi velikih razlik med sposobnostmi otrok je treba pri začetnem [Slika 1] Domino - množenje [Slika 2] Domino - deljenje pouku matematike posebno pozornost posvetiti individualizaciji in diferenciaciji postopkov in zahtev. (Cotič, 2004) V lanskem šolskem letu sem poučevala v 3. b razredu, v katerem je bilo vključenih 25 učencev v podaljšano bivanje. Z dopoldansko učiteljico sva bili v zelo tesni navezi, saj sva skupaj načrtovali in se dogovarjali za delo v razredu tako v dopoldanskem kot v popoldanskem času. Tako sva se odločili, da bova v okviru samostojnega učenja pri matematiki pred preverjanjem in ocenjevanjem uvedli tako imenovani MATEMATIČNI TEDEN. Klasičnih domačih nalog torej ni bilo. Na podlagi dejstev o pomenu igre in uporabi didaktičnega materiala sem pripravila in izdelala različne didaktične igre za matematični 026 Matematika v podaljšanem bivanju [Slika 3] Spomin - množenje, deljenje sklop poštevanka v okviru do 10 in deljenje (domino, spomin, sestavljenka, igra s kocko in karte). Didaktične igre: 1. Domino : a) množenje Pravila igre: Vse domine so obrnjene tako, da so števila in računi vidni. Nato prvi igralec vzame eno domino in naslednjo postavi zraven nje, ko izračuna dani račun množenja ali poišče pravi račun na dani rezultat. Enako naredi soigralec. Če opravi nalogo pravilno, igro nadaljuje naslednji igralec. Igra se konča, ko zmanjka domin. b) deljenje Pravila igre: Vse domine so obrnjene tako, da so števila in računi vidni. Nato prvi igralec vzame eno domino in nasle- [Slika 4] Sestavljanka 27 dnjo postavi zraven nje, ko izračuna dani račun deljenja ali poišče pravi račun na dani rezultat. Enako naredi soigralec. Če opravi nalogo pravilno, igro nadaljuje naslednji igralec. 2. Spomin Pravila igre: Kartončke z računi množenja, deljenja in rezultati razvrstijo po mizi tako, da so vidni samo oranžni krogci. Prvi igralec odkrije en kartonček, prebere račun, pove rešitev ter jo poskuša odkriti na enem izmed ostalih pokritih kartončkov. Če ga ne najde, položi oba kartončka v prvotni položaj. Enako nadaljuje naslednji igralec. Če igralec odkrije par, kartončka shrani pri sebi. 3. Sestavljanka Pravila igre: Na posameznih delih razrezane slike so na hrbtni strani napisani računi množenja in deljenja, na plakatu pa so napisani rezultati. Učenec vzame en del slike, prebere račun, ga izračuna in položi sliko na rezultat. Naslednji soigralec naredi isto. Tako sestavljajo sliko. 4. Igra s kocko Pravila igre: Kocka ima na vsaki stranici napisana 2 večkratnika in račun deljenja ali množenja. Ena stranica je prazna, na njej je narisan moder krog, kar pomeni »Pojdi naprej«. Učenec vrže kocko, izračuna dani račun ter pove račun deljenja ali množenja k danim večkratnikom. Če pove pravilne rešitve, vrže kocko naslednji igralec, sicer mora ponovno metati. Igra poteka v smeri urnega kazalca. 5. Igralne karte Pravila igre: Vodja skupine razdeli vsem soigralcem enako število kart. V smeri urnega kazalca vleče karto z računom prvi igralec in izračuna račun. Če pravilno izračuna, položi karto pred seboj, sicer jo obdrži in tako ima več kart. Na koncu zmaga tisti, ki ima več kart na svojem kupčku s pravilnimi rešitvami. O tednu matematike sem učence že predhodno pripravila, jim pokazala igre ter jih seznanila s pomenom takega tedna. Razdelila sem jih v pet heterogenih skupin, razred pa pripravila za tako obliko dela. Vsaka skupina je imela določeno vodjo, ki je skrbel za to, da so skupaj prebrali in razložili navodila posameznih iger. Skupine učencev so tako štiri dni v tednu preko iger utrjevale poštevanko in deljenje. Igre so krožile po vseh skupinah. Moja naloga pri tem je bila, da sem prehajala od skupine do skupine ter ugotavljala, ali snov posamezni učenci obvladajo. Opozarjala sem jih tudi na medsebojno pomoč in svetovanje. Zapisovala sem si opažanja ter o tem obvestila razredničarko. Na podlagi tega je lažje načrtovala ocenjevanje oziroma dodatno utrjevanje znanja v dopoldanskem času. Zadnji dan matematičnega tedna, v petek, pa smo zaključili s frontalno evalvacijo 028 Matematika v podaljšanem bivanju v obliki matematičnega kviza, ki je postal vrhunec tedna in hkrati dokaz, da delo ni bilo zaman, saj je večina učencev to snov odlično obvladala. Y Zaključek Opisala sem primer celotedenskega utrjevanja pomembnega matematičnega sklopa za nadaljnjo matematiko, ki sva ga v sodelovanju z dopoldansko učiteljico temeljito načrtovali. e Literatura Rezultati take oblike dela so se v končni fazi pokazali v odlično rešenih nalogah za ocenjevanje znanja. Cilj takega dela pa je prav gotovo, da so učenci začutili pripadnost skupini, odgovornost do nje, med seboj so se vzpodbujali in si nudili pomoč. Tudi starši so izrazili željo, da bi tudi pri drugih predmetih občasno oblikovali podoben teden. Dopoldanski učiteljici sem zelo hvaležna, da je v tako veliki meri pripravljena sodelovati in načrtovati delo tudi za popoldanski čas in mi prisluhniti. 1. Korenič M.; Podaljšano bivanje- življenje v šoli, Nova Gorica 2007 2. Cotič M., Felda D., Igraje in zares v svet matematičnih čudes. Kako poučevati matematiko v 3. razredu devetletne osnovne šole, DZS , Ljubljana 2004 29 • | ■ V • • V V Nic pomeni nic - nečesa* Nothing means nothing - of something I Povzetek Velik vpliv na učenje posameznikov ima poučevanje, ki ga izvaja učitelj. Če želimo izboljšati poučevanje in tudi učenje v šoli, morajo biti učitelji seznanjeni s kritičnimi lastnostmi posameznega koncepta, ki jih mora učenec zaznati in prepoznati, če ga želi razumeti. Ni namreč dovolj, da učitelj kritične lastnosti učencem zgolj poimenuje. Za učenje je veliko večjega pomena, da so odkrite v okviru dejavnosti, ki jih za učence zasnujejo učitelji. Namen tega preglednega članka je predstaviti razvoj koncepta števila nič v preteklosti, opozoriti na najpogostejša napačna razumevanja, ki se kot posledica različnih konceptov, napačnih predstav ... pojavljajo pri razumevanju koncepta števila nič po celotni vertikali osnovnošolskega izobraževanja, in tako opozoriti na kritične lastnosti, ki lahko učiteljem olajšajo načrtovanje dejavnosti v procesu poučevanja. Ključne besede: število nič, zahtevni matematični koncepti, razumevanje, poučevanje, učenje. Anja Janežič Osnovna šola Martina Krpana Ljubljana I Abstract Teaching performed by the teacher has a great influence on the learning of individuals. If we want to improve teaching and in consequence also learning in school, teachers must be acquainted with the critical properties of an individual concept, which the pupil must detect and recognize if he wants to understand Matematiaa v šo li ~ XVIII. [2012] ~ 02 ¿1-004 it. It is namely not enough that the teacher names the critical properties to the pupils. It is much more important for learning that they are discovered in the framework of activities, which are designed for pupils by teachers. The aim of the following synoptic article is to present the development of the concept of the number zero in the past to warn against frequent misunderstandings which are the consequence of different concepts, wrong notions.....and appear in the understanding of the concept of the number zero along the whole vertical of primary education. In this way I am trying to expose the critical proprieties, which can be of a great help to teachers in planning of activities in the teaching process. Keywords: number zero, demanding mathematical concepts, understanding, teaching, learning. a Uvod b Začetki števila nič V prvem poglavju svoje knjige Svet števil je Guedj (1996, str. 13) zapisal: »Človeštvo je za prehod od množine do števila potrebovalo dolgo dobo. Koncept števila, ki se nam zdi nekaj samoumevnega, je rezultat dolgotrajnega razvoja abstraktnega mišljenja.« Po njegovih besedah je pogoj za nastanek števila abstraktno gledanje na predmete, pri katerih upoštevamo zgolj obstoj, zanemarimo pa njihove lastnosti in posebnosti. Nekaj podobnega kot velja za nastanek števila, velja tudi za število nič. Za prehod od prvih števil do števila nič je človeštvo potrebovalo kar nekaj časa, saj je bilo, kot pravita Berlinghoff in Gouvea (2008), treba o številih razmišljati »kot o idejah, ki obstajajo tudi tedaj, ko ne preštevamo ničesar« (Berling-hoff in Gouvea, 2008, str. 83). Šele tedaj, pravita, »obravnavanje ničle kot števila postane povsem smiselno« (prav tam). Zgodovinarji so začetke ničle postavili v Mezopotamijo leta 1600 pred Kr. Babilonci so v tem času že poznali sistem mestne vrednosti. Prazno mesto, s katerim so pokazali, da je eno mesto izpuščeno, so najprej označevali s presledkom, ker pa so bili presledki zaradi hitrosti pisanja različno veliki, so jih pozneje nadomestili s piko. Z odkrivanjem ničle so nadaljevali Hin-dujci. Še pred letom 600 pred Kr. so poznali mestni desetiški sestav, kot ga imamo še danes. Za označitev praznega mesta so uporabili majhen krogec. Omenjeni sestav so se v 9. stoletju naučili uporabljati Arabci in pod njihovim vplivom se je v naslednjih dveh ali treh stoletjih postopoma razširil tudi v Evropo. Simboli za posamezne številke so se sicer nekoliko spremenili, načela pa so ostala ista. (Arabci so krogec uporabljali za označitev števila pet, za označitev praznega mesta so uporabljali piko.) Indijsko besedo za odsotnost količi- 31 ne, sunja, so Arabci prevedli z besedo sifr, Evropejci pa z latinsko zephirum, skupaj z le napol polatinjeno besedo cifra. Ti dve besedi sta se razvili v angleški besedi zero in cipher (ničla in številka). Še danes uporabljamo simbol števila nič (krog ali oval) za oznako tega, da katera potenca števila deset ni uporabljena. Hindujci so poleg tega, da so označili neko odsotnost količine, do 9. stoletja ugotovili še nekaj, in sicer, da je odsotnost količine tudi količina sama - nič so začeli obravnavati kot število (Berlinghoff in Gouvea, 2008). Tedaj se je začel razcvet aritmetike in algebre, o čemer priča tudi tale zapis: »Ničla se je do konca 18. stoletja iz pripomočka za označevanje praznega mesta razvila v pravo algebrsko orodje. Na poti do matematične dovršenosti, pa je bil v zvezi s številom nič storjen še en pomemben korak. Ko so matematiki v 19. stoletju posplošili strukturo sistema števil in v sodobno algebro uvedli kolobarje in obsege, je število nič postalo zgled posebnega elementa teh struktur. Dejstvo, da se število ne spremeni, ko mu prištejemo 0, in da množenje števila z nič zopet privede do števila nič, je postalo definicija »aditivne enote«, posebnega elementa teh abstraktnih struktur.« (Berlinghoff in Gouvea, 2008, str. 85) Y Nekaj prednosti uvedbe števila nič za razvoj aritmetike Ljudstva v preteklosti (Egipčani, Kitajci, Grki, Maji, Idijci, Azteki, Etiopci, Hebrejci in Rimljani) so poznala več različnih številskih sistemov. Zgodovinarji znanosti so jih razvrstili v tri skupine; aditivne (seštevalne), mešane (seštevalne in množilne) ter pozicijske ali mestne. Aditivni številski sistemi za izražanje števil uporabljajo operacijo seštevanja. Število se zapiše z nizanjem simbolov, vsota simbolov pa pomeni zapisano število. Mešani številski sistemi za izražanje števil uporabljajo dve operaciji: seštevanje in množenje. Seštevanje tudi tu deluje kot štetje vrednosti zaporednih enot. Množenje pa deluje po sistemu: dvesto = dve ■ sto je predstavljeno z dve, ki mu sledi sto. Nizanje števk ima pomen množenja (Guedj, 1996). Omenjena številska sistema sta povsem koristna za zapis števil (število 345 (CCCXXXXV) lahko v rimskem aditivnem številskem sistemu zapišemo brez težav), a imata nekaj pomanjkljivosti: a) Za vedno večja števila si je treba izmišljevati vedno nova imena. b) Možnosti za predstavljanje števil, ki jih je sicer neskončno, so omejene. c) Težko primerjamo velikost števila glede na količino uporabljenih znakov; kot primer naj navedem števili 888 (DCCCLXXXVIII), kjer za zapis potrebujemo 12 znakov, in število 1001 (MI), kjer za zapis potrebujemo dva znaka. Za manjše število v omenjenem aditivnem številskem sistemu lahko tako porabimo več znakov kot za večje. č) Zmnožiti števili 57 (LVII) in 38 (XXXVIII) je praktično nemogoče, še bolj pa razumeti, kako smo prišli do zmnožka 2166 (MMCLXVI). Ravno zaradi omenjenih pomanjkljivosti je bila potreba po iznajdbi drugačnega številskega sistema - sistema mestne vrednosti 032 Nič: pomeni nič - nečesa tako velika. Sistem mestne vrednosti pa se je v popolnosti lahko razvil šele, ko je ničla postala število, saj so tako imeli s pomočjo 10 simbolov neomejeno število možnosti za predstavljanje števil. Težava z iskanjem vedno novega znaka in imena zanj je bila rešena, saj vsaka števka (vključno z ničlo) lahko zasede vsako mesto. Ista števka v zapisu, odvisno od mesta na katerem stoji, predstavlja različno število, ima torej različno ime. V številu 56845 je ime prve števke 5 petdeset tisoč, zadnje pa 5 enic. Mesto v zapisu števila ima določeno ime in šteje samo po sebi. Čeprav je število ena manjše od števila devet, je enica v številu 1001 vredna veliko več kot katera koli deve-tica v številu 999. Prav tako so z iznajdbo števila nič in posledično sistema mestne vrednosti olajšali vsakdanje primerjanje števil med sabo. Količina uporabljenih števk za zapis nekega števila je v mestnem sistemu v premem sorazmerju z velikostjo števila (število 999 je manjše od števila 1001, ker prvega sestavljajo tri števke, drugega pa štiri) prav tako pa so operacije med števili možne in jih lahko razložimo. 5 Število nič v različnih oblikah in kontekstih Kar nekaj raziskovalcev; Hughes (1986), Clemson in Clemson (1994), Worhington in Carruthers (2008), matematiko pojmuje kot (tuj) jezik, ki se ga morajo otroci naučiti. Značilnost jezika je v tem, da lahko iste besede pomenijo različno, odvisno od konteksta, v katerem so uporabljene. Število nič lahko v vsakdanjem življenju izrazimo v dveh oblikah - govorjeni ali za- pisani obliki. Znotraj posamezne oblike lahko opazujemo enako besedo ali simbol, ki v različnih govornih kontekstih pomeni nekaj drugega. Kontekst je lahko matematične ali družbene narave, število nič je lahko kardi-nalno ali ordinalno. Lahko ima numerične vrednosti (število fižolčkov v posodi), lahko pa le opisne (številka registrske tablice). V nadaljevanju bo predstavljenih nekaj različnih besednih oblik, nato pa še nekaj različnih simbolnih oblik. Beseda nič ima v slovenskem jeziku še nekaj sopomenk: ničla, nula, ničeln, ničen, nulti, ničti ... Beseda nič v stavku: »tri plus nič je tri« ima matematični kontekst in pomeni nekaj povsem drugega kot beseda »nič« v reku: »O tem se povsod šušlja, čisto brez nič ne bo.« (družbeni kontekst) Prav tako se razlikuje sporočilo stavka: »Pozabil si napisati ničlo«, ki je lahko tako v matematičnem kontekstu (ničla kot simbol) kot v družbenem (ničla kot del telefonske številke) od tega: »V primerjavi z njim je prava ničla.« (družbeni kontekst) Različno lahko uporabljamo tudi besedo nula. Stavek »Njihova poštna številka je ena, tri, šest, nula« (družbeni kontekst) ima precej drugačen pomen od »Sedem minus sedem je nula« (matematični kontekst) ali »Ena nula zate« (kardinalna numerična vrednost - množica zadetkov na gol). Čeprav je uporaba besednega izražanja ideje o ničli na videz zapletena, pa je simbolna oz. vizualna lahko še bolj. Simbol ničle (0) lahko opazujemo skoraj na vsakem koraku, saj je drugi znak za javno stranišče 00, ob polnoči naša digitalna ura kaže 00:00 ali 24:00, nogometna tekma Slovenija : Anglija se lahko konča 1 : 0, voda zmrzne pri 0 °C, telefonska številka vsebuje področno kodo 031 ali 02, štoparica pred začetkom merjenja časa kaže 0: 00, pri predmetu zgodovina se pogovarjamo o dilemi, ali leto 0 obstaja ali ne, pri kemiji imajo kemijski elementi oksi-dacijsko število 0, zdravnik nam lahko pove, da smo zaradi odsotnosti določenih antige-nov univerzalni krvodajalci s krvno skupino 0, pri slovenskem jeziku lahko srečamo zapis črke O, ki je zelo podobna številu 0, število 0 je lahko sestavni del naše registrske številke na avtomobilu, lahko je oznaka za pritličje v veleblagovnici, termometer lahko kaže 0 °C ... Glede na zapisano, mora otrok prepoznati, v katerem kontekstu govorimo, kar pa po mnenju Gardnerja (1997, v Worhington in Carruthers, 2008) lahko spodbudimo le z zgodnjim seznanjanjem otroka z različnimi zvrstmi znotraj simbolnega sistema. e Razumevanje števila nič Avtorici Wheeler in Feghali (1983, str. 147) v svojem članku navajata, da lahko »razumevanje števila nič pri otrokih vsebuje napačne predstave, ki pa imajo na razumevanje velik vpliv«. Kot eno od takih predstav navajata dva primera: »Nič ni število« in »Nič je nič«. Obstajajo različne teorije o tem, kdaj je posameznik sposoben razumeti idejo števila nič. Piaget pravi, da koncept števila nič ni v celoti razvit, dokler ne dosežemo formalne operativne stopnje, nekje do 11. leta starosti (Wheeler in Fegali, 1983, po Inhelder in Piaget, 1969). To je stopnja, ko miselne operacije niso več omejene s konkretnimi primeri, torej takrat, ko posameznik lahko abstraktno in hipotetično razmišlja v kontekstu jezikovnega in logičnega sistema (Marjanovič Umek, 2004). S trditvijo Piageta, ki pravi, da je za učenje posamezne ravni znanja najprej potreben telesni razvoj, se ne strinja veliko razvojnih psihologov, ki mu očitajo celo, da s svojimi sklepi podcenjuje sposobnosti majhnih otrok (Hughes, 1986, po Gelman in Galli-stel, 1978; Donaldson, 1978). Novejša spoznanja o kognitivnem razvoju otrok so bolj naklonjena ruskemu razvojnemu psihologu Levu Semjonoviču Vigotskemu, čigar ideja je bila, da »učenje predhodi in usmerja razvoj, ključni mehanizem, ki pojasnjuje odnos med učenjem in razvojem pa je območje bližnjega razvoja. Otrokovo mišljenje usmerja odrasli oz. kompetentnejši sovrstnik, s tem pa aktivira kognitivne strukture, ki so že razvite, a ne do mere, da bi jih otrok lahko uporabil samostojno« (Svetina, 2005, str. 104). Omeniti velja še enega psihologa, to je Bruner (1966, v Clemson in Clemson, 1994), ki je s svojim delom veliko prispeval h kognitivni in pedagoški psihologiji. Določil je tri vrste reprezentacij; enaktivno, ikonično in simbolno. Enaktivno pomeni učenje skozi akcijo, medtem ko je ikonična reprezentacija odvisna od uporabe slik, ki pomagajo odkrivati vzorce ali poti. Simbolična reprezenta-cija pomeni uporabo simbolov, skozi katere lahko raziskujemo in manipuliramo hipoteze. Brunerjev model ima jasno povezavo z Piagetovimi stopnjami razvoja, a se razlikuje v bistveni stvari - njegov model ni tako jasno povezan z leti posameznika. Pravi, da opisane modele učenja kot odziv na novo učenje uporabljamo vsi, nekateri na višji stopnji kot drugi, vendar pa nakazuje na to, da se mlajši otroci bolj nagibajo k enaktivnemu mišljenju. Razumevanje števila nič je v preteklosti raziskovalo več avtorjev. Prvi med njimi je Hughes (1986), ki je del poskusa predstavitve 034 Nič: pomeni nič - nečesa pomešal med sabo in ponovno vprašal po konzervi, ki ima določeno vsebino. Tako je preveril, ali so jim oznake kakor koli pomagale pri igranju igre. Rezultati so pokazali, da je dve tretjini predšolskih otrok in vsak otrok v prvem razredu sposobnih pokazati pravo konzervo, glede na svojo oznako. Oznake so bile podobno, kot v prvem poskusu: nerazumljive, piktografične, ikonične in simbolne. Vseeno, pa sta bili prisotni dve pomembni razliki: - Prva od njih zadeva število nič, saj so v nasprotju s prepadenimi pogledi na prazno mizo otroci zdaj odsotnost predmetov v konzervi predstavili enako kot druge količine. Predšolski otroci so največkrat ničlo ponazorili kot prazen prostor, prvo-šolčki pa s simbolom 0. - Druga razlika se je nanašala na število piktografičnih odgovorov, ki se je v primerjavi s prvo zelo zmanjšala. Seveda pa je bila mogoča rešitev tudi ta, da število kock nadomestijo z ustreznim številom predmetov, narisanih na pokrovu, in otroci so uporabljali tudi ta način. Čeprav je bila večina odgovorov popolna jasna vsakemu človeku, je bilo vseeno nekaj takih, kjer na prvi pogled število ni bilo jasno, zato je raziskovalec otroka povprašal, kako ve, da je notri točno določeno število. Opisuje primer razgovora z dečkom, ki pravi: »Ta črtica ima rep, to pomeni, da konzerva nima notri ničesar.« tisti 035 pa, ki jim oznake že takoj niso pomenile ničesar, jih niso znali dešifrirati niti po tednu dni. Koncept števila nič se je v kulturi pojavil dokaj pozno, predvsem zaradi potrebe po ekonomičnosti zapisa in zaradi lažjega računanja. Tudi v naši kulturi velja koncept števila nič kot težko razumljiv. Nekako kontra-diktorno se zdi, da »simbol, ki sicer označuje odsotnost neke količine, takrat, ko je zapisan poleg nekega drugega števila, njegovo vrednost poveča 10krat« (Hughes, 1986, str. 90). Gre za to, da simbol ničle uporabljamo v dva namena. Prvi je ta, da označujemo odsotnost neke količine, drugi pa zato, da povemo, da kakšna potenca števila deset ni bila uporabljena (število 507 S 5S 0D 7E). V tem primeru ima število nič v istem številu lahko različno vlogo. Pri številu 5040 ima prva ničla od leve proti desni funkcijo stotic (0 S), zadnja pa enic (0E). Dejstvo, da ima neka števka (v tem primeru število nič) v istem številu lahko različno funkcijo je lastnost sistema mestne vrednosti. Pri njem lahko z majhnim številom števk napišemo neomejeno število številk. Sistem, ki je izjemno domiseln in enostaven za uporabo, pa je sestavljen kompleksno, iz različnih idej in principov (Clemson in Clemson, 1994). Težave, ki jih prinaša število nič, po mnenju Hughesa (1986) ne izhajajo iz tega, da je ničla simbol, ki ponazarja nič (količinsko), kar se kaže tudi v zgoraj opisani raziskavi. Težave v razumevanju povzroča ničla v sistemu mestne vrednosti. Da težava ni v ničli kot simbolu, ki ponazarja količinsko odsotnost, je razvidno tudi iz primera 6-letnega dečka Erika, ki sem ga približno 3 mesece pred vstopom v prvi razred osnovne šole vprašala: »Kaj je nič?« Odgovoril mi je: »Nič je takrat, ko nimaš nobenega jabolka«. Prav tako ni imel težav z reševanjem računov, kot je 3 - 3 = 0, če si je pri tem pomagal s konkretnimi primeri za ponazoritev števila 3, na primer večjimi lego kockami ali prsti. Računanje je potekalo tako, da je najprej na mizo postavil 3 lego kocke/dvignil tri prste, nato pa 3 kocke z mize vzel/pokrčil tri prste. Podobno se kaže v primeru, ki ga navaja Hughes (1986). Piše o primeru deklice, ki pri petju pesmi o šarkljh v pekarni (katerih število se vztrajno manjša, zato ker jih kupuje deček) s pomočjo krčenja prstov pride do ugotovitve, da v trgovini ne ostane nič več rozinovih peciv. Avtorica Catterall (2005) se je ukvarjala z predstavami otrok o številu nič, bolj konkretno o odnosu med številom nič in drugimi števili. Otrokom, starim od 5 do 11 let je dala naloge, pri katerih so po velikosti razvrščali kartončke z različnimi kombinacijami števil. Ena kombinacija je vsebovala naravna števila (1, 2, 3), število nič in ulomke, druga le decimalna števila, manjša od ena s številom nič, tretja ulomke in število nič, četrta decimalna števila in število nič, peta nekaj naravnih števil in število nič, šesta pa števila od 0 do 9. Rezultati so pokazali, da otroci ničli kot simbolu na številski osi pogosto pripisujejo mesto zraven številke ena (%, 0, 1), pogosto se pojavi tudi otrokovo pojmovanje, da je nič preprosto nič, brez vrednosti, brez pomena in vpliva in je tako popolnoma vseeno, katero mesto zasede, še več, nekateri jo celo ignorirajo. Poleg tega so otroci pogosto v dilemi, ali je ničla celo število ali ne. V zvezi s tem, ali je nič celo število ali ne, je raziskovalka z otroki opravila kratek razgovor. Ugotovila je, da cela števila pojmu- 036 Nič: pomeni nič - nečesa jejo kot polna števila, ne delčke, kot jim to predstavljajo ulomki. Cela števila naj bi zapisovali pred ulomki (4%), a v tem primeru število nič izpade kot celo število, saj pravijo, da enostavno ne pišemo 0%. Ko raziskovalka vpraša, zakaj je tako, ji odgovorijo: Ker je to neumno, ker lahko režemo cela števila na dele, ne moremo pa tega storiti z ničlo. Ne moreš rezati od nič, ne moreš imeti polovice od nič. Haylock in Cockburn (2008, str. 37) pravita, da do razumevanja koncepta nekega števila prihaja na podlagi mreže povezav med »konkretnimi situacijami, simbolom, jezikom in slikami«, t. j. pri povezavi enega simbola z zelo različnimi situacijami. Nevarnost, ki obstaja, pa je, da v mreži povezav, ki tvorijo matematični koncept, preveč utrjujemo le eno povezavo, zanemarjamo pa enako ali pomembnejše povezave. Če ena povezava prevladuje v otrokovem mišljenju, je pozneje težko graditi na novih izkušnjah, ki jih včasih težko povežemo s točno določenim delom mreže povezav (Haylock in Cockburn, 2008). Otrok se v svojem življenju najprej sreča z vsakdanjo uporabo števila nič v pomenu odsotnosti neke količine. Odrasli okrog njega se lahko pogovarjajo: »Tekma se je končala z rezultatom tri proti nič.« ali »Nimam nič več piškotov. Kupiti bom morala nove.« Tudi pri začetnem aritmetičnem računanju, npr. seštevanju in odštevanju, ko si otrok pomaga s konkretnimi predmeti ali prsti v primeru reševanja, računa: 2 + 0 = verjetno razmišlja takole: »Imam dve kocki oz. dva dvignjena prsta. Nato ne dodam nobene kocke/ne dvignem nobenega prsta. Ostala sta mi torej dva prsta/dve kocki. To je verjetno rezultat. Kot je razvidno iz primera, otrok tudi v mate- matičnem svetu razmišlja o ničli kot o neki odstotnosti količine. Beseda nič oz. simbol nič v neki meri zagotovo pomeni odsotnost količine. Pa je res le to? Pravilen odgovor bomo dobili že, če bomo pri pisnem računanju deljenja opazovali otroka oz. učenca. Pri reševanju računa 0 : 2, se bo vsaj na začetni stopnji zagotovo znašel pred kognitivnim konfliktom. V vsakdanjem življenju se nam namreč deljenje nič bombonov na dva dela zdi brez smisla, v matematičnem problemu pa ga ima. Če na primer nič bombonov razdelimo dvema otrokoma, to lahko naredimo pravično. Vsak od otrok bo dobil nič bonbonov. Težave otrok s številom nič po mnenju Haylock in Cockburn (2008) nastanejo zato, ker prevladuje eden od konceptov števila nič. Število nič ima namreč tako količinski kot abstraktni pomen, saj označuje ničelno množico, je število, predstavlja točko na številski osi, ki deli pozitivna in negativna števila, je simbol, označuje prazno mesto v sistemu mestne vrednosti, je nevtralen element za seštevanje in odštevanje. Različne koncepte lahko dojamemo na različnih stopnjah kognitivnega razvoja, velikokrat se zgodi, da ravno zaradi nerazumevanja nekega pomena pride do nejasnosti oz. težav pri pouku matematike. Kljub temu pa ima ničla na mišljenje in razumevanje okrog nas velik pomen. Catterall (2005) pravi, da do težav z razumevanjem števila nič pri otrocih pride zato, ker so otroci »neodločni glede tega, kateri koncept ničle naj v dani situaciji poudarijo in katerega ignorirajo. Njihovo razmišljanje se je premaknilo z abstraktnega premisleka o simbolu na konkretni premislek o vrednosti ničle« (Catterall, 2005, str. 69). Pravi, da »ob- in uporabo števila ter prazno skupino, ki jo vidijo, ko abstraktni simbol ničle pojasnjujemo v konkretnem izrazu« (prav tam). Vseh konceptov ničle otrok ne dojame naenkrat, ampak skozi proces učenja. Potrebne so različne izkušnje, preko katerih jih otrok spoznava in postopno dojema. Z Vpliv števila nič na matematiko kot predmet Iznajdba števila nič ima na matematiko izjemen vpliv, saj je bilo šele z iznajdbo ničle mogoče neovirano seštevati, odštevati, množiti in deliti, med seboj lahko primerjamo velikost števil ... Prav iznajdba ničle je pripomogla k temu, da se je matematika kot disciplina tako razvila, hkrati pa postala tudi sestavni del drugih discipin. V nadaljevanju bom predstavila, kje v učnem načrtu pri matematiki je potrebno razumevanje ideje o številu nič in kje ležijo najpogostejše težave v razumevanju. a) Prvi razred V prvem razredu se učenci s številom nič srečajo v obliki štetja, zapisa in branja števil od 0 do 20, to je simbolno obliko. Poleg tega se srečajo z oceno števila predmetov v množici, ki nima več kot 10 elementov, to je ničlo kot kardinalnim številom. Ločijo med glavnim in vrstilnim pomenom števila, določijo predhodnik in naslednik števila, prepoznajo in nadaljujejo oz. oblikujejo preprosto zaporedje števil in primerjajo števila po velikosti. Poleg že omenjenega spoznajo še seštevanje in odštevanje v obsegu od 0 do 10. Težave, ki jih v prvem razredu lahko povzroči število nič, so naslednje: - Učencem se lahko zdi nesmiselno, da označujejo odsotnost predmeta s prisotnostjo nekega števila. - Učenci se lahko pri predhodniku ali nasledniku števila zmotijo pri zapisu, saj ima število 9 velikokrat v vrsti kot svojega naslednika zapisano število 0. - Poleg tega lahko zaradi zmotnega mišljenja o tem, da je število nič popolnoma nič, brez vrednosti, brez pomena, delajo napake pri urejanju števil po velikosti. Caterall (2005) v svojem članku o dojemanju ničle v odnosu do drugih števil piše o razmišljanju otrok, da je zaradi zgoraj opisanih lastnosti ničle vseeno, katero mesto ničla zasede pri urejanju po velikosti, nekateri otroci ničlo celo popolnoma prezrejo. - Pri primerjanju števil lahko naletimo na težavo nerazumevanja tega, da ima število ena, če ima na desni strani zapisano število nič, lahko desetkratno vrednost (razumevanje sistema mestne vrednosti). Tako lahko učenci števili 1 in 10 zapišejo eno poleg drugega, na podlagi predvidevanja, da ničla zraven ne spremeni ničesar. - Pri seštevanju in odštevanju s številom nič učenci lahko razmišljajo na način, kot ga opisujeta avtorici * Anthony in Walshaw (2004), in sicer, da prištevanje vsakega števila nekemu drugemu številu pomeni povečanje, odštevanje nekega števila od drugega pa zmanjšanje tega števila. V tem primeru števila nič otroci sploh ne uvrščajo med števila, prihaja pa do napačnih izračunov. 038 Nič: pomeni nič - nečesa b) Drugi razred V drugem razredu se učenci s številom nič srečajo v obliki štetja, zapisa in branja števil do sto, z razlikovanjem desetiških enot (enic, desetic in stotic), urejanjem množice naravnih števil do 100 po velikosti, določanjem predhodnika in naslednika danega števila, oblikovanjem in zapisovanjem preprostega zaporedja števil in zapisovanjem odnosov med števili. Poleg tega se srečajo še z razumevanjem vloge števila 0 in 1 pri seštevanju in odštevanju in z razumevanjem, da je število 0 razlika dveh enakih števil. Poleg težav, ki sem jih izpostavila že v prvem razredu, je pogosto kot problematična za otroke omenjena mestna vrednost. Težave lahko nastopijo pri branju in pisanju števil v dogovorjenih oblikah. Branje in pisanje števil pa težav ne povzroča le majhnim otrokom, ampak, kot ugotavlja Brown (1981, v Wheeler in Feghali, 1983) tudi odraslim. Nekaj manj kot polovica študentov v njegovi študiji namreč ni znala pravilno zapisati števila štiristo tisoč tri in sedemdeset. Nekaj podobnega sta ugotovila tudi Crooks in Flockton (2002, v Wheeler in Feghali, 1983) ki sta raziskovala zapis v standardnih oblikah pri novozelandskih študentih. Ugotovila sta, da le približno 11 % učencev 4. razreda in 50 % učencev 8. razreda zna pravilno zapisati število dvesto tisoč triinštirideset v simbolni obliki. c) Tretji razred V tretjem razredu se učenci s številom nič srečajo v obliki usvojitve produktov v obsegu 10 ■ 10 (poštevanka) do avtomatizma. To predznanje je nujno za množenje z nič. Poleg tega se učijo uporabljati računske zakone pri seštevanju in množenju in spoznajo vlogo števil 0 in 1 pri množenju in deljenju. Kot pravita Haylock in Cockburn (2008, str. 113, 114), so »težave pri množenju in deljenju z ničlo posledica lastnosti ničle v odnosu do množenja in deljenja. Pojasniti 7 ■ 0 = 0 in 0 ■ 7 = 0 ni preveč težko. Mislimo na sedem množic z nič elementi oz. nič množic s sedmimi elementi. Preprosto dovolj je, da uporabimo ponavljajoče seštevanje kot sestavni del množenja. Vendar pa morajo biti otroci seznanjeni s temi mislimi in o njih razpravljati, saj lahko sicer identiteto za seštevanje narobe uporabijo na primerih množenja. Na problem 7 ■ 0 lahko odgovorijo z odgovorom 7. Napačen odgovor izvira iz nepravilne domneve, da množenje z nič pomeni »ne naredi ničesar«. Pri množenju ima to lastnost »ne naredi ničesar« število 1. Deljenje, ki vsebuje ničlo, pravita Haylock in Cockburn (2008), je nekoliko bolj zapleteno. Koliko je 0 : 5 in koliko 5 : 0? Račun 0 : 5 lahko pretvorimo v situacijo: Imamo 0 bombonov, ki jih želimo pravično razdeliti petim ljudem. Vsak od njih bo dobil enako število - nič bombonov (0 : 5 = 0). Račun 5 : 0 pa nima smisla pri deljenju na enake dele - ne moremo predvideti deljenja 5 predmetov med nič ljudi. Lahko pa rečemo: »Koliko množic z nič elementi je potrebnih, da dobimo 5 elementov?« Računanje lahko poteka dolgo časa, pa še vedno ne bomo niti blizu 5 elementom. Pravimo, da tega ne moremo izračunati. Obstaja pa še poseben primer deljenja, in sicer: 0 : 0. V tem primeru imamo nešteto rešitev, saj je rezultat 0 ali katero koli naravno število b (0 : 0 = 0, ker je 0 ■ 0 = 0, pa tudi 0 : 0 = b, saj je 0 ■ b = 0). Pravimo, da izraz 0 : 0 ni definiran. 39 č) Četrti razred V četrtem razredu se učenci s številom nič srečajo v obliki pisnega odštevanja, ustnega množenja in deljenja z 10 in 100, pisnega in ustnega množenja z enomestnim številom v množici naravnih števil do 1000, pisnega množenja z dvomestnim številom v množici naravnih števil do 1000, pisnega deljenja z enomestnim številom s preizkusom, pisnega deljenja z večkratniki števila 10, uporabljanja računskih operacij pri reševanju problemov ... Pri odštevanju lahko opazujemo težave otrok s pisnim odštevanjem, ko je račun zapisan v stolpcu, na primer 5000 - 189. »V pokončnem formatu, ničla v enicah in de-seticah odštevanca, to je spodnjega števila, pomeni, da ni treba ničesar odvzeti, zaradi česar se nekateri učenci sprašujejo, kaj storiti. Ko je ničla sestavni del zmanjševanca ali zgornjega števila, pomeni, da tam ni ničesar, od koder bi lahko jemali.« (Anthony in Walshaw, 2004, po Klein, 2000). Opisani drugi situaciji sem bila priča tudi sama, in sicer pri pouku dopolnilnega pouka. Deklici težav ni toliko povzročala ničla v enicah kot pa to, da si »ne more sposojati od desetic«, ker »tam ni ničesar«. Ničla svoje posebnosti kaže v računih pisnega množenja oblike: 24 ■ 12. Ker gre pri pisnem množenju za algoritem, s pomočjo katerega med seboj najprej množimo števili 24 in deset, nato 24 in 2, na koncu pa seštejemo vmesne rezultate, je zapis tak: 24 -12 24Ö" +_48 288 Pozneje pa zapis števila nič v prvem, delnem izračunu izpuščamo, vendar mu mesto še vedno ohranjamo. 24 ■ 12 24 +_48_ 288 Prihaja do napak, pri katerih učenci napačno podpisujejo enice, desetice in stotice, in do nerazumevanja tega, zakaj do zamika sploh prihaja. V četrtem razredu se, če tega niso usvojili v tretjem razredu, pri učencih pojavijo težave pri računanju podobnih primerov, kot je: 4024 : 4. V tem primeru je prva števka deljenca deljiva s 4 in potrebno je deljenje z naslednjo, z ničlo. V tem primeru veliko učencev zmotno predvideva, da je 0 : 4 = 0 (preprosto nič, nič vredno) in zato delnega količnika ne vpiše med končnega. V rezultatu se zmotno mišljenje pokaže kot eno mesto prekratko število. V omenjenem primeru se skriva še ena »past«. V nadaljevanju deljenja, kjer je treba dve deliti s štiri, lahko učenci zmotno predvidevajo, da rešitev v celih številih ni mogoča (ničle ne uvrščajo med cela števila), in zato ponovno izpustijo zapis ničle. Predvidevamo lahko, da je izpust zapisa števke nič posledica uporabe algoritma, ko je prva števka v deljencu, manjša od delitelja. V primeru 234 : 3 = dve stotici namreč lahko razdelimo na tri enake dele le tako, da vsak del dobi vrednost nič. Ker pri tem že upoštevamo to, da število nič, zapisano na levi strani, pred naravnim številom nima vpliva na velikost števila, delni količnik nič preprosto izpuščamo oziroma ga ne zapisujemo, kar pa lahko učencem povzroča veliko težavo v razumevanju, ki je lahko tudi posledica neusvojenega koncepta mestne vrednosti. Učencem je treba pojasniti, zakaj v prvem primeru ničle ne pišemo in zakaj jo je treba pisati pozneje. Vidimo lahko, da tu razumevanje koncepta ničle zopet igra veliko vlogo, 040 Nič: pomeni nič - nečesa saj v nasprotju z ničlami na levi strani, pred naravnim številom, števila, napisana na desni strani naravnega števila, vpliv na velikost števila imajo. d) Peti razred V petem razredu se učenci z ničlo srečujejo v kontekstu naslednjih ciljev: pisati in brati števila do milijona, razlikovati desetiške enote, urediti naravna števila do milijona, opredeliti predhodnika in naslednika števila, poznati in razlikovati liha ter soda števila, zapisati in brati števila prek milijona, števila zaokrožiti na desetice, stotice, ponazoriti grafično na številski premici vse štiri računske operacije, seštevati in odštevati v obsegu do milijona (ustno in pisno), množiti naravna števila v obsegu do milijona, deliti z dvomestnim naravnim številom, zapisati s potenco produkt enakih faktorjev in obratno, izračunati vrednost potence naravnih števil, razčleniti naravna števila na večkratnike potenc števila 10 (desetiški sestav), rešiti s premislekom in z diagramom preproste enačbe (računske enakosti) in znati napraviti preizkus. Poleg drugih lastnosti, ki jih ima število nič in sem jih že omenila v prejšnjih razredih, je tudi ta, da je ničla sodo število. Za učence je to lahko težava, saj je težko reči, da je sodo nekaj, česar po njihovih velikokrat zmotnih interpretacijah ni. Za razumevanje ničle kot sodega števila je treba ponovno ra- Število nič ima posebnosti tudi v potencah. Katero koli število, različno od nič, namreč ima potenco nič, je enako natančno 1. Prvič, otroci zaradi podobnosti zapisa potenco 50 pretvorijo v račun 5 ■ 0, katerega rezultat je enak 0. Drugič, pri veliko otrocih opažam, da se to pravilo enostavno naučijo na pamet, ko pa jih vprašaš po razlagi, tega ne znajo pojasniti. Uvajanje tega pravila bi po mojem mnenju moralo potekati na primeru, deljenju dveh potenc z enakima osnovama: a4 a ■ a ■ a ■ a 1 ■ 1 ■ 1 ■ 1 a4 a ■ a ■ a ■ a 1 ■ 1 ■ 1 ■ 1 Število nič ima pomembno funkcijo pri množenju desetic in deljenju desetic. Pri množenju števil 10 in 100 enostavno le preštejemo število ničel in jih zapišemo za številom ena (10 ■ 100 = 1000). Pri deljenju pa števili ničel odštejemo. Število nič ima pomembno funkcijo tudi pri reševanju preprostih enačb. Če namreč vse vrednosti prenesemo na eno stran enačbe, je druga stran enaka nič. V tem primeru se velikokrat zgodi, da otroci pozabijo na to in preprosto ne vedo, kaj na to mesto zapisati, čeprav gre za preprost račun odštevanja dveh enakih števil (zmanjševanca in odšte-vanca), katerega rezultat je nič. e) Šesti razred dve 041 naravni števili (količnik je lahko decimalno število) in narediti preizkus. Število nič svojo izrednost in posebnost v šestem razredu kaže pri decimalnih številih. Prvič, nekaterim otrokom je izredno težko po velikosti urediti števila: 0,003, 0,03, 0,3 in 0,30, saj ne moremo primerjati velikosti števila na podlagi dolžine zapisa, poleg tega je pomembno, kje se nahaja števka, večja od 0, saj po njej lahko ocenimo, za kako velike dele celote gre (desetine, stotine ...). Pri tem je pomembno, da se otroci zavedajo, da za zadnjo decimalko večjo od nič lahko stoji neomejeno število ničel, pa vendar to ne vpliva na povečanje števila za določeno potenco števila 10 kot pri celih številih. Nasprotno pa ničla pred decimalko, večjo od nič, število 10-krat pomanjša. Najpogostejše napake, ki se pojavljajo pri tem cilju, so, da učenci svoje znanje o tem, da ima daljše število, kot je, večjo vrednost, prenesejo tudi na decimalna števila. O težavi primerjanja decimalnih števil med seboj, razpravljata tudi Anthony in Walshaw (2004). Pri množenju decimalnih števil ima ničla posebnost v tem, da je število decimalnih mest v rezultatu enako seštevku decimalnih mest obeh faktorjev. Nevarnost, ki obstaja, je, da otrok prezre ničle in kot rezultat upošteva le naravna števila. f) Sedmi razred V sedmem razredu se otroci z ničlo srečujejo v kontekstu naslednjih ciljev: ulomek zapisati kot k in ulomek zapisati z decimalnim številko in jo zaokrožiti na zahtevano število decimalnih mest. k-a ,(a*0) Tu se ponovi in nekoliko poglobi znanje o deljenju števil, predvsem pa vedenje o tem, da deljenje z ničlo poteka v neskončnost in zato rezultata ne moremo definirati. g) Osmi razred V osmem razredu se otroci z ničlo srečujejo v kontekstu naslednjih ciljev: racionalnemu številu poiskati nasprotno vrednost, ugotoviti, kateri množici števil pripada dano število, racionalnemu številu določiti absolutno vrednost, poznati in uporabljati znak za absolutno vrednost. Težave, povezane z ničlo, v tem razredu so lahko te, da ne vedo, kaj je to absolutna vrednost - oddaljenost nekega števila od številskega izhodišča (0) na številski premici. Ta vrednost je vedno pozitivna, medtem ko ničla ni ne pozitivna ne negativna. h) Deveti razred V devetem razredu se otroci z ničlo srečujejo v kontekstu naslednjih ciljev: izračunati produkt vsote in razlike dveh danih členov ter kvadrat dvočlenika, izračunati kvadrat dvočlenika in razliko kvadratov, določiti ničlo na grafu linearne funkcije, izračunati ničlo funkcije. Tu se najpogosteje pojavljajo napake pri tem, da učenci zamešajo, katera vrednost mora biti enaka nič (x ali y), da ima graf funkcije ničlo. Prav tako se pojavljajo težave pri reševanju enačb, katerih rešitev je enaka nič. n Sklep Veliko avtorjev: Wheeler in Feghali (1983), Hughes (1986), Clemson in Clemson (1994), Anthony in Walshaw (2004) ter Cat- a 042 Nič: pomeni nič - nečesa teral (2005) pridejo do skupnih ugotovitev, in sicer, da je predstava o ničli kot prazni, ničvredni in nepomembni zelo odmevna, predvsem kar zadeva razumevanje koncepta števila nič. Da bi otrokom pomagali in preventivno delovali pred težavami v razumevanju, je po mnenju Clemson in Clemson (1994) treba otroke seznaniti z različnimi izkušnjami, treba jim je dati možnost, da uporabljajo konkretne predmete pri delu s števili. Da bi razumeli globlji pomen števila nič, jim moramo pomagati razumeti princip sistema mestne vrednosti, za katerega so v zgodovini porabili veliko razmisleka in idej, da so ga razvili, mi pa pričakujemo, da ga bodo otroci usvojili takoj. To lahko naredimo tako, da jim omogočimo uporabo simbolov na zanje nov način. S tem se strinjata Anthony in Walshaw (2004), ki pravita, da morajo pravila otroci doumeti na podlagi lastnih dejavnosti. Šele z nakopičenim bogastvom izkušenj in spodbudami k induktivnemu mišljenju, ki jim pomaga pri povezavi, bodo otroci zgradili svoje matematične koncepte in nekega dne, navdušeno, kot otrok, o katerem pišeta *Anthony in Walshaw (2005, str. 38, po Wheeler in Feghali, 1983), avtor izjave v naslovu članka, sprevideli, da nič lahko označuje več stvari, ne le odsotnost količine, kar pa je zagotovo prva otrokova izkušnja z omenjenim številom. S Viri in literatura 1. Anthony, J. G. in Walshaw, A. M. (2004). Zero, a none number. Teaching children mathematics, 11 (1), (3842). 2. Berlinghoff, P. W. in Gouvea, Q. F. (2008). Matematika skozi stoletja. Ljubljana: Modrijan. 3. Catterall, R. (2005). Exploring children's conceptions of zero in relationship to other numbers. V J. Fragner in B. Hudson, B. (ed.), Researching the teaching and learning of matehematics II (str. 55-75). Linz: Pädagogische Akademie des Bundes in Oberösterreich. 4. Clemson, D. in Clemson W. (1994). Mathematics in the early years: Teaching and learning in the first three years of school. London: Routhledge. 5. Guedj, D. (1996). Svet števil. Ljubljana: DZS. 6. Haylock, D. in Cockburn, A. (2008). Understanding mathematics for young children: a guide to foundation stage and lower primary teachers. Los Angeles (idr.): SAGE. 7. Hughes, M. (1986). Children and number. Difficulties in learning mathematics. Oxford: Basil Blackwell. Razvojna psihologija. Ljubljana: Znanstveno-razisko-valni inštitut Filozofske fakultete. 9. Svetina, M. (2005). Izkustveno mišljenje kot prehod med predoperacionalnim in konkretnologičnim mišljenjem pri otrocih. Pedagoška obzorja 14 (1), (101— 118). 10. Wheeler, M. in Feghali, I. (1983). Much ado about nothing: Preservice Elementary School Teacher's Concept of Zero. Journal for Reasearch in Mathematics Education 14 (3), (147-155). 11. Worthington, M. in Carruthers, E. (2008). Children's mathematics: making marks, making meaning. London: P. Chapman; Thousands Oaks, New Delhi: SAGE. 044 Nič: pomeni nič - nečesa Stili zaznavanja in diferenciacija Types of perception and differentiation I Povzetek V prispevku je predstavljenih nekaj teoretičnih izhodišč o diferenciaciji in stilih zaznavanja, zato da bi osvežili strokovno znanje o tem. Sledi primer učne ure pri pouku matematike, v kateri je didaktično-metodična diferenciacija zasnovana na upoštevanju zaznavnih stilov pri učencih. Najprej smo s pomočjo vprašalnika določili zaznavne stile, nato smo učne metode, didaktična sredstva in pristope k poučevanju in učenju ustrezno prilagodili. Če otroku omogočimo, da spozna, kateremu zaznavnemu kanalu daje prednost pri sprejemanju in notranji predstavitvi čutnih vtisov iz okolja, bo tudi zaradi tega lahko pri učenju uspešnejši, njegovo znanje pa večje in trajnejše. Ključne besede: stili zaznavanja, diferenciacija, matematika, osnovna šola Tadeja Možina OŠ Dragotina Ketteja Ilirska Bistrica I Abstract The article presents some theoretical starting points for the differentiation and types of perception in order to enable refreshment of professional knowledge. It also describes an example of mathematical lesson in which didactic-methodical differentiation is based on pupils' styles of perception. With the help of a questionnaire, we first defined the types of perception, matching them to suitable learning methods, didactical materials, and te- Matematika v šoli ~ XVIII. [2012] ~ 045-056 aching and learning approaches. If we enable a child to find out which perception channel he/she favors at reception and inner representation of sensual sensations from surroundings, then he/she will be able to learn more successfully and his/her knowledge will be more substantial and permanent. Keywords: types of perception, differentiation, mathematics, primary school a Uvod Diferenciacija pouka je eden od pomembnejših večdimenzionalnih in večplastnih pojavov, ki ima v prenovljenem programu osnovne šole še poseben pomen in je tudi sistemsko urejen v Zakonu o osnovni šoli (1996; 2006) in v Pravilniku o izvajanju diferenciacije pri pouku v osnovni šoli (2006). Diferencirati pomeni, da učni tempo, raven zahtevnosti, načine poučevanja in druge elemente učnega procesa spreminjamo tako, da jih prilagajamo individualnim potrebam učencev, njihovim učnim slogom in interesom. Diferenciacija je zahteven proces, ki zahteva skrbno pripravo in načrtovanje ter temelji na predhodnih didaktičnih analizah različnih dejavnikov vzgojno-izobraževalne-ga procesa. Namen diferenciacije je optimizirati in racionalizirati izobraževalni proces za posameznika ter ga prilagoditi stopnji njegovega kognitivnega razvoja. Učenci se nenehno spreminjajo, spreminjajo pa se tudi njihovi interesi, motivacija, učni (ne)uspeh in vzgoj-no-izobraževalne okoliščine. Z upoštevanjem drugačnosti vsakega posameznika se izboljšujejo tudi njegove vzgojno-izobraže-valne in razvojne možnosti. Vse to pa pogojuje diferenciacijo (kot pretežno organizacijski ukrep, s katerim demokratično usmerjamo učence po njihovih določenih razlikah v občasne ali stalne homogene ali heterogene učne skupine, da bi tako šola z bolj prilagojenimi učnimi cilji, vsebinami in didaktično-meto-dičnim stilom dela bolje uresničevala socialne in individualne vzgojno-izobraževalne namene; Strmčnik, 1987), ki se vedno znova prilagaja in postopoma vodi v individualizacijo (kot didaktično načelo, ki zahteva od šole in učitelja, da odkrivata, spoštujeta in razvijata utemeljene individualne razlike med učenci, da skušata sicer skupno poučevanje in učenje čim bolj individualizirati in personificirati, se pravi prilagoditi individualnim vzgojnim in učnim posebnostim, potrebam, željam in nagnjenjem posameznega učenca ter mu omogočiti kar se da samostojno učno delo; Strmčnik, 1987). Diferenciramo oziroma prilagajamo lahko vse sestavine in organizacijo vzgojno-iz-obraževalnega procesa, to je: cilje, vsebino, rabo didaktičnih sredstev, čas, organizacijo, didaktične oblike, metodično ravnanje in odnose med subjekti izobraževalnega procesa. Na odločitev o izvedbi diferenciacije vpliva učitelj na podlagi svojega strokovnega znanja, izkušenj, poznavanja modelov, organizacijskih oblik in metod, predvsem pa učenci. Upoštevamo njihovo predznanje (formalno in neformalno), kognitivne zmožnosti glede na starost, spoznavni stil, učni slog, učni tempo, vpliv spola ... Učenci so namreč s 046 Stili zaznavanja in diferenciacija svojimi relativno konstantnimi razlikami najpomembnejša sestavina učnega procesa. Ne moremo mimo dejstva, da morajo učenci s svojimi razlikami odločilno vplivati na celoten učni potek in da ni mogoče več vztrajati pri zmoti, po kateri se morajo le učenci prilagajati pouku. (Strmčnik, 1987) Kaj in kako bomo diferencirali, je treba skrbno načrtovati in pripraviti. Ta proces je v vseh pogledih zahteven, zato naj temelji na predhodni didaktični analizi vseh sestavin vzgojno-izobraževalnega procesa. Stili zaznavanja Učenci in ljudje nasploh se med seboj razlikujemo v marsičem. Obstajajo psihološke (sposobnosti, interesi, motivacija, osebnostne lastnosti . ), socialne (družinsko in družbeno okolje), fiziološke (stanje čutil, živčevja, zdravstveno stanje, senzomotori-ka), fizikalne in druge razlike. Vsak človek je individuum in tudi svet okrog sebe vsak dojema na svoj način. Informacije iz okolja sprejemamo preko vseh čutov, enemu ali dvema pa dajemo prednost. Nekateri posamezniki se učijo z glasnim branjem, se o učni snovi pogovarjajo ali jo drugim razlagajo, drugi raje poslušajo, tretji rišejo skice, miselne vzorce, si delajo zapiske. So tudi ljudje, ki se najbolje učijo tako, da nekaj delajo, izvajajo, saj za ohranjanje pozornosti potrebujejo dovolj gibanja. Tovrstne razlike med ljudmi niso povezane le z ravnijo umskih sposobnosti, temveč gre za kakovostne razlike v tem, čemu daje kdo prednost in v kakšnih okoliščinah bolje deluje. Gre za različne stile zaznavanja, spoznavanja in učenja. »Z izrazom stil spoznavanja označujemo razmeroma dosledne in trajne posebnosti posameznika v tem, kako sprejema, ohranja, predeluje in organizira informacije ter na njihovi osnovi rešuje probleme. Stili so izraz širših razsežnosti osebnostnega funkcioniranja, saj sodelujejo tudi čustveno-motivacijske plati osebnosti. Učni stil je soroden pojem, le da je nekoliko širši in zajema tudi tipične strategije učenja, pa tudi cilje in pojmovanja učenja. Stil zaznavanja pa označuje zaznavni kanal - čutilo (vid, sluh, tip, okus ...), ki mu posameznik daje prednost pri sprejemanju in notranji predstavitvi čutnih vtisov iz okolja.« (Marentič - Požarnik, 2000, str. 152). »Ljudje se razlikujemo po tem, katerim čutnim vtisom (kanalom) dajemo prednost pri zaznavanju, predstavljanju, učenju in tudi sporočanju. Največkrat gre za delitev na vizualni (vidni), avditivni (slušni) in kineste-tično-čustveni stil (čutili vonja in okusa sta v učenje redkeje vključeni).« (Marentič -Požarnik, 2000, str. 153). Nekaj zunanjih (vedenjskih) značilnosti ljudi z različnimi stili zaznavanja (Marentič -Požarnik, 2000, str. 153). Vizualni stil. Posameznik: - uporablja predvsem besede, ki označujejo barve in vidne vtise: ima jasne predstave, uvidi bistvo problema; - je organiziran, sistematičen; - je miren, premišljen; - si zapomni predvsem slikovno gradivo, podobe; - hrup ga manj moti; - težko si zapomni ustna navodila; - raje bere sam, kot da posluša; - stvari urejuje po barvah; - želi pregled, vizijo (na papirju skice, miselne vzorce ...). Avditivni stil. Posameznik: - uporablja izraze »to mi dobro zveni«, »to je odgovor na vprašanje«; - ima rad predavanja, razprave, razgovore in si veliko zapomni; - pri branju premika ustnice, rad glasno bere; - govori sam s seboj, pri učenju uporablja notranji dialog; - hrup ga moti pri delu; - govori ritmično; - ima rad glasbo; - dobro posnema govorni ton, barvo (narečno) melodijo; - vse si zapomni po vrsti, po korakih; - bolje govori, kot piše. Kinestetični stil. Posameznik: - uporablja izraze »imam slab občutek«, »obliva me kurja polt«; - ljudi, stvari, predmetov se dotika, se jim približa; - veliko se giblje, gestikulira; - uči se ob ravnanju s predmeti; - si več zapomni med hojo; - ob branju si kaže s prstom; - bolje si zapomni celovito izkušnjo kot podrobnosti; - govori počasi; - rad bere »akcijske« knjige; - važnejši mu je dober občutek kot videz (obleke ...). [Slika 1] Plakat v učilnici enak stil ali kadar je učitelj dovolj prožen in svoj način dela prilagaja. Pomembni napotki za učitelja (Marentič - Požarnik, 2000, str. 161): - učitelj naj se zaveda značilnosti in posledic svojega prevladujočega stila in stila svojih učencev, - upošteva naj značilnosti večinskega stila učencev in se mu prilagaja, - učence naj ozavešča o teh razlikah, jih navaja na samoopazovanje, razmislek, jim ponudi načine za spoznavanje lastnega stila, - učence naj spodbuja k izkoriščanju lastnih prednosti in k sistematičnemu razvijanju šibkih področij, - daje naj napotke za domače učenje, ki bodo upoštevali posebnosti stilov (npr. uporaba zvočnih posnetkov snovi, miselnih vzorcev, učne kartoteke ...). Pri večini ljudi gre za kombinacije stilov, vendar je eden prevladujoč. Najpogostejši je vizualni stil. Učitelj ne more podrobno poznati vseh stilov in njihovih različic. Pomembno je, da je za tovrstne razlike občutljiv in da pozna svoj stil. Rezultati v učno-vzgojnem procesu so najboljši, kadar imajo učenci in učitelj Primer diferenciacije pri pouku matematike na podlagi stilov zaznavanja Ideja za metodično-didaktično diferenciacijo na podlagi poznavanja stilov zaznavanja pri učencih je bila podana na enem izmed študijskih srečanj učiteljev matematike v osnovni šoli. Razlike med učenci, ki narekujejo potrebe po diferenciaciji in individua- 048 Stili zaznavanja in diferenciacija lizaciji, lahko identificiramo tudi s poznavanjem njihovega stila zaznavanja. Diferenciacija obsega vrsto strategij, s katerimi se učinkovito odzivamo na različne učne potrebe v učni skupini. Učenje učencem približamo tako, da jih vključimo v dejavnost, pri kateri se lahko oprejo na svoja močna področja in nagnjenja. Če otrok ve, kateri zaznavni stil pri njem prevladuje, lahko s pridom izkoristi njegove prednosti za uspešnejše učenje. a) Ugotavljanje stila zaznavanja v učni skupini Točke I z j a v a 1. Nekaj lažje razumem, če se o tem pogovarjam z drugimi ljudmi. 2. Pri pouku precej pozorno opazujem učiteljev obraz. 3. Pri zapisovanju v zvezek ali pri branju učne snovi uporabljam barve (npr. mar- kerje, naglaševalce besedila, flomastre, barvice). 4. Dobre zamisli se mi porajajo, kadar sem telesno dejaven. 5. Raje imam ustna kot pisna navodila. 6. Raje poslušam besedilo na kaseti, kot da bi ga sam prebral. 7. Bolj kot ustni opis poti, ki jo moram prehoditi, mi ustreza narisan zemljevid. 8. Slabše se odrežem pri pisnih testih kot pri ustnem spraševanju. 9. Ko se učim, ne sedim rad za mizo, ampak si raje izbiram različna mesta (npr. na tleh, na postelji ...) 10. Delam si zapiske, vendar so le-ti nekoliko neurejeni. 11. Z lahkoto razumem in berem zemljevide, preglednice, grafe ipd. 12. Ne morem dolgo sedeti pri miru. 13. Rad delam stvari z rokami. 14. Če nekaj delam, me prižgan radio moti. 15. Kadar se učim, imam rad veliko premorov. 16. Kadar govorim, uporabljam tudi telesno govorico (npr. kretnje). 17. Težko si predstavljam neznane stvari, pojave, naprave. 18. Raje bi takoj začel izdelovati neki izdelek, kot da bi prej poslušal navodila o tem, kako ga narediti. 19. Rad pripovedujem šale in si jih zlahka zapomnim. 20. Kadar berem ali poslušam razlago, si veliko zapisujem. 21. Med poslušanjem razlage pogosto delam čačke po papirju oz. klopi. 22. Četudi med ustno razlago ne gledam učitelja, lahko dobro sledim temu, kar govori. 23. Rad ustvarjam modele iz tega, kar se učim. 24. Ko pišem test, si zlahka predstavljam stran v zvezku ali knjigi, kjer je snov, ki sem se jo učil. 25. Raje delam projektne naloge, kot pa pišem spise, povzetke in obnove. 26. Kadar pišem, rad govorim. 27. Ko berem, v mislih 'poslušam' besede. 28. Če si pišem, si bolje zapomnim. 29. Pri ljudeh si ne zapomnim dobro njihove zunanjosti, bolje si zapomnim njihove besede. 30. Če si želim nekaj zapomniti, npr. telefonsko številko nekoga, mi pomaga, če si o njej v mislih ustvarim podobo. 31. Če se učim na glas, si snov bolje zapomnim. 32. V mislih si lahko predstavljam tisto, kar berem, poslušam. 33. Raje berem sam, kot da mi bere kdo drug. Točke: 5 - skoraj vedno 4 - precej pogosto 3 - včasih 2 - redko 1 - skoraj nikoli Vir: CPI. Medmrežje: http://www.cpi.si/files/cpi/userfiles/Publikacije/Mapa_ucnih_dosezkov1.pdf (18. 2. 2010) Vprašalnik za ugotavljanje stila zaznavanja sem uporabila v oddelku 7. razreda (heterogena skupina 16 otrok). Učencem sem najprej njihovi razvojni stopnji primerno predstavila teoretična izhodišča. Značilnosti posameznega stila zaznavanja, spoznavanja in učenja sem v strnjeni obliki predstavila tudi na plakatu. Učenci so pokazali navdušenje za sodelovanje in so z veseljem odgovorili na vprašanja. Vprašalnik za ugotavljanje stila zaznavanja Za ugotavljanje stila zaznavanja je na voljo kar nekaj različnih vprašalnikov, vendar ni vsak primeren za dvanajstletne otroke. Za spodnjega sem se odločila, ker so izjave v njem preproste in učencem razumljive. Tudi sistem točkovanja je preprost. Učenec vsako izjavo ovrednoti s točkami od 1 (če trditev zanj skoraj nikoli ne velja) do 5 (če trditev zanj skoraj vedno velja). Glede vprašalnika sem se posvetovala tudi s šolsko pedagoginjo, ki se je z mojo izbiro strinjala. 050 Stili zaznavanja in diferenciacija Učenec Vidni tip Telesno-gibalni tip Slušni tip Tip Učenec/ka 1 40 30 28 vidni Učenec/ka 2 44 38 35 vidni Učenec/ka 3 37 32 38 kombinacija vidnega in slušnega Učenec/ka 4 46 28 39 vidni Učenec/ka 5 28 32 29 telesno-gibalni Učenec/ka 6 37 50 33 telesno-gibalni Učenec/ka 7 43 31 35 vidni Učenec/ka 8 35 44 24 telesno-gibalni Učenec/ka 9 41 49 46 telesno-gibalni Učenec/ka 10 34 33 36 slušni Učenec/ka 11 30 35 34 kombinacija slušnega in telesno-gibalnega Učenec/ka 12 29 37 34 telesno-gibalni Učenec/ka 13 37 24 26 vidni Učenec/ka 14 37 37 33 kombinacija vidnega in telesno-gibal-nega Učenec/ka 15 29 24 25 vidni Učenec/ka 16 45 34 34 vidni Razlaga rezultatov po zgornjem vprašalniku Točke, ki jih učenec zbere pri izjavah 2., 3., 7., 11., 14., 20., 24., 28., 30., 32. in 33. seštejemo ter jih dodelimo vidnemu tipu. Podobno vsoto točk zbranih ob izjavah 4., 9., 10.,12., 13., 15., 16., 18., 21., 23. in 25. dodelimo telesno-gibalnemu tipu. In nazadnje še točke za slušni tip. Dobimo jih kot seštevek točk ob izjavah 1., 5., 6., 8., 17.,19., 22., 26., 27., 29. in 31. Ugotovitve Vsak izmed nas izkazuje mešanico vseh treh stilov zaznavanja, vendar pri večini eden prevladuje. Seštevek točk, ki je večji od 40, kaže na posebej močno izražen stil zaznavanja. V skupini sodelujočih sedmošolcev je torej: sedem otrok vidnega tipa, pet telesno-gi-balnega tipa, en otrok je slušni tip. Pri treh posameznikih sta izrazitejša dva stila hkrati, tako ne moremo govoriti o enem prevladujočem. Izid se je v večini primerov ujemal z težav 051 hitro ugotovili tudi, kateri je moj prevladujoči stil zaznavanja. Ugotovitev, dobljenih z vprašalnikom, ni mogoče neposredno prevesti v napotke za učenje posameznika. Pomembno je, da se zavedamo, da ne gre pri učenju le za kvantitativne razlike med učenci (eni se slabše učijo, drugi bolje, eni hitreje, drugi počasneje), ampak so med njimi tudi kvalitativne razlike, kar pomeni, da se različni ljudje različno učijo in drugače spoznavajo. b) Izvedba učne ure Pri učni uri usvajanja nove učne snovi (Težišče trikotnika) sem za usvojitev istega cilja predvidela različne učne poti, delovna sredstva, pripomočke, navodila, in sicer tako, da bi zadostila vsem trem učnim tipom. Vizualni učenci najučinkoviteje predelujejo informacije, kadar vidijo, kaj se učijo (berejo, opazujejo, pišejo). Slišnim učencem ustrezajo ustne razlage, poslušanje navodil. Kinestetični učenci se najbolje učijo, kadar lahko delajo s predmeti, se dotikajo, premikajo ali celo hodijo po razredu. V učni uri učenci sledijo istemu cilju (opredeliti in določiti težišče trikotnika), razlikujejo pa se načini procesiranja podatkov. Na podlagi rezultatov, dobljenih z vprašalnikom, sem učence razvrstila v tri skupine. Zaradi različnih pristopov k določanju težišča trikotnika sem se odločila za heterogene skupine. V vsaki je bil vsaj po en predstavnik vsakega zaznavnega tipa. Na ta način so se učenci med seboj dopolnjevali, si pomagali in sodelovali. Pozorna sem bila tudi na to, da so bile vse tri skupine učno približno enako močne. Potrebščine za delo: - model trikotnika narisan na kosu lepenke, škarje; - delovni list za vsakega posameznika v skupini; - mali prenosni računalnik z nameščenim programom GeoGebra. Učenci so potrebovali še vsak svoje geometrijsko orodje. Učenec - vidni tip je naglas prebral navodila za delo v skupini - tudi zaradi učencev, ki so slišni tipi. Reševanje prve naloge Učenca - telesno-gibalni tip sta iz lepenke izrezala trikotnik in mu s pomočjo konice svinčnika poiskala podporno točko tako, da je trikotnik miroval v ravnovesni legi. Točko sta označila in poimenovala T. Reševanje druge naloge To nalogo rešujejo vsi učenci v skupini (ne glede na zaznavni tip), saj je temeljni učni cilj te ure trikotniku načrtovalno določiti težišče. Ker je trikotnik na delovnem listu skladen s trikotnikom iz lepenke, učenci preverijo, ali sta točki v obeh primerih identični. [Slika 3] Načrtovanje težišča 052 Stili zaznavanja in diferenciacija [Slika 4] Reševanje naloge z računalniškim programom [Slika 5] Zaslonsko okno programa GeoGebra Reševanje tretje naloge Pri risanju trikotnika in njegovega težišča s pomočjo programa za dinamično geometrijo GeoGebra se učenci izmenjujejo, si medsebojno pomagajo (drug drugega učijo uporabe računalniškega programa) in opa- Delovni list za delo v skupini List z navodili za delo v skupini zujejo premikanje težišča, če premikajo eno od oglišč trikotnika. Pred koncem učne ure predstavnik skupine poroča. Skupaj oblikujemo povzetke. 7. razred 1. naloga Določanje težišča modelu trikotnika iz lepenke Iz lepenke izrežite model trikotnika. Model nato previdno postavite na konico svinčnika tako, da bo stal v ravnovesni legi. Podporno točko, v kateri model miruje, označite s T. 53 2. naloga Določanje težišča trikotniku s pomočjo načrtovanja Rešite nalogo na delovnem listu. Načrtujte natančno in po korakih. Primerjajte točko T, dobljeno s pomočjo svinčnika (iz prve naloge), s tisto, ki ste jo dobili s pomočjo načrtovanja (iz druge naloge). Kaj ugotovite? 3. naloga Določanje težišča trikotnika z računalniškim programom GeoGebra. V programu GeoGebra ( www.geogebra.org ) načrtajte poljuben trikotnik in mu določite težišče. Označite eno od oglišč trikotnika in ga premikajte po risalni površini. Opazujte, kaj se dogaja s težiščem. Ali je težišče trikotnika lahko zunaj njegove notranjosti? Delovni list za individualno delo List z navodili za individualno delo 7. razred Določanje TEŽIŠČA trikotnika s pomočjo načrtovanja Težišče trikotnika poiščemo na naslednji način. 1. Poiščemo razpolovišča vseh treh stranic. To naredimo s pomočjo simetral daljice. Dobljena razpolovišča po vrsti označimo, na primer: središče stranice c: K središče stranice a: L središče stranice b: M 2. Narišemo daljice, ki povezujejo središče daljice z nasprotnim ogliščem. Daljico, ki povezuje središče stranice z nasprotnim ogliščem, imenujemo težiščnica. Težiščnice smiselno označimo: težiščnica na c: tc (povezuje točko K in oglišče C) težiščnica na a: ta (povezuje točko L in oglišče A) težiščnica na b: tb (povezuje točko M in oglišče B) Kaj opazite?_ 054 Stili zaznavanja in diferenciacija Dopolni: 1 Sklep dela 055 v priprave na pouk, še posebno da zadosti tistemu stilu zaznavanja, ki ga sam najmanj uporablja. Ker se pedagog z različnostjo v razredu sooča vsak dan, je prav, da svoj reper- 8 Viri in literatura toar postopkov, metod in pristopov nenehno širi in s tem zagotavlja upoštevanje enakih možnosti in drugačnosti za vse učence. 1. Heacox, D. (2009). Diferenciacija za uspeh vseh. Ljubljana: Rokus Klett. 2. Kramar, M. (2004). Didaktični vidiki diferenciacije pouka. V: Kramar, M. (ur.), Diferenciacija in nivojskipouk v prenovljeni šoli. Ljubljana: založništvo Supra. 3. Marentič - Požarnik, B. (2000). Psihologija učenja in pouka. Ljubljana: DZS, 1. izdaja 4. Pravilnik o izvajanju diferenciacije pri pouku v osnovni šoli (Ur. l. RS, št. 63, 16. 6. 2006). 5. Strmčnik, F. (1987). Sodobna šola v luči učne diferenciacije in individualizacije. Ljubljana: Zveza organizacij za tehnično kulturo. 6. Zakon o spremembah in dopolnitvah Zakona o osnovni šoli (Ur. l. RS, št. 60/2006). 7. Zakon o osnovni šoli (Ur. l. RS, št. 12/96). 8. Interno gradivo s študijskih skupin za matematiko v OŠ v šol. letu 2009/2010 9. http://www.cpi.si/files/cpi/userfiles/Publikacije/ Mapa_ucnih_dosezkov1.pdf 10. http://www.geogebra.org 056 Stili zaznavanja in diferenciacija f Povezovanje matematičnih in strokovnih znanj v programu lesarski tehnik Linkage of mathematical and professional knowledge in program for woodworking technician I Povzetek V prispevku vam predstavljam primer poklicne situacije, ki sem jo poimenovala lesena strešna konstrukcija. S situacijo bom pokazala, kako zagotavljam diferenciacijo znanj glede na zmožnosti in prizadevanja dijakov (osnovna in nadaljevalna znanja). Pri reševanju situacij dijaki lahko uporabljajo različne vrste tehnologije (grafična računala in program Graph) in s tem razvijajo tudi zmožnost uporabe tehnologije pri reševanju situacij. Ključne besede: povezovanje znanj, poklicna situacija, uporaba tehnologije matematike I Abstract In the article we present an example of a professional problem which I have named the wooden roof construction. With the help of this situation, I will try to show how I assure differentiation of knowledge depending on the abilities and endeavors of pupils (basic and advanced knowledge). Pupils can use different kinds of technology (graphic calculators and the program Graph) in solving problems, developing their ability to use technology for resolving different problems in the process. Keywords: integrating knowledge, professional situation, use of technology mathematics. a Uvod 0 Nevenka Križman Šolski center Ljubljana, Srednja lesarska šola Sem profesorica matematike in fizike na Srednji lesarski šoli v Ljubljani. Poučujem že šestnajsto leto, večinoma matematiko v Matematika v šoli ~ XVIII. [2012] ~ 057-067 srednjih poklicnih, strokovnih in poklicno--tehniških programih dijake, ki se izobražujejo za poklic mizar in lesarski tehnik. V zadnjih letih se je šola opremila z informacijsko tehnologijo, npr. računalniki, grafična računala, interaktivne table ... Omenjeno informacijsko tehnologijo smiselno uvajam v pouk, saj mi ponuja nešteto možnosti, kako matematiko približati dijakom in jo narediti tudi zabavno. Poučevanje s tehnologijo mi predstavlja velik izziv, saj se mi pri pripravi učnih gradiv porajajo ideje, pri katerih uporabljam veliko poklicnih in življenjskih situacij, ob katerih bodo dijaki lažje razmišljali in videli smisel matematike. b Situacija Navajam situacijo iz stroke s katerimi povezujem: - matematična znanja znotraj same matematike in - matematična in strokovna znanja. težavnosti. Različica 1 je lažja, različica 2 pa težja. Pri reševanju različice 1 so dijaki lahko poljubno izbirali naloge glede na svoje zmožnosti in znanje. Pri različici 2 so dijaki vodeno reševali naloge. Iz situacije (1. in 2. različica lesene strešne konstrukcije) lahko pripravimo več izpitnih listkov za ustni del poklicne mature iz matematike po novem modelu, z upoštevanjem navodil in priporočil iz predmetnega izpitnega kataloga. LESENA STREŠNA KONSTRUKCIJA -RAZLIČICA 1: Lesena strešna konstrukcija je določena z linearnima funkcijama f(pč) = —:c + 3 in g(x) = -|x + 3 ter *-osj°. 2 1. Nariši prečni prerez podstrešja in določi največjo možno višino podstrešja ter dolžini posameznih špirovcev1. Merilo: 1 enota = 1 m Situacija izhaja iz stroke, ki je dijakom dobro razumljiva, tako da ob njej lahko dijak samostojno matematično razmišlja. Prikazala vam bom dve različici lesene strešne konstrukcije, kjer gre za razliko v V slovenskem knjižnem jeziku ima špirovec enak pomen kot šperovec (špirovec = špero-vec). V besedilu je izraz špirovec, ker se bolj uporablja pri vsakdanjem pogovoru v lesarski stroki. Pravopis pa nas napoti na škarnik. [Slika 1] Podstrešja Vir slike: http://www.instalater.si/slike/instalater_3/ streha_0_copy_2.jpg (17.1.2011) Vir slike: http://www.academia.si/clanek/72/image001.jpg (17.1.2011) 058 Povezovanje matematičnih in strokovnih znanj v programu lesarski tehnik 2. Izračunaj ploščino lika, ki ga predstavlja prečni prerez podstrešja. 3. Izračunaj naklonska kota obeh špirov-cev na minuto natančno. 4. Izračunaj kot slemena (kot med špi-rovcema) na desetinko stopinje. 5. Izračunaj, za koliko se spremeni na-klonski kot strehe, če vrh slemena zvišamo za 1 m. 6. Izračunaj, za koliko se spremeni na-klonski kot strehe, če vrh slemena znižamo za 1 m. 7. Izračunaj površino strehe, če je njena dolžina 20 m. 8. Izračunaj prostornino podstrehe, če je dolžina strehe 20 m. 9. Izračunaj, na kolikšni razdalji od sle-menske lege še lahko stoji človek, ki je visok 1,8 m. LESENA STREŠNA KONSTRUKCIJA -RAZLIČICA 2: Lesena strešna konstrukcija je določena z linearnima funkcijama /(x) = x + 2 in g(x) = -jX + 5 ter x-osjo. 1. Nariši prečni prerez podstrešja in določi največjo možno višino podstrešja ter dolžini posameznih špirovcev. Merilo: 1 enota = 1 m 2. Izračunaj ploščino lika, ki ga predstavlja prečni prerez podstrešja. 3. Izračunaj naklonska kota obeh špirov-cev na minuto natančno. 4. Izračunaj kot slemena (kot med špi-rovcema) na desetinko stopinje. 5. Izračunaj, za koliko se spremeni na-klonski kot strehe, če vrh slemena zvišamo za 1 m. 6. Izračunaj, za koliko se spremeni na-klonski kot strehe, če vrh slemena znižamo za 1 m. 7. Izračunaj površino strehe, če je njena dolžina 20 m. 8. Izračunaj prostornino podstrehe, če je dolžina strehe 20 m. 9. Izračunaj, na kolikšni razdalji od sle-menske lege še lahko stoji človek, ki je visok 1,8 m. Matematične teme, ki jih situacija vsebuje, so naslednje: - števila (računanje s števili, zaokroževanje in ocenitev podatkov); - funkcije, enačbe in diferencialni račun (linearna funkcija - graf linearne funkcije, koeficient k in začetne vrednosti n, naklon premice, kot med premicama ); - geometrija (notranji in zunanji koti v trikotniku, pojem sokota, skladnosti, kotne funkcije, Pitagorov izrek, pojem dolžine, ploščine likov, površine in prostornine teles, podobnost trikotnikov). Te matematične teme sem povezala s stroko oziroma z nekaterimi pojmi, ki jih dijaki srečujejo pri svojem prihodnjem poklicu, in sicer s podstreho, prečnim prerezom strehe, poimenovanjem posameznih delov strehe, kot so stropnik, špirovec, sleme ... Y Reševanje situacije Predstavljam vam različne poti reševanja, ki so jih ubrali moji dijaki in kaj sem pri njihovem delu spremljala. Pri pripravi situacij sem bila pozorna tudi na cilje preverjanja po predmetnem izpitnem katalogu 2011 (v nadaljevanju PIK 2011), ker želim zagotoviti sprotno in kakovostno pripravo dijakov na poklicno maturo. Zaradi raznolikosti situacij in zaradi spreminjanja področja spremljanja sem opredelila navidezno težavnost in takso-nomske stopnje po Gagneju. 59 Pri leseni strešni konstrukciji, ki zajema osnovna znanja, bom za vsako alinejo opisala: 1. Možne poti reševanja (grafično ali računsko) 2. Cilje preverjanja po PIK-u 2011 3. Navidezna težavnost (opredeljena subjektivno glede na izkazano znanje dijakov) Lestvica navidezne težavnosti: - lahka - srednje težka - težka 4. Taksonomske ravni po Gagneju (Vir: Cotič, M., Žakelj, A. - Gagnejeva takso-nomija pri preverjanju in ocenjevanju matematičnega znanja, Sodobna pedagogika 1/2004, 182-191) Gagnejeva taksonomska lestvica: Osnovna in konceptualna znanja - osnovna znanja in vedenja - konceptualna znanja Proceduralna znanja - rutinska proceduralna znanja - kompleksna proceduralna znanja Problemska znanja - strategije reševanja problemov - aplikativna znanja Osnovna in konceptualna znanja - Osnovna znanja in vedenja Osnovno znanje in vedenje obsega poznavanje pojmov, priklic dejstev. Razdelimo ga lahko na štiri elemente: - poznavanje posameznosti - reproduktiv-no znanje, - poznavanje specifičnih dejstev - znanje definicij, formul, - poznavanje terminologije - seznanjenost s simboli terminologije, - poznavanje klasifikacij in kategorij - poznavanje matematičnih objektov (množice, enačbe ... ) - Konceptualno znanje Konceptualno znanje obsega razumevanje pojmov, dejstev. Razdelimo ga na več elementov: - prepoznavanje pojmov - predstava - prepoznavanje terminologije in simbolike v dani situaciji - definicije in izreki - povezave Proceduralna znanja Proceduralno znanje zajema poznavanje in obladovanje algoritmov. Delimo ga na dva elementa: - rutinsko proceduralno znanje - reševanje nesestavljenih nalog, nalog z malo podatki, - kompleksno proceduralno stanje - poznavanje in obladovanje postopkov in metod, reševanje sestavljenih nalog z več podatki. Problemska znanja Problemsko znanje je sposobnost uporabe obstoječega znanja v novih situacijah. Temeljni elementi problemskega znanja so prepoznava problema, postavitev smiselnih vprašanj, preveritev podatkov, strategija reševanja, komunikacijskih, miselnih, operacijskih procesov, procesov zapisovanja, uporaba znanja, miselne spretnosti, metako-gnitivne zmožnosti (utemelji svoja stališča). 060 Povezovanje matematičnih in strokovnih znanj v programu lesarski tehnik LESENA STREŠNA KONSTRUKCIJA - Lesena strešna konstrukcija je določena RAZLIČICA 1: z linearnima funkcijama f{x) = -x + 'i in g(x) = ~x + 3 ter *-osj°. 2 1. Nariši prečni prerez podstrešja in določi največjo možno višino podstrešja ter dolžini posameznih špirovcev. Merilo: 1 enota = 1 m 1. Možne poti reševanja: Nalogo lahko rešuje grafično in računsko. Dijak nariše grafa danih linearnih funkcij v isti pravokotni koordinatni sistem. Označi prečni prerez podstrešja in ugotovi, da je dobljeni lik enakokraki trikotnik. Grafični način reševanja: Izmeri eno dolžino špirovca, saj sta enako dolga ter največjo možno višino podstrešja. Računski način reševanja: Dijak računsko poišče presečišče danih linearnih funkcij. Ugotovi, da druga koordinata presečišča predstavlja največjo možno višino podstrešja. Izračuna presečišče ene dane linearne funkcije z x -osjo. Prva koordinata te točke predstavlja polovico dolžine stropnika. Dolžino špirovca pa lahko izračuna na dva načina. 1. način: Uporabi obrazec za izračun razdalje med dvema točkama. 2. način: Uporabi Pitagorov izrek. 2. Cilji preverjanja znanja po - narisati graf linearne funkcije PIK-u 2011: - določiti ničlo in začetno vrednost funkcije - rešiti sistem dveh linearnih enačb - izračunati razdaljo med dvema točkama v ravnini - ločevati vrste trikotnikov glede na stranice in kote - uporabljati Pitagorov izrek 3. Navidezna težavnost: - lahka 4. Taksonomske ravni: - problemska znanja 2. Določi ploščino lika, ki ga predstavlja prečni prerez podstrešja. 1. Možne poti reševanja: Ploščino lika lahko izračuna na dva načina. 1. način: Uporabi formulo za izračun ploščine trikotnika: osnov-nica x pripadajoča višina /2, ter izračuna njegovo ploščino. 2. način: Uporabi Heronov obrazec. 2. Cilji preverjanja znanja po PIK-u 2011: - uporabljati lastnosti trikotnika - računati ploščino trikotnika pravokotnika - poznati enote za merjenje ploščine 3. Navidezna težavnost: - lahka 4. Taksonomske ravni: - proceduralna znanja 61 3. Izračunaj naklonska kota obeh špirovcev na minuto natančno. 1. Možne poti reševanja: Naklonska kota lahko izračuna na dva načina. 1. način: Uporabi kotne funkcije v pravokotnem trikotniku in upošteva skladnost kotov. 2. način: S smernimi koeficienti danih linearnih funkcij in upošteva pojem sokota. Dijak lahko tudi izmeri velikost kota, da preveri, ali se izračunana vrednost kota ujema z izmerjeno. 2. Cilji preverjanja znanja po PIK-u 2011: - pozna kotne funkcije ostrih kotov v pravokotnem trikotniku in jih znati uporabljati - računati s koti - pozna enoto za merjenje kotov - smiselno zaokroževati - oceniti rezultat - pozna pomen konstante k 3. Navidezna težavnost: - težka 4. Taksonomske ravni: - problemska znanja 4. Izračunaj kot slemena (kot med špirovcema) na desetinko stopinje. 1. Možne poti reševanja: Kot slemena lahko izračuna na dva načina. 1. način: Naklonska kota že pozna. Upošteva, da je vsota notranji kotov 180°. 2. način: Uporabi obrazec za izračun kota med dvema premicama. Dijak lahko tudi izmeri velikost kota, da preveri, ali se izračunana vrednost kota ujema z izmerjeno. 2. Cilji preverjanja znanja po PIK-u 2011: - uporabljati lastnosti trikotnika - računati s koti 3. Navidezna težavnost: - lahka 4. Taksonomske ravni: - osnovna znanja in vedenja 5. Izračunaj, za koliko se spremeni naklonski kot strehe, če vrh slemena zvišamo za 1 m. 1. Možne poti reševanja: Na podoben način kot prej lahko izračuna nov naklonski kot špirovca in nato razliko. 2. Cilji preverjanja znanja po PIK-u 2011: - pozna kotne funkcije ostrih kotov v pravokotnem trikotniku in jih znati uporabljati - računati s koti - pozna enoto za merjenje kotov - smiselno zaokroževati - oceniti rezultat - pozna pomen konstante k 3. Navidezna težavnost: - srednje težka 4. Taksonomske ravni: - problemska 062 Povezovanje matematičnih in strokovnih znanj v programu lesarski tehnik 6. Izračunaj, za koliko se spremeni naklonski kot strehe, če vrh slemena znižamo za 1 m. 1. Možne poti reševanja: Ponovi postopek reševanja, kot ga je izvajal pri zvišanju slemena za 1 m. 2. Cilji preverjanja znanja po PIK-u 2011: - pozna kotne funkcije ostrih kotov v pravokotnem trikotniku in jih znati uporabljati - računati s koti - pozna enoto za merjenje kotov - smiselno zaokroževati - oceniti rezultat - pozna pomen konstante k 3. Navidezna težavnost: - srednje težka 4. Taksonomske ravni: - problemska znanja 7. Izračunaj površino strehe, če je njena dolžina 20 m. 1. Možne poti reševanja: Ugotovi, da je streha sestavljena iz dveh pravokotnikov. Ena stranica tega pravokotnika predstavlja dolžino strehe, druga pa dolžino špirovca. Najprej izračuna ploščino enega pravokotnika, nato pa še površino strehe. 2. Cilji preverjanja znanja po PIK-u 2011: - pozna in uporablja lastnosti pokončnih teles (prizme) - pri ustreznih podatkih za dano telo izračunati površino telesa 3. Navidezna težavnost: - lahka 4. Taksonomske ravni: - konceptualna znanja 8. Izračunaj prostornino podstrehe, če je dolžina strehe 20 m. 1. Možne poti reševanja: Ugotovi, da ima podstrešje obliko prizme, katere osnovna ploskev je prečni prerez strehe, višina prizme pa dolžina strehe. Nato izračuna prostornino strehe. 2. Cilji preverjanja znanja po PIK-u 2011: - pri ustreznih podatkih za dano telo izračunati prostornino telesa 3. Navidezna težavnost: - srednje težka 4. Taksonomske ravni: - problemska znanja 9. Izračunaj, na kolikšni razdalji od slemenske lege še lahko stoji človek, ki je visok 1,8 m. 1. Možne poti reševanja: Tukaj pa uporabi podobnost trikotnikov. 2. Cilji preverjanja znanja po PIK-u 2011: - pozna in uporablja definicijo podobnosti trikotnikov 3. Navidezna težavnost: - težka 4. Taksonomske ravni: - problemska znanja 63 5 Reševanje situacije s pomočjo programa Graph Posamezne alineje situacije lesena strešna konstrukcija - različico 1 so dijaki reševali tudi z uporabo programa Graph. Za uspešno poklicno delo se dijaki tudi pri matematiki učijo uporabljati računalniške programe za reševanje matematičnih »orehov«. Slika prikazuje prečni prerez podstrešja, ki ga dobi z vnosom funkcij. Določi presečišče - druga koordinata presečišča je največja možna višina podstrešja. Kako so uporabljali program Graph za reševanje situacije lesena strešna konstrukcija - različico 2, vam bom prikazala v nadaljevanju: LESENA STREŠNA KONSTRUKCIJA -RAZLIČICA 2: Lesena strešna konstrukcija je določena z linearnima funkcijama f{x)-x + 2 in g(x) = --x + 5 ter x-osjo. v iTTTTiMiTR^ir ~ ---1 ' I - " 1 ■ Slika prikazuje dolžino krajšega špirovca, ki jo odčita, ko vnese mejo od -2 do 2. Na podoben način dobimo tudi dolžino daljšega špirovca (meja od 2 do 10). 1. Nariši prečni prerez podstrešja in določi največjo možno višino podstrešja ter dolžini posameznih špirovcev. Merilo: 1 enota = 1 m 2. Izračunaj ploščino lika, ki ga predstavlja prečni prerez podstrešja. Določi ploščino pod krajšim Določi ploščino pod daljšim Sešteje obe ploščini. špirovcem. špirovcem. 064 Povezovanje matematičnih in strokovnih znanj v programu lesarski tehnik 3. Izračunaj za koliko se spremeni naklonski kot strehe, če vrh slemena zvišamo za 1 m. Določi vrh slemena ter vnese Iz trendne črte odčita smerni Na enak način ponovi posto-in izriše niz točk. Nato izbere koeficient in s pomočjo ra- pek za krajši špirovec. trendno črto. čunala izračuna kot in spre- membo naklona strehe pri daljšem špirovcu. 4. Izračunaj, za koliko se spremeni naklonski kot strehe, če vrh slemena znižamo za 1 m. To reši na podoben način kot pri zvišanju slemena. 5. Izračunaj, na kolikšni razdalji od slemenske lege še lahko stoji človek, ki je visok 1,8 m. Vnese linearno funkcijo h(x) = 1,8, ki predstavlja višino človeka. Določi in odčita desno presečiščno točko ter izračuna razdaljo od slemena. Na podoben način določi in odčita levo presečiščno točko ter izračuna razdaljo od slemena. 65 e Sklep Dijaki so pri reševanju takih situacij motivirani, zanima jih, so vedoželjni. Vidijo, da je matematika mnogo več kot le reševanje rutinskih nalog. Dejavno sodelujejo, saj jih usmerjam in jim posredujem namige za iskanje poti reševanja situacije. Ob uporabi tehnologije, kot je na primer program Graph, pridejo hitreje do rezultatov - manj »peš« računajo. Kljub temu pokažejo, da imajo veliko matematičnega znanja in spretnosti ter da ta znanja in spretnosti znajo tudi uporabiti. Učenje in poučevanje matematike ob uporabi tehnologije je zanimivejše, nazornejše, kakovostnejše in razvija tudi druga matematična znanja in spretnosti, ki so zapisana v katalogu znanj za matematiko v programih srednjega strokovnega in poklicno-tehniške-ga izobraževanja. Pri pripravi in osmišljanju situacij sodelujem s kolegi, ki poučujejo strokovno-tehni-ške predmete. Dijak se lahko ob tako med predmetno pripravljenih situacijah uri v različnih poteh reševanja in sam presodi, kdaj in kako povezuje pridobljena matematična in strokovna znanja ter spretnosti pri reševanju situacij. Pri tem razvija potrebo: - po avtonomnosti (npr. pri reševanju izzivov, ki jih bo najbrž nekoč kot obrtnik dobil od svojih strank), - po kompetentnosti (npr. za povezovanje znanj in vseživljenjsko učenje) - in pripadnosti bodočem poklicu. Z Viri in literatura 1. Katalog znanja ključne kvalifikacije Matematika, Srednje poklicno izobraževanje, sprejet na Strokovnem svetu RS za splošno izobraževanje, 15. 2. 2007. 2. Katalog znanja za matematiko v programih srednjega strokovnega izobraževanja, sprejet na Strokovnem svetu RS za splošno izobraževanje,15. 2. 2007. 3. Pravilnik o poklicni maturi. Uradni list Republike Slovenije. Št. 44/2008 z dne 7. 5. 2008. 4. Pravilnik o spremembah in dopolnitvah Pravilnika o poklicni maturi. Uradni list Republike Slovenije. Št. 9/2009 z dne 6. 2. 2009. 5. Suban Ambrož, M. (2011). Spremembe in novosti na poklicni maturi iz matematike. Matematika v šoli. ZRSŠ. Ljubljana. 6. Magajna, Z. (2005). Razvoj pouka matematike v poklicnih in srednjih strokovnih šolah. Matematika v šoli. ZRSŠ. Ljubljana. 7. Kmetič, S. (2008): Vloga računalniške učne tehnologije, Vzgoja in izobraževanje, Vol. XXXIX, No. 5, str. 066 Povezovanje matematičnih in strokovnih znanj v programu lesarski tehnik 8. Cotič, M., Žakelj, A. - Gagnejeva taksonomija pri preverjanju in ocenjevanju matematičnega znanja, Sodobna pedagogika 1/2004, 182-191) 9. Razvojni projekt v okviru projekta Skriti zaklad (2002-2004). Grafična žepna računala pri pouku matematike v srednji poklicni šoli. 10. Cirkovic, S. Grafična žepna računala pri pouku matematike v srednji poklicni šoli. Matematika v šoli 12 (2005), številka 3,4, str. 208-215. 11. Rojko, C.(2007), Mednarodni pilotni raziskovalni projekt, Matematično izobraževanje v rokah učencev. 12. Sambolic Beganovic, A. Zakaj vpeljati grafična računala v pouk matematike? Zbornik/Mednarodna konferenca Splet izobraževanja in raziskovanja z IKT -SIRIKT 2008, Kranjska Gora, 16.-19. april 2008. Mag. Mojca Orel, Maja Vreča, Saša Matjašič, Maja Kosta. Ljubljana: Arnes, 2008, 107 13. Rojko, C. (2008): Razvoj uporabe IKT pri pouku matematike, Vzgoja in izobraževanje, Vol. XXXIX, No. 5, str. 59-66. 14. Bačnik, A. (2008): Didaktični potencial interaktivnih tabel, Vzgoja in izobraževanje, Vol. XXXIX, No. 5, str. 20-24. 15. Bačnik, A. (2007): Elektronske table - aktivno ali interaktivno? V: Zbornik SIRIKT 2007. Uredili: Vreča, M., Bohte, U., Arnes. Ljubljana str. 84-88. 16. Sambolic Beganovic, A. (2008): Kako pri pouku matematike uporabljam interaktivno tablo? V:Zbornik SIRIKT 2008. Uredili:Orel, M., Vreča, M., Lenarčič, A., Kosta, M., Arnes. Ljubljana 64-65. 17. Sambolic Beganovic, A., Rupnik Rožmanec, B. (2010): Uporaba IKT kot pomoč dijakom z učnimi težavami. V: Zbornik SIRIKT 2010. Uredili: Lenarčič, A., Kosta, M., Blagus, K., Miška, d. o. o., Ljubljana, str. 236. 18. Geršak, M., Prošek, M. (2005). Lesarstvo: zbirka nalog. Ljubljana: Zveza lesarjev Slovenije. Lesarska založba. Matematika - pomemben pripomoček dobremu slaščičarju Mathematics - important help to confectioner Rosana Vrh Makarovič Tehniški šolski center Nova Gorica Biotehniška šola Šempeter pri Gorici Z Povzetek Pri urah matematike se velikokrat srečam z vprašanjem dijakov, zakaj neki morajo imeti matematiko, saj bi bili radi samo slaščičarji, kuharji, vrtnarji ... Dejstvo, da je matematika pot do znanja vsakemu dobremu mojstru, jih ne prepriča. Pri svojih urah skušam matematiko s prilagojenimi nalogami približati njihovemu poklicu, kar mi včasih bolj, drugič manj uspe. V sodelovanju z učitelji praktičnega pouka smo projektne dneve slaščičarjev namenili povezovanju znanj matematike in dela v slaščičarski delavnici. Ključne besede: odstotni račun, sklepni račun, povezovanje znanj I Abstract During mathematics lessons I must often answer the pupils' question, why on earth they need mathematics when they just want to become pastry men, cooks, gardeners... The fact that mathematics is a way to knowledge for every good craftsman doesn't convince them. At my lessons I try to accommodate mathematics to their chosen profession with adapted exercises, a task at which I am sometimes more and sometimes less successful. In cooperation with teachers of practical work we dedicated the confectioners' project days to connecting the Matematika v šol i ~ XVIII. [2012] ~ 068-072 knowledge of mathematics with the skills needed in a confectionery shop Keywords: percent invoice, rule of three, integrating knowledge a Uvod - Matematika v slaščičarstvu Smiselno bi bilo, da se najprej opravijo izračuni, šele nato sledi praktična izvedba dela (pripraviti je treba količino sestavin, zgnesti testo, oblikovati, aranžirati ...). S kolegi cama praktičnega pouka sem se dogovorila, da to opravimo v nasprotnem vrstnem redu. Dijaki so najprej opravili vaje v slaščičarski delavnici. Nato smo se pri urah matematike prepričali, da bi imeli veliko manj težav, če bi že prej znali svoje znanje iz matematike uporabiti pri praktičnem pouku. Dijaki so reševali naloge po skupinah, glede na letnik, ki so ga obiskovali. b Izvedba učne ure - Znanje matematike pomaga 1 Naloge za 1. letnik - odstotni in sklepni račun 1. naloga: Kupec slaščičarskemu mojstru naroči pladenj peciva za 25 ljudi. Želel je, da bi bilo 30 % princes krofov, 40 % sadnih rezin, preostalo pa naj bo orehova rulada. Koliko posameznih mini desertov mora biti na pladnju? Dijaki vedo, da se za eno osebo računata 2 mini deserta, kar je približno 100 g, torej mora biti na pladnju 50 mini desertov. Izračun: 50 mini des...........100 % x sad.rezin..............40 % 50 ■ 40 % 100 % x = 20 sadnih rezin 50 mini des...........100 % x princ.krof..............30 % x = 50 ■ 30 % 100 % x = 15 princes krofov orehova rulada 100 % - (30 %+40 %) = 30 % x = 50 ■ 30 100 % x = 15 orehovih rulad Na pladnju bo 20 sadnih rezin, 15 princes krofov in 15 orehovih rulad. [Slika 1] Mini deserti po naročilu 69 2. naloga: Z maso osnovnega recepta za jogurtovo strjenko lahko napolnimo dva silikonska modela za strjenko. Dobili smo naročilo za 30 jogurtovih strjenk. Koliko sestavin moramo uporabiti? Dijaki vedo, da z enim silikonskim modelom naredijo 6 strjenk. Osnovni recept za jogurtovo strjenko Sestavine: 500 ml navadnega jogurta 9 g želatine v listih 100 g sladkorja 9 g vanilin sladkorja 500 ml sladke smetane 1118 ml zmesi (zaradi poenostavitve dela slaščičarji enačijo ml z g) Izračun: 500 ml nav. jogurta.........12 strjenk x ml nav. jogurta............30 strjenk 500 ml- 30 12 x = 1250 ml navadnega jogurta Z enakimi izračuni dobimo še potrebne količine preostalih sestavin. Za 30 jogurtovih strjenk torej potrebujemo: 1250 ml navadnega jogurta 22,5 g želatine v listih 250 g sladkorja 22,5 g vanilnega sladkorja 1250 ml sladke smetane 2 Naloge za 2. letnik - odstotni račun in linearna funkcija 1. naloga: Iz polovice narejenega kvašenega testa smo naredili tri potice. Četrtino tega testa smo porabili za makovo, četrtino za ocvirkovo, preostanek pa za orehovo potico. [Slika 4] Testo za potico 070 Matematika - pomemben pripomoček dobremu slaščičarju Kolikšna je bila skupna masa testa in koliko smo ga porabili za makovo oz. ocvirkovo potico, če vemo, da smo ga za orehovo potico porabili 1,5 kg? Izračun: orehova potica 1,5 kg...........25 % skupaj_x kg...........100 % 1,5 kg -100 % x= - 25% x = 6 kg testa Skupna masa testa je bila 6 kg. Makove in ocvirkove potice je skupaj toliko kot orehove, kar pomeni, da je vsake 75 dag . 2. naloga: Slaščičarsko podjetje za dostavo peciva prodajalnam v mestu za prevoz zaračuna 50 €. Za vsak kupljen kg peciva odšteje pri prevozu 0,007 €. Koliko znaša cena prevoza pri nakupu 25 kg peciva? Izračun: f(x) = kx + n x ... kupljeno pecivo (25 kg) k ... 0.007 € n ... 50 € f(x) = 50 - 0.007 ■ 25 f(x) = 50 - 0,175 f(x) = 49,83 € Prodajalna plača za prevoz 25 kg peciva 49,83 € . 3 Naloge za 3. letnik -geometrijska telesa 1. naloga: Spekli smo 7 cm visoko, okroglo torto premera 26 cm. Koliko čokoladnega preliva potrebujemo za okrasitev torte, če za 1 cm2 potrebujemo 0,2 ml preliva? [Grafikon 1 ] Delež testa za potico Izračun: Računamo površino valja brez ene osnovne ploskve P = mr2+ 2 jirv P = 7T-132+ 2TT-13-7 P = 531 cm2+572 cm2 P = 1103 cm3 1103 cm2........x ml 1 cm2.......0.2 ml = 221 ml Za okrasitev torte potrebujemo 221 ml čokoladnega preliva. 2. naloga: V skledi imamo 2795 ml zmesi za jogurtovo strjenko. Radi bi jo vlili v silikonske modele polkrogle oblike s premerom 7 cm. Koliko silikonskih modelov potrebujemo? Dijaki poznajo silikonske modele - v vsakega lahko vlijejo 6 strjenk. 1 4tit3 1 cm3 .„.1 ml 90 cm3....90 ml V = 2 V = 90 cm3 . torej prostornina ene strjenke 2795 ml : 90 ml = 31 polkrogel 1 silikonski model.... 6 strjenk x silikonskih modelov......31 strjenk 71 t \ [Slika 5] Torta s čokoladnim prelivom [Slika 6] Silikonski model za strjenko polkrogle oblike 31 x = 5,17____potrebujemo 5 silikonskih modelov Z 2795 ml jogurtove strjenke lahko napolnimo 5 silikonskih modelov. Y Sklep Zastavljeni način dela je pritegnil večino dijakov, ki so sodelovali v skupinah. Poznali so že rezultate dela v slaščičarski delavnici, zato so se z vnemo lotili izračunov. Zelo jih je zanimalo, ali pridejo do enakih zaključkov. Med samim delom je bilo nekajkrat slišati vzklik:»Saj res, to je to!« Ob koncu dela smo bili vsi zadovoljni. Učiteljici praktičnega pouka sta predvidevali, da bo delo v slaščičarski delavnici potekalo hitreje in z manj zapletov pri preračunavanju sestavin. Dijaki so ugotovili, da si z znanjem matematike vendarle lahko olajšajo delo in da matematika ni tak bavbav, kot jih je večina mislila. 072 Matematika - pomemben pripomoček dobremu slaščičarju Naravoslovje kot medpredmetno zasnovan izbirni predmet v drugem letniku gimnazije Science as an intercurricularly based optional subject in 2nd grade of Gimnasia I Povzetek Medpredmetne in kurikularne povezave ter timsko poučevanje so gotovo zelo pomembni elementi razvoja sodobne šole. Zato smo se na Gimnaziji Ravne odločili, da dijakom drugega letnika ponudimo medpredmetno zasnovan izbirni predmet Naravoslovje v obsegu dveh ur tedensko. Pri Naravoslovju dijaki pridobivajo naravoslovna znanja skozi avtentično in izkustveno učenje ter razvijajo zmožnosti za ustvarjalno in učinkovito uporabo naravoslovja v vsakdanjem življenju. 70 ur letno si enakovredno razdelimo štirje predmeti, in sicer matematika, fizika, kemija in biologija. V obliki timskega poučevanja se priključi profesorica nemščine in jim predstavi strokovno terminologijo v nemškem jeziku. Povezovalni element vseh štirih predmetov je vsebina. Članek opisuje Naravoslovje z organizacijske in izvedbene plati, cilje, ki jih pri predmetu dosegamo in odzive dijakov ter vključenih učiteljev. Ključne besede: Naravoslovje,timsko delo, medpredmetno povezovanje, eksperimentalno delo, IKT, avtentično in izkustveno učenje. I Abstract Team teaching, intersubject and intercurricular cooperation are definitely the most important elements in the development of the modern school. That is why our team at Gimnazija Ravne decided to offer pupils of the second grade an intercurricular- Matematika v šol i ~ XVIII. [2012] ~ 068-072 Simona Vreš Gimnazija Ravne na Koroškem -based optional subject called Science. The subject Science, based on the gymnasium syllabus for the second grade, was offered to our pupils as a two-hour per week optional subject. With the help of intercurricular cooperation at science lessons, the second grade students are acquainted with the role of science in their everyday life. Every year, seventy lessons are equally distributed between four science subjects (mathematics, physics, chemistry and biology), and successfully implemented on the basis of the team-teaching method which also enables the joining in of a German language teacher, who tries in an original way to acquaint pupils with the adequate terminology in the German language. Content is connecting element of all four subjects. This paper describes natural science from an organizational and operational side and also introduces the goals achieved within the subject, as well as responses of students and teachers involved. Key words: Science, teamwork, intercurricular cooperation, experimental work, ICT and authentic learning experience. a Uvod Naraščajoča diferenciacija in mladostniki, ki so zrasli s televizijo, računalnikom in med-mrežjem, ki stvarnost doživljajo kot celoto, ne pa strukturirano po merilih posameznih disciplin, terjajo od nas večjo avtentičnost pouka. Ta že po definiciji terja intenzivnejše in globlje povezave med šolskimi predmeti. Današnjim mladostnikom bi bilo smiselno ponuditi novo učno okolje, ki omogoča izkustveno pridobivanje znanja za reševanje realnih življenjskih vprašanj in podpira pristope, ki vključujejo avtentično učenje z medpredmetnim načrtovanjem in timskim sodelovanjem. Tudi posodobljeni učni načrti, ki so nastali v sklopu projekta Posodobitev gimnazij, vsebujejo številne novosti, med drugim: - razvoj kompetenc, - cilje medpredmetnih področij in krosku-rikularnih tem, - vrednotenje dosežkov, - pričakovane dosežke/rezultate, - sistematično poudarjanje uporabe IKT. Vsega zapisanega se na Gimnaziji Ravne še kako dobro zavedamo, zato smo odprti za številne spremembe in novosti. Zavedamo se, da moramo vsaj v določeni meri opustiti klasičen način poučevanja in dijakom omogočiti, da z uporabo sodobne tehnologije skozi avtentično in izkustveno učenje preidejo iz pasivne v aktivno vlogo. Ena izmed novosti na to temo, ki smo jo na Gimnaziji Ravne uspešno uvedli v šolskem letu 2009/2010, je medpredmetno zasnovan izbirni predmet Naravoslovje, ki ga ponujamo dijakom drugega letnika kot alternativo družboslovju in tretjemu tujemu jeziku. V šolskem letu 2010/2011 smo izvajali Naravoslovje v sodelovanju z Zavodom republike Slovenije za šolstvo kot inovacijski projekt z naslovom »SKUPAJ ZMOREMO VEČ« - medpredmetno zasnovan izbirni predmet v drugem letniku gimnazije. 074 Naravoslovje kot medpredmetno zasnovan izbirni predmet v drugem letniku gimnazije b Izhodišča za uvedbo novosti V zadnjih letih je zanimanje za naravoslovje med našimi gimnazijci v upadanju. Ugotavljamo, da dajejo dijaki vse večji pomen družboslovju, hkrati pa imajo pri matematiki in naravoslovnih predmetih več težav, kot so jih imeli pred leti. Dijaki se vse prevečkrat učijo matematiko, fiziko ali kemijo na pamet, brez nekega globljega razumevanja in brez videnja uporabnosti v življenju. Zato je njihovo znanje zgolj površinsko, brez prenosljivosti in povezanosti pridobljenih znanj. Posledično se tudi vse manj dijakov naše gimnazije (pa tudi najbrž marsikatere druge v Sloveniji) odloča za študij na naravoslovnih fakultetah. Na Gimnaziji Ravne se nastalega problema zavedamo, razmišljamo o vzrokih, ki so privedli do takega stanja ter iščemo nove pristope za spodbujanje veselja do naravoslovnih predmetov in uvid v njihovo uporabnost. Razlogov za nastali položaj je najbrž veliko. Omenili bi le tri mogoče razloge za nastali problem. 1. V zadnjih letih se je zanimanje za gimnazijski program močno povečalo, posledično so se ustanavljale nove gimnazije. Število gimnazijskih oddelkov na našem območju se je povečalo in na našo gimnazijo so se začeli vpisovati tudi dijaki z nižjim učnim uspehom v osnovni šoli. Ti dijaki imajo pogosto učne težave ravno pri naravoslovnih predmetih, zato učitelji vedno več časa namenijo ponavljanju in utrjevanju snovi, kar pa ob neustrezni diferenciaciji pouka lahko boljše dijake uspava. Tako kaj hitro postanejo naravoslovni predmeti za nekatere dijake prezahtevni, za druge pa nezanimivi. 2. Prav gotovo so generacije, ki se zdaj vpisujejo v gimnazijski program, drugačne, predvsem bolj radovedne in inovativne. Medmrežje in sodobna tehnologija sta jim dosegljiva 24 ur na dan, zato od nas pričakujejo drugačna znanja. Današnji mladostniki se ob frontalnem podajanju snovi (kamor sodi tudi uporaba predstavitev v Power Pointu) dolgočasijo. Namesto transmisijskega pouka si želijo sodobnih metod, dejavnega sodelovanja v učnem procesu, uporabe sodobne tehnologije pri pouku, eksperimentiranja. Dijaki danes od učitelja pričakujejo predvsem, da jim je mentor pri njihovem lastnem raziskovanju. Drugačno učno okolje, ki omogoča dijakom izkustveno pridobivanje znanja za reševanje realističnih situacij, bi prav gotovo pri marsikaterem dijaku spodbudilo veselje do naravoslovnih predmetov ter pripomoglo k razvijanju njegovih naravoslovnih in matematičnih kompetenc. 3. Današnji gimnazijec si želi uporabnih znanj. Zelo pogosto je med dijaki vprašanje: »Kdaj bom pa pozneje v življenju to potreboval?«. Dijaki pri klasičnem načinu poučevanja ne vidijo pomembne vloge naravoslovja v vsakdanjem življenju. Doseganje ciljev posameznih predmetov skozi reševanje realističnih problemov in celosten pogled z različnih zornih kotov na problem dajeta dijaku možnost spre-videti uporabnost naravoslovja v življenju. Dijakom lahko s skupno načrtovanimi in timsko izpeljanimi dejavnostmi ustvarimo razmere za učinkovitejšo prenosljivost in povezanost znanja. Na Gimnaziji Ravne smo se odločili, da poučevanjem, z uporabo IKT in eksperimentalnim delom razvijemo učno okolje, ki bi ga veljalo vključevati v učni proces. Naravoslovje smo ponudili dijakom drugega letnika kot dve urni izbirni predmet predvsem z naslednjimi nameni: - Iskanje novih vsebin in pristopov, ki bodo vključevali izkustveno in avtentično učenje z medpredmetnim načrtovanjem in timskim sodelovanjem. Ugotavljali smo namreč, da dijaki nimajo razvitega pozitivnega odnosa do naravoslovja in da pridobljenega znanja pri posameznih predmetih ne znajo uporabiti pri drugih predmetih. Konkretno nas je zmotilo, da npr. dijak določene računske operacije pri matematiki izvede brez posebnih težav, pri fiziki ali kemiji pa v podobni situaciji odpove. - Ustvarjanje novega učnega okolja za poučevanje naravoslovnih vsebin, ki omogoča dijakom izkustveno pridobivanje znanja za reševanje realističnih situacij. Opazili smo, da imajo dijaki težave pri prenosu teoretičnega znanja v uporabno znanje in zmožnost reševanja realističnih problemov. Znanje, pridobljeno po tej poti, bo kakovostnejše in trajnejše. - Dvig motivacije in odkrivanje skritih sposobnosti tako dijakov kot učiteljev. Z ustrezno načrtovanimi makrodejavnost-mi in smiselno uporabo sodobne tehnologije lahko naredimo pouk privlačnejši, z akcijskim raziskovanjem pa postopoma spreminjamo način razmišljanja in uvajamo spremembe. - Z interdisciplinarnimi povezavami lahko bistveno pripomoremo k uresničitvi cilja - razvoja naravoslovnih in matematičnih kompetenc, ki je sicer zapisan kot cilj štiriletnega poučevanja posameznih naravoslovnih predmetov. Y Načrtovanje Ker gre za medpredmetno zasnovan predmet, je treba delo dobro načrtovati. Nujni so redni sestanki tima izvajalcev. Delo za šolsko leto 2010/2011 smo začeli načrtovati že maja 2010. Po zbranih prijavah dijakov se je izkazalo, da je zanimanje precejšnje in da bomo dijake razdelili v dve skupini. Seveda je bil potreben poseg v urnik, saj moramo biti hkrati prosti vsi izvajalci in vsi dijaki drugega letnika. Najprimernejša rešitev se nam je zdela blo-kura, po možnosti na začetku ali koncu pouka . Tako smo imeli Naravoslovje na urniku ob torkih prvi dve uri. Junija smo začeli načrtovati, kako se bomo interdisciplinarno povezali in dejavnosti tudi timsko poučevali. Tako smo izpolnili skupno tabelo za načrtovanje, v katero smo zapisali povezovalni element, skupne cilje, skupne dejavnosti, pričakovani skupni rezultat, vsebinske, procesne in funkcionalne cilje po posameznih predmetih, dejavnosti za uresničevanje teh ciljev in pričakovani rezultat. Za vse štiri predmetne sklope smo izdelali časovni načrt izvajanja posameznega predmeta. Del tabele za načrtovanje, ki se nanaša na povezovalni element, skupne cilje, skupne dejavnosti, pričakovani skupni rezultat: 5 Izvedba pouka Izbirni predmet Naravoslovje je v šolskem letu 2010/2011 izbralo 41 dijakov, ki smo jih razdelili v dve skupini. Vsak dijak, vključen v Naravoslovje, je šel skozi vse štiri predmetne sklope po naslednjem terminskem načrtu: 076 Naravoslovje kot medpredmetno zasnovan izbirni predmet v drugem letniku gimnazije TABELA ZA SKUPNO NAČRTOVANJE Povezovalni element: Vsebina : gibanje Skupni cilji: • Pridobivanje naravoslovnih znanj skozi avtentično in izkustveno učenje. • Iskanje novih pristopov za spodbujanje veselja do naravoslovnih predmetov. • Razvoj naravoslovnih in matematičnih kompetenc. • Razvijanje zmožnosti za ustvarjalno in učinkovito uporabo pridobljenih znanj iz naravoslovja. Skupne dejavnosti: • Uporaba IKT • Obdelava istih podatkov pri različnih predmetih. • Priprava končne predstavitve izdelkov dijakov. • Medpredmetne povezave. • Timsko poučevanje. Pričakovani skupni rezultati: V predmetnik drugošolcev gimnazijskega programa bomo vnesli nekaj novega. Na ravni medpredmetnih povezav bodo dijaki spoznavali vlogo naravoslovja v vsakdanjem življenju. Dijaki bodo na isti problem pogledali z različnih zornih kotov. Pričakujemo, da: • bodo dijaki tako dobili celovitejši pogled na dani problem, • bomo dijakom približali naravoslovje, za katerega v zadnjih letih zanimanje upada, • bomo z medpredmetnimi povezavami dosegli večjo povezanost in prenosljivost znanja. [Tabela 1] Tabela za skupno načrtovanje vsi predmeti 1. Skupina: 7. 9. 2010 do 2. 11. 2010 FIZIKA - 16 ur 9. 11. 2010 do 4. 1. 2011 KEMIJA - 16 ur 11. 1. 2011 do 15. 3. 2011 MATEMATIKA -16 ur 22. 3. 2011 do 17. 5. 2011 BIOLOGIJA -16 ur 2. Skupina: 7. 9. 2010 do 2. 11. 2010: KEMIJA - 16 ur 9. 11. 2010 do 4. 1. 2011 FIZIKA - 16 ur 11. 1. 2011 do 15. 3. 2011 BIOLOGIJA -16 ur 22. 3. 2011 do 17. 5. 2011 MATEMATIKA -16 ur Ure od 24. 5. 2011 do 7. 6. 2011 so bile namenjene pripravi zaključnih predstavitev, zadnji dve uri 14. 6. 2011 pa so se dijaki predstavili celotnemu profesorskemu zboru in zunanjim gostom. Pri vseh predmetih se je timsko vključila profesorica nemškega jezika in dijakom omogočila spoznavanje strokovnih terminov v nemškem jeziku. Pouk ni potekal na klasičen način, ampak so bile vse ure naravoslovja namenjene samostojnemu raziskovanju dijakov z metodami, ki jih omogoča sodobna tehnologija. Zaradi dobre opremljenosti šole smo si pri naravoslovju lahko privoščili marsikaj. 77 [Slika 1] Dijaki pri delu v mobilni učilnici računalniških učilnic na razpolago še neke vrste mobilno učilnico. To pomeni, da imamo 20 prenosnih računalnikov, ki so namenjeni izključno dijakom za delo pri pouku. Pri fiziki in matematiki je bila zato v ospredju uporaba sodobne informacijsko- -komunikacijske tehnologije. Pri fiziki so se dijaki, opremljeni s kamero, najprej odpravili na teren (igrišče) in posneli različne primere gibanja tako žoge kot telesa. Opremljeni z dobrimi posnetki, so se odpravili nazaj v učilnico obdelat zbrane podatke. Vsak na svojem prenosniku so z uporabo programa Logger Pro raziskovali vlogo fizike pri različnih primerih gibanja v športu. Tudi pri matematiki smo pozabili na nerešljive enačbe, neživljenjske probleme, zeleno tablo in kredo. Namesto tega smo uporabili računalnike in interaktivno tablo. Ukvarjali smo se z raziskovanjem realističnih problemov. Vsak dijak je na svojem prenosniku z uporabo programa Graph iskal uporabnost matematike v vsakdanjem življenju. Pri raziskovanju so si pomagali z naslednjim učnim listom. [Slika 2] Obdelava podatkov s programom Logger Pro - tir gibanja žoge pri odbojki 078 Naravoslovje kot medpredmetno zasnovan izbirni predmet v drugem letniku gimnazije UPORABA PROGRAMA GRAPH Odprite program GRAPH. NALOGA 1 1. V koordinatni sistem narišite pet različnih točk z enako absciso 4. (Funkcija/Vstavi zaporedje točk) - Za slog točke uporabite krogec rdeče barve velikosti 3. - Točke poimenujte A,B,C,D,E, jih zapišite s pisavo rdeče barve velikosti 14. (Funkcija/ Vstavi oznako ali izberi ikono A) 1 ■ ■ 1 -i 2. V koordinatni sistem narišite tri različne točke ^(5-^,-2 75), G (-5 02,—) in H(10 J,3) - Za slog točke uporabite kvadratek zelene barve velikosti 3. 3. Narišite daljico zelene barve širine 4 s krajiščema M(-6,l), N(8,2) 4. Narišite množice točk {T(x,y)} v ravnini, ki zadoščajo naslednjim pogojem: (Funkcija/ Vstavi relacijo) {T(x, y); x = 3} (polna črta modre barve širine 5) {T(x, y); y = 4} (polna črta kostanjeve barve širine 5) {T(x, y); (x < -2) A (y > 3)} (vijola barve) Pri vsaki narisani množici točk v enaki barvi s pisavo Arial zapišite pogoj, ki množico določa. (Funkcija/Vstavi oznako ali izberi ikono A) 5. Narišite množice točk {T(x,y)} v ravnini, ki zadošča pogoju: {T(x, y); (-4 < x < -1) A (y < -4)}. 6. Narišite kvadrat svetlo modre barve z dolžino stranice 2 enoti, ki bo imel središče v koordinatnem izhodišču. 7. Shranite datoteko na disk z imenom Vajal. NALOGA 2 1. S programom Graph narišite sliko. - Robovi kvadrata naj bodo rdeči, pravo kotnik naj bo obarvan zeleno. 2. Shranite datoteko na disk z imenom Vaja2 NALOGA 3 1. Odprite novo datoteko. 2. Narišite graf funkcije f(x) = -2x + 1. (Funkcija/Vstavi funkcijo) - Premica naj bo rdeče barve, narisana s črto širine 5. - Na graf zapišite funkcijski predpis z rdečo barvo. (Funkcija/Vstavi oznako ali izberi ikono A) 3. Tabelirajte funkcijo na intervalu [-5,5] s korakom 1. (Računaj/Tabela) - Tabelo prenesite v koordinatni sistem (Kopiraj (označi tabelo+CTRL C) /Funkcija/ Vstavi oznako/Prilepi (CTRL V)) 4. Iz tabele odčitajte koordinati točke A z absciso -2 in koordinati točke B z ordinato -3. 5. Izračunajte razdaljo med točkama A in B. (Računaj/Dolžina poti) 6. Določite ničlo funkcije f(x). (Računaj/Ovrednoti/Lepi na x os) 7. Izračunajte ploščino lika, ki ga omejuje premica y = -2x + 1 z obema koordinatnima osema. (Računaj/Ploščina) 8. Narišite graf funkcije g(x), ki ima isto začetno vrednost kot funkcija f(x) in ima ničlo x = -4. - Premica naj bo zelene barve, narisana s črto širine 5 in opremljena s funkcijskim predpisom enake barve. 9. Narišite graf funkcije h(x), ki je vzporeden grafu funkcije f(x) in ima začetno vrednost 3. - Premica naj bo modre barve, narisana s črto širine 5 in opremljena s funkcijskim predpisom enake barve. 10. Odčitajte koordinati presečišča med grafoma funkcij g(x) in h(x). (Računaj/Ovrednoti/ Lepi na presečišče) 11. Shranite datoteko na disk z imenom Vaja3. NALOGA 4 1. Odprite novo datoteko. ^ Narišite graf linearne funkcije /(%) = —x-3. Postavite pet vprašanj o lastnostih funkcije f(x) in s pomočjo programa Graph odgovorite na zastavljena vprašanja. NALOGA 5 1. Odprite novo datoteko. 2. Narišite graf funkcije f(x)=\x - 1\ - 4 (Funkcija/Vstavi funkcijo/f(x)=abs(x-1)-4) - Graf naj bo rdeče barve, narisan s črto širine 5. - Na graf zapišite funkcijski predpis z rdečo barvo. (Funkcija/Vstavi oznako ali izberi ikono A) 080 Naravoslovje kot medpredmetno zasnovan izbirni predmet v drugem letniku gimnazije 3. Tabelirajte funkcijo na intervalu [-5,5] s korakom 1. (Računaj/Tabela) 4. Iz tabele (tabelo prenesete v koordinatni sistem) odčitajte koordinati točke A z absciso -2 in koordinati točke B z absciso 2. 5. Izračunajte razdaljo med točkama A in B. (Računaj/Dolžina poti) 6. Določite ničlo funkcije f(x). (Računaj/Ovrednoti/Lepi na x os) 7. Izračunajte ploščino lika, ki ga omejuje graf funkcije f z abscisno osjo. (Računaj/Ploščina) 8. Narišite graf funkcije g(x)=x - 3| - 4. - Graf naj bo zelene barve, narisan s črto širine 5 in opremljen s funkcijskim predpisom enake barve. 9. Narišite graf funkcije h(x)^x - ^ + 1. - Graf naj bo modre barve, narisan s črto širine 5 in opremljen s funkcijskim predpisom enake barve. 10. Odčitajte koordinati presečišča med grafoma funkcij f(x) in g(x). (Računaj/Ovrednoti/ Lepi na presečišče) 11. Shranite datoteko na disk z imenom Vaja5. NALOGA 6 1. Odprite novo datoteko. r 1 47 2. V koordinatni sistem narišite grafa funkcij f(x) = -x + 35 in g(x) = ~2X+ ~2~ 3. Spremenite nastavitve koordinatnega sistema tako, da bosta grafa vidna. 4. Odčitajte koordinati presečišča med grafoma funkcij. 5. Določite ploščino lika, ki ga omejujeta oba grafa in ordinatna os. NALOGA 7 Če višino odraslega človeka, merjeno v centimetrih, zmanjšamo za 100, dobimo njegovo normalno maso, merjeno v kilogramih. Idealna masa je enaka 90 % normalne mase. 1. Odprite novo datoteko. 2. Zapišite odvisnost idealne mase odraslega človeka od njegove višine in jo grafično ponazorite. (Funkcija/Vstavi funkcijo) 3. Spremenite nastavitve koordinatnega sistema tako, da bo graf pregleden na intervalu [150,200]. 4. Tabelirajte funkcijo na intervalu [150,200] s korakom 1. 5. S pomočjo tabele izpišite idealno maso 180 cm visokega odraslega človeka. 6. Kako visok naj bi bil odrasel človek z idealno maso 63 kg? 7. Raziščite ta realni problem za otroke. 8. Shranite datoteko na disk z imenom Vaja7. NALOGA 3 S pomočjo programa Graph lahko določimo linearno funkcijo, katere graf poteka skozi dve različni točki. 1. Odprite novo datoteko. 2. V koordinatni sistem narišite točki A(1,1) in B(3,3). 3. Narišite premico rdeče barve širine črte 2, ki poteka skozi dani točki. (Funkcija/Vstavi trendno črto/linearna) 4. Odčitajte predpis za dobljeno linearno funkcijo. 5. Narišite še premico modre barve širine črte 2, ki poteka skozi točki C(-1,2) in D(3,0) ter odčitajte predpis za dobljeno linearno funkcijo. 6. Shranite datoteko na disk z imenom Vaja8. NALOGA 9 Usain Bolt je zmagal in postavil svetovni rekord v teku na 100 m na olimpijskih igrah leta 2008 v Pekingu. Vmesni časi njegovega rekordnega teka so zapisani v spodnji preglednici. 10 m 20 m 30 m 40 m 50 m 60 m 70 m 80 m 90 m 100 m 182 2'80 3'75 4'65 5'50 6'33 7'14 7'95 8'76 9'69 1. Odprite novo datoteko. - V koordinatni sistem narišite ustrezne točke. - Izdelajte model Boltovega teka. Model predstavite z grafom in zapišite funkcijski predpis dobljene prilagoditvene funkcije. - Tabelirajte dobljeno funkcijo na intervalu [0,200] s korakom 5. 2. Iz tabele odčitajte, v kolikšnem času je Bolt pretekel prvih 15 m in prvih 45 m. 3. Iz tabele odčitajte, koliko metrov je Bolt pretekel v 7 s. 4. Iz tabele odčitajte, v kolikšnem času bi Bolt pretekel 200 m. Ali je ocena realna? Zakaj? NALOGA 10 Čez nekaj mesecev bo tekaško tekmovanje. Ana, Bor in Ema želijo na tekmovanju sodelovati. Ker morajo biti za tekmovanje dobro pripravljeni, so si izdelali vsak svoj načrt priprav. Vsi trije začnejo s pripravami isti teden. - Ana se je odločila, da prvi teden preteče 10 km, nato pa vsak naslednji teden 1 km več. - Bor se je odločil, da preteče v začetnem tednu le 4 km, potem pa razdaljo povečuje za 2 km tedensko. - Ema se je odločila, da preteče prvi teden le 5 km, potem pa razdaljo povečuje za 20% tedensko. 082 Naravoslovje kot medpredmetno zasnovan izbirni predmet v drugem letniku gimnazije Izpolnite naslednjo tabelo. 1. teden 2. teden 3. teden 4. teden 5. teden Ana Bor Ema 1. Odprite novo datoteko. 2. V koordinatni sistem narišite ustrezne točke Aninega teka. - Izdelajte model Aninega teka. Model predstavite z grafom in zapišite funkcijski predpis dobljene prilagoditvene funkcije.((Funkcija)/(Vstavi) (trendno črto)/(Linearna)) - Graf naj bo modre barve narisan s širino črte 3 in opremljen s funkcijskim predpisom enake barve. Podatek RA2=1 pove, da se premica popolnoma prilega danemu zaporedju točk. - Iz grafa odčitajte, koliko je Ana pretekla 8. teden. (Računaj/(Ovrednoti)) - Iz grafa odčitajte, kateri teden je Ana pretekla 16 kilometrov. 3. V koordinatni sistem narišite ustrezne točke Borovega teka. - Izdelajte model Borovega teka. Model predstavite z grafom in zapišite funkcijski predpis dobljene prilagoditvene funkcije. Premica naj bo rdeče barve narisana s širino črte 3 in opremljen s funkcijskim predpisom enake barve. - V katerem tednu bosta Ana in Bor pretekla enako število kilometrov? ((Računaj)/Ovre dnoti/(Lepi na presečišče))) 4. Shranite datoteko na disk z imenom Vaja 10. NALOGA 11 1. Odprite novo datoteko. 2. V koordinatni sistem narišite ustrezne točke Eminega teka. 3. Model Eminega teka predstavite s premico in zapišite funkcijski predpis dobljene prilagoditvene funkcije. Vrednost R2 v spisku funkcij pove, da se premica ne prilega najbolje danemu zaporedju točk. 4. Raziščite še ostale matematične modele za Emin tek in ugotovite najboljši model. NALOGA 12 1. Odprite datoteko Vaja 10. 2. Narišite v koordinatni sistem najboljši model za Emin tek. 3. Graf naj bo zelene barve, narisan s širino črte 3 in opremljen s funkcijskim predpisom enake barve. 5. Iz grafa odčitajte, kateri teden je Ema pretekla več kot 17 kilometrov. 6. Tekmovanje bo 11. teden od začetka Aninih, Borovih in Eminih priprav. Koliko kilometrov so Ana, Bor in Ema pretekli en teden pred tekmovanjem? 7. V katerem tednu bo število kilometrov, ki jih bo pretekla Ema, preseglo število kilometrov, ki jih bo pretekla Ana? 8. Zapišite interval, na katerem so ti trije modeli realni za dani problem. NALOGA 13 1. V programu LOGGER PRO odprite datoteko, na kateri imate shranjene podatke o položaju žoge pri odbojki. 2. V program GRAPH prenesite naslednje podatke: x stolpec = čas y stolpec = višina žoge ((CTRC/Odpri program Graph/Funkcijaj/Vstavi zaporedje točk/CTRV). 3. Poiščite prilagoditveno funkcijo, ki se najbolj prilega danemu zaporedju točk. Dani model predstavite z grafom in zapisom funkcijskega predpisa. 4. Postavite tri smiselna vprašanja in na njih odgovorite. Učni list zajema vse dejavnosti pri matematiki v obsegu 16 ur. Kot je razvidno iz učnega lista, so dijaki v zadnji 13. nalogi podatke, pridobljene pri fiziki iz programa Logger Pro, prenesli v program Graph in z matematičnega vidika pogledali na gibanje žoge. Ali res žoga pri odbojki potuje po krivulji, ki je graf tiste zapletene funkcije, katere ne bom potreboval nikoli v življenju? Pri biologiji in kemiji so bile v ospredju laboratorijske vaje in eksperimentalno-razi-skovalni pristop. Sodobno opremljen laboratorij na naši šoli ponuja veliko možnosti za raziskovanje. Ker smo vključili še računalnike, je bilo delo posebej zanimivo. Pri kemiji so dijaki posebej raziskali pomen prehrane športnika. Iskali so povezavo med športnikom in njegovo prehrano Delo je potekalo v obliki laboratorijskih vaj na temo hrana in hranila. Vaje so izvajali v dvojicah. Izvedli so naslednje laboratorijske vaje: energijska vrednost hrane, sestava in lastnosti beljakovin, ogljikovi hidrati, aditivi v hrani in vitamini - vitamin C. Biologijo so začeli z opazovanjem gibanja človeških tkiv (gibanje krvi po telesu, peristaltika črevesa, dihalni epitel in delo mišic). Z dejavnostjo na trenažerjih v fitnescentru na DTK so spoznali posamezne mišice in sklope mišic, ki so dejavne med vadbo. S polarijem so merili srčni utrip in porabo kalorij. Sledila je računalniška obdelava in analiza podatkov. Spoznali so učinke treninga (na mišice, dihalni sistem, krvni obtok), od kod energija za telesno vadbo in koliko kisika porabimo ter kako vadba vpliva na pretok krvi. 084 Naravoslovje kot medpredmetno zasnovan izbirni predmet v drugem letniku gimnazije [Slika 3] Obdelava podatkov s programom Graph - prilagoditvena funkcija (tir gibanja žoge pri odbojki) e Evalvacija Metode evalviranja, ki smo jih uporabili, so bile: - anketni vprašalniki, - ustna refleksija vključenih dijakov, - ustna refleksija vključenih učiteljev, - sklepna predstavitev dijakov. Anketni vprašalnik Za dijake smo pripravili vprašalnike. Na podlagi izpolnjenih vprašalnikov dijakov na začetku in nato ob koncu izvajanja projekta bi izpostavili le nekaj pomembnih dejstev: - uvid v povezanost naravoslovnih predmetov, - uvid v uporabnost v vsakdanjem življenju, - dijaki se s takim načinom dela (izkustveno učenje, uporaba IKT, eksperimentalno delo) veliko več naučijo. Refleksija dijakov Sprotna ustna refleksija dijakov je pokazala zadovoljstvo in visoko motivacijo dijakov za medpredmetne povezave. Posebej nas veseli to, da so po koncu izvajanja projekta vsi dijaki izjavili, da so ponosni na svoje dosežke in bi se, če bi jim bila ta možnost ponujena, takšnega načina pouka še udeležili. Splošno mnenje dijakov je, da so pri Naravoslovju uživali, da so se zaradi smiselne vključitve sodobne tehnologije v pouk in obravnave gibanja z različnih zornih kotov veliko več naučili. Refleksija učiteljev Na podlagi sprotne ustne refleksije učiteljev lahko rečemo, da smo učitelji ob tem dosegli strokovni razvoj za interdisciplinarno povezovanje, timsko poučevanje, uvajanje aktivnega učenja in uporabo sodobne tehnologije. S pomočjo akcijskega raziskovanja smo postopoma spreminjali način razmišljanja o novih načinih poučevanja in uvajali spremembe. Izbirni predmet Naravoslovje smo umestili v predmetnik tako, da smo dijakom omogočili uporabo znanja, pridobljenega pri rednih urah, za reševanje realnih situacij, pri tem pa nismo povečali njihove tedenske obremenjenosti. Ugotovili smo, da je vrstni red predmetov pomemben. Tako je dobro, da sta fizika in bi- ologija na vrsti pred matematiko, saj se lahko pri matematiki zelo uspešno navežemo na ta dva predmeta. Vsekakor pa smo se strinjali, da je skupno načrtovanje zahteven proces, ki terja veliko časa in usklajevanja, vendar daje zelo dobre rezultate. Sklepna predstavitev dijakov Vsak dijak je dejavno sodeloval pri pripravi in izvedbi sklepne predstavitve. Vse dijake smo ocenili na podlagi vnaprej pripravljenih opisnih kriterijih za pridobitev ocene (izdelava naloge in njena predstavitev). Izkazalo se je, da so dijaki večinoma pridobili odlično oceno ali najmanj prav dobro oceno. Vsak dijak je izdelal svoj primer uporabe pridobljenega znanja ter primer pojasnil odločno in prepričljivo. Vsak dijak je svoj primer predstavil z uporabo IKT-tehnologije ali eksperimenta in je bil pri predstavitvi samostojen. Znal je odgovoriti na postavljena vprašanja sošolcev in profesorja nosilca predmeta. Primer samostojno izdelane naloge pri matematiki: SKOK S PALICO - NALOGE Sergej Bubka je upokojeni ukrajinski »skakalec s palico«. Večkrat je bil izbran za najboljšega svetovnega atleta. Na spletni strani: http://en.wikipedia.org/wiki/Sergey_Bubka poišči njegove rezultate za zunanjo progo (Outdoor) ter dopolni spodnjo tabelo (pri več enakih letnicah prepišeš najboljši rezultat): 1984 1985 1986 1987 1988 1991 1992 1994 Sergej Bubka - Izdelajte model Bubkovih rezultatov. Model predstavite z grafom in zapišite funkcijski predpis dobljene prilagoditvene funkcije (označite z zeleno barvo). - Iz grafa razberite njegov rezultat leta 2011. - Katera krivulja je v realnosti bolj primerna, polinomična ali linearna? Pojasnite! 086 Naravoslovje kot medpredmetno zasnovan izbirni predmet v drugem letniku gimnazije - Na enaki spletni strani poiščite še Sergejeve rezultate za notranjo progo (Indoor). Dopolnite tabelo. 1984 1986 1987 1989 1990 1991 1992 1993 Sergej Bubka - Izdelajte tudi model Bubkovih rezultatov za to progo. Model predstavite z grafom in zapišite funkcijski predpis dobljene prilagoditvene funkcije (označite ga z rdečo barvo). - Primerjajte njegove rezultate. Kaj opazite? - Kakšen rezultat bi na notranji progi lahko pričakovali leta 2034? SKOK S PALICO - REZULTATI - Tabela: 1984 1985 1986 1987 1988 1991 1992 1994 Sergej 5,94 m 6,00 m 6,01 m 6,03 m 6,06 m 6,10 m 6,13 m 6,14 m Bubka - f(x) = 0.0013x2 + 0.25x - 5.779 * kvadratna funkcija (R2 =0.98) - Če gledamo premico, potem bi, po grafu sodeč, bil njegov rezultat 6,4 m, če pa gledamo parabolo, pa je njegov rezultat 5,8 (torej manjši od prejšnjih). - Bolj primerna je linearna, saj se njegovi rezultati izboljšujejo in se bojo verjetno tudi naprej izboljševali. Kvadratna pa nam nakazuje na to, da se rezultati po določenem času začnejo slabšati, so manjši. Vendar pa ta oblika grafično bolj natančno poveže točke med sabo. - Tabela: 1984 1986 1987 1989 1990 1991 1992 1993 Sergej 5,83 m 5,95 m 5,97 m 6,03 m 6,05 m 6,12 m 6,13 m 6,15 m Bubka - f(x) = 0.0014x2 + 0.27x - 7.7 * kvadratna funkcija (R2 =0.98) - V letih do leta 1987je imel na notranji progi slabše rezultate kot na zunanji, istega leta sta se rezultata ujemala, za tem pa je prišlo do spremembe, saj je na notranji progi imel boljše rezultate na zunanji pa slabše (ravno obratno ko na začetku). - 4,7 m (kvadratna funkcija) oz. 7,5 m (linearna funkcija). [Slika 4] Obdelava podatkov s programom Graf- prilagoditvena funkcija (Sergej Bubka - skok s palico) Z Sklep Naravoslovje je predmet, pri katerem dijaki skozi avtentično in izkustveno učenje samostojno odkrivajo uporabnost naravoslovja v vsakdanjem življenju. Predmet je na Gimnaziji Ravne umeščen v gimnazijski predmetnik, in sicer kot dveurni izbirni predmet v drugem letniku, s čimer se dijakom ne poveča tedenski obseg ur. Z interdisciplinarnim in timskim poučevanjem ter izbiro primerne vsebine znotraj Naravoslovja ponudimo dijakom možnost, da dani problem spoznajo z matematičnega, fizikalnega, biološkega in kemijskega vidika, posamezne strokovne izraze pa usvojijo tudi v nemškem jeziku. Iz evalvacije dijakov je razvidno, da nam je v šolskem letu 2010/2011 cilje uspelo precej uresničiti. Naše delo nameravamo na podlagi rezultatov evalvacije in refleksije učiteljev nadgraditi s še bolj usklajeno načrtovanimi dejavnostmi za dijake. Novost, ki smo jo vpeljali, je lahko uporabna za preoblikovanje pouka v gimnazijah, srednjih šolah in tudi v tretji triadi osnovne šole. Na vsaki stopnji si uporabniki izberejo primerno vsebino, ki jo obravnavajo na ustrezni ravni zahtevnosti. Sodobne metode dela, medpredmetno povezovanje, timsko poučevanje in uporaba IKT ponujajo izziv, ne samo učencem in dijakom, ampak tudi njihovim učiteljem. 088 Naravoslovje kot medpredmetno zasnovan izbirni predmet v drugem letniku gimnazije Na Gimnaziji Ravne bo tako zasnovan kom kot alternativa Družboslovju in tretjemu izbirni predmet Naravoslovje ponudba dija- tujemu jeziku tudi v naslednjih šolskih letih. n Viri in literatura 1. Posodobitve pouka v gimnazijski praksi MATEMATIKA, didaktični priročnik (ZRSŠ 2010). 2. Medpredmetne in kurikularne povezave, Priročnik za učitelje (ZRSŠ, 2010). 3. http://www.padowan.dk (21. 8. 2011). 4. http://www.vernier.com (21. 8. 2011). 89 Nekaj več o praštevilih More about the prime numbers Kristijan Breznik Mednarodna fakulteta za družbene in poslovne študije, Celje Janko Marovt Ekonomsko-poslovna fakulteta, Maribor I Izvleček Praštevila imajo v matematiki posebno mesto, zato so, in so bila tudi v preteklosti, zelo dobro proučevana. V delu bomo podali nekaj znanih problemov, trditev in domnev, ki so povezana s praštevili. Večina problemov je zelo enostavno zastavljenih, seveda pa to ne pomeni, da so tudi rešitve enostavne. Ključne besede: teorija števil, praštevila, sestavljena števila. I Abstract Prime numbers have a special place in mathematics and for that reason they have been very well researched. We present some well known problems, statements and conjectures related to prime numbers. Most of the problems are formulated in a simple way, but that doesn't mean that their solutions are simple. Key words: number theory, prime numbers, composite numbers. a Uvod Matematika je kraljica znanosti, teorija števil pa je kraljica matematike. Tako je nekoč trdil in zapisal slavni nemški matematik Johann Carl Friedrich Gauss (1777-1855). Matematika v šoh -o XVIII. [25162] ~ 595-596 Ker je teorija števil zelo stara matematična veda, so matematiki na tem področju odkrili veliko zanimivega, vendar pri tem naleteli tudi na veliko problemov. V delu bomo podali nekaj znanih primerov, domnev in izrekov iz teorije števil. Nekateri primeri so že dokazani ali ovrženi, nekateri so še vedno odprti, prav vsi pa bodo tako ali drugače povezani s praštevili. Praštevila zavzemajo v teoriji števil prav posebno vlogo. Njihove lastnosti s pridom izkoriščajo v kriptografiji in teoriji kodiranja. Nekaj osnovnih idej uporabe si lahko pogledamo v [4] in [5]. Učenci se srečajo s praštevili že v osnovni šoli in nato praviloma svoje znanje o njih nadgradijo v prvem letniku srednje šole. Delo pred vami je lahko predvsem smiselna dopolnitev k predpisani učni snovi, lahko pa je tudi motivacija za vpeljavo snovi o prašte-vilih. Osnovnošolsko razumevanje matematičnih pojmov presega le zadnje poglavje o praštevilskem izreku. V uvodu poglejmo še nekaj lastnosti pra-števil, ki so bralcu verjetno že dobro znane, vendar si bo lahko vseeno malo osvežil spomin. V vsakem nekoliko boljšem srednješolskem učbeniku lahko najdemo enostaven dokaz, da je vseh praštevil neskončno mnogo. Prvi dokaz o tem je bil najden že v Evkli-dovih Elementih, ki so nastali še pred našim štetjem. Precejšen korak k temu, kako hitro narašča število praštevil (o čemer govori zadnje poglavje), je napravil Euler, ki je dokazal, da je vsota recipročnih vrednosti praštevil 11111 - + - + — + — + — + ••• 3 5 7 11 13 divergentna. Posledično vrsta recipročnih vrednosti vseh naravnih števil, imenovana tudi harmonična vrsta, prav tako divergira. b Goldbachova domneva Ena najbolj znanih in najstarejših še odprtih domnev v povezavi s teorijo števil je Goldbachova domneva. Leta 1742 je manj znani pruski matematik Cristian Goldbach (1690-1764) pisal pismo Eulerju (1707-1783), ki je bil takrat avtoriteta na matematičnem področju, v katerem mu je omenil svojo domnevo, da lahko vsako liho naravno število večje od 5 zapiše kot vsoto treh praštevil. Izvirno pismo si lahko pogledamo na spletnem naslovu [6]. Euler je v odgovoru to izjavo malce dopolnil, in danes je ta problem znan kot Goldbachova domneva: Vsako sodo naravno število, večje od 2, je vsota dveh pra-števil. Domneva, zapisana v zgornji obliki, je v matematiki znana kot krepka Goldbachova domneva. Problem, zapisan v izvirni obliki, kot ga je Eulerju v pismu predstavil Goldbach, pa je znan kot šibka Goldbachova domneva. Seveda bi iz dokaza krepke Goldbachove domneve takoj sledilo, da velja tudi šibka Gold-bachova domneva. Namreč, če lahko vsako sodo število, večje od 2, zapišemo kot vsoto dveh praštevil, potem z dodajanjem števila 3 dobimo vsa liha naravna števila večja od 5. Poglejmo nekaj konkretnih primerov zapisa sodega naravnega števila, večjega od dve, kot vsote dveh praštevil, torej krepke Goldba-chove domneve: 4 = 2 + 2 6 = 3 + 3 8 = 3 + 5 10 = 3 + 7 = 5 + 5 12 = 5 + 7 14 = 3 + 11 = 7 + 7 16 = 3 + 13 = 5 + 11 18 = 5 + 13 = 7 + 11 91 Nekatera večja soda naravna števila lahko zapišemo kot vsoto dveh praštevil tudi na več načinov, recimo 42 = 5 + 37 = 11 + 31 = 13 + 29 = 19 + 23. Problem je na videz zelo preprost, vendar ga matematikom vse do danes ni uspelo razrešiti. Kljub temu v matematičnih krogih prevladuje mišljenje, da je Goldbachova domneva pravilna. Pri preverjanju domneve si matematiki zadnje čase pomagajo z vedno zmogljivejšimi računalniki. Z njimi so do julija leta 2008 domnevo že preverili za vsa soda naravna števila, ki so manjša od 1018. Pokazano je bilo celo nekoliko več, namreč da lahko vsa soda naravna števila, večja od 6 in manjša od 33 1 06, zapišemo kot vsoto dveh tujih si praštevil. Toda še tako zmogljiv računalnik odpove pri preverjanju domneve za prevelika števila. Stroga matematična dokazovanja pa gredo ravno v nasprotno smer. Matematiki želijo dokazati, da od nekega števila naprej domneva velja za vsa soda naravna števila. Razlog je v tem, da bi potem za ostala manjša soda naravna števila domnevo preverili neposredno z računom ali s pomočjo vedno zmogljivejših računalnikov. Poglejmo nekaj rezultatov, ki so se približali dokazu pravilnosti Goldba-chove domneve. V tridesetih letih prejšnjega stoletja je bilo pokazano, da lahko vsako sodo število zapišemo kot vsoto največ 300.000 praštevil, kar pa je seveda še zelo daleč od vsote dveh praštevil. Pred nekaj leti so rezultat precej popravili, namreč dokazali so, da je vsako sodo naravno število vsota največ šestih praštevil (Goldbachova domneva govori o dveh praštevilih). Za največji korak proti dokazu Goldbachove domneve štejemo delo kitajskega matematika Jinga Runa Chena, ki je leta 1966 dokazal, da lahko vsako sodo naravno število zapišemo kot vsoto praštevila in števila, ki je produkt največ dveh praštevil [1]. Leta 2000 je bilo ponujenih milijon dolarjev tistemu, ki bi mu uspelo dokazati Goldba-chovo domnevo v dveh letih. Na žalost nihče ni prijavil dokaza, kar verjetno pomeni, da bo Goldbachova domneva ostala pretrd matematični oreh še kar nekaj časa. Med poukom lahko v razredu Goldba-chovo domnevo uporabimo kot zanimivost, lahko pa učence zaposlimo s preverjanjem te domneve za nekatera manjša soda naravna števila in tako hkrati preverjamo njihovo vedenje o praštevilih. Y Praštevilski dvojčki Praštevilski dvojček je praštevilo, za katerega obstaja praštevilo, ki se od njega razlikuje za 2. Najmanjši praštevilski dvojček tvorita števili 3 in 5. Naslednji pari praštevilskih dvojčkov so 5 in 7, 11 in 13, 17 in 19 itd. V povezavi s praštevilskimi dvojčki še vedno obstaja odprta naslednja domneva. Domneva o praštevilskih dvojčkih: Pra-številskih dvojčkov je neskončno mnogo. Čeprav se je veliko matematikov trudilo dokazati to preprosto domnevo, dokaza do danes še niso našli. Poglejmo, kaj se je pomembnega dogajalo v povezavi z njo. Leta 1849 je francoski matematik de Polignac podal splošnejšo domnevo. Trdil je, da za poljubno naravno število k obstaja neskončno mnogo parov praštevil p in p', da je p'- p = 2k V posebnem primeru, ko je k = 1, dobimo ravno prej omenjeno domnevo o praštevilskih dvojčkih. Tudi primera, ko je k = 2 in k = 3, sta dobila svoji imeni. Pri k = 2 govorimo o 092 Nekaj več o praštevilih bratranskih praštevilih, pri k = 3 pa o sexy praštevilih (poimenovanje izhaja iz latinske besede sex, ki pomeni šest). Na primer 7 in 11 sta bratranski praštevili, števili 7 in 13 pa bi naj bili sexy praštevili. Podobno kot za pra-številske dvojčke tudi za bratranska in sexy praštevila matematikom ni uspelo pokazati, da jih je neskončno. V resnici domneva ostaja odprta za vsa naravna števila k. Matematiki so se precej ukvarjali ne samo z dokazovanjem praštevilske domneve, temveč tudi s samim iskanjem praštevilskih dvojčkov. Do zdaj je bil najuspešnejši Francoz Eric Vautier, ki je v začetku leta 2007 našel do zdaj največji praštevilski dvojček. To sta števili 2003663613 ■ 2195000+1 in 2003663613 ■ 2195000 -1. Ti dve naravni števili imata v desetiškem zapisu natanko 58711 števk. Za ilustracijo velikosti teh dveh praštevil lahko povemo, da bi z njunim zapisom skoraj napolnili ves 60-listni zvezek velikega formata. Pomemben rezultat v povezavi s praštevil-skimi dvojčki, ki pa je dokazan, je Brunov izrek. Ta nam pove, da vsota obratnih vrednosti praštevilskih dvojčkov konvergira. Z drugimi besedami povedano, obstaja vsota izraza r i+1 u i+1 u i+1W-L+-L v {3 5 J 1,5 7 J {11 13 J { 17 19 J in je končno število. Omenjeno vsoto imenujemo Brunova konstanta, ki znaša B «1,92160... Ze uvodoma smo povedali, da vsota vseh recipročnih vrednosti praštevil divergira, torej ni končno število. To pomeni, da je praštevilskih dvojčkov precej manj kot praštevil, vendar jih je še zmeraj lahko neskončno mnogo. 5 Bertrandova domneva Čeravno jo matematiki še danes imenujejo Bertrandova domneva, bi to težko zagovar-j ali, saj je ta trditev v celoti dokazana. Tudi Bertrandova domneva je, podobno kot druge tukaj omenjene trditve, zelo preprosto zastavljena. Bertrandova domneva: Med poljubnim naravnim številom, večjim od 1, in njegovim dvakratnikom je vsaj eno praštevilo. Izraz Betrandova domneva se je ohranil iz zgodovinskih razlogov. Francoski matematik Joseph Bertrand (1822-1900) je to domnevo dokazal za vsa naravna števila, manjša od treh milijonov, za preostala pa je le domneval, da ta trditev drži. Svojo domnevo je sicer s pridom uporabljal pri matematičnem raziskovanju. Za manjša naravna števila lahko tudi mi domnevo preverimo z neposrednim računom. Med 2 in 4 je praštevilo število 3. Podobno je med 3 in 6 praštevilo 5 ter med 4 in 8 sta celo dve praštevili, 5 in 7 itd. Seveda pa to ni dovolj, da bi lahko potrdili veljavnost domneve za vsa naravna števila. Prvi, ki je to domnevo v resnici dokazal, je bil ruski matematik Pafnutij Čebišov (1821-1894), in sicer mu je to uspelo sredi devetnajstega stoletja. Nekateri zato menijo, da bi Bertrandovo domnevo lahko imenovali tudi izrek Čebišova. Kot zanimivost lahko omenimo, da je precej krajši dokaz več let pozneje predstavil madžarski matematik Paul Erdos (1913-1996), eden največjih sodobnih matematikov, ki so se ali se še ukvarjajo s teorijo števil. S kratkim dokazom te domneve je potrdil svojo nadarjenost, saj je bil takrat šele osemnajstletni študent. Dokaz Bertrandove domneve, ki temelji na Erdosovem razmišljanju, lahko najdemo na spletnem naslovu Erdos, sta veliko prispevala moderni teoriji števil in ju bomo v nadaljevanju še omenjali. Seveda se matematiki pozneje niso zadovoljili le z dokazom te domneve in so si postavljali nova vprašanja, povezana z njo. Med drugim so dokazali tudi izboljšano Bertran-dovo domnevo, ki pravi, da je med poljubnim naravnim številom n > 3 in številom 2n - 2 vsaj eno praštevilo. Če vzamemo za n = 4, potem je 2n - 2 = 6 in med 4 in 6 je praštevilo 5. Podobno je med 5 in 2 ■ 5 - 2 = 8 praštevilo 7 itd. Seveda je bilo pričakovati, da bodo slej kot prej odkrili tudi kakšen pretrd oreh. Tako obstaja še vedno neodgovorjeno naslednje vprašanje: Ali za poljubno naravno število n, večje od 1, velja, da je med n2 in (« + l)2vsaj eno praštevilo? Tega seveda ni težko preveriti za prvih nekaj naravnih števil. Med 22 = 4 in (2 +l)2 =9 najdemo praštevili 5 in 7. Med 9 in 16 je prav tako nekaj praštevil. Problem nastane pri večjihnaravnih številih, za katera tega vprašanja ni mogoče preveriti niti s pomočjo najzmogljivejših računalnikov. £ Fermatova praštevila in Mersennova število. Pozneje jim je uspelo pokazati, da so Fermatova števila za 6 < n < 16 sestavljena števila. Kljub prizadevanjem jim do danes ni uspelo najti nobenega drugega Fermatovega praštevila, razen petih, prej omenjenih. Hitro pa lahko pokažemo naslednje: če je praštevilo res oblike 2m +1 , potem je število m zagotovo potenca števila 2. Dokaz s protislovjem je zelo kratek in vsebuje razcep dvočlenika. Poglejmo si ga. Denimo, da število m ni potenca števila 2. Potem je m zagotovo deljivo z nekim lihim naravnim številom, recimo k, večjim od 1, kar lahko zapišemo: ( k f m > / ( m\ 2m +1 = 2* + 1 = 2* +1 2k v \ \ \ / f mY"2 2k V V + •••-2* +1 Francoski matematik Pierre S. de Fermat (1601-1665) je znan predvsem po Fermato-vem zadnjem izreku in Fermatovem malem izreku. Nas bo bolj zanimala Fermatova domneva: Števila oblike Fn—22 +1 so praštevila za vsako nenega-tivno celo število n. Za n = 0, 1, 2, 3, 4 res po vrsti dobimo praštevila 3, 5, 17, 257 in 65537. Praštevilom take oblike, Fermatu v čast, danes pravimo Fermatova praštevila, vsem številom oblike 2 +1, pa kar Fermatova števila. Fermatova domneva je bila relativno hitro ovržena. Leta 1732 je že prej omenjeni Euler pokazal, da je F5 =641-6700417 in je tako sestavljeno To pa že pomeni, da je število 2m +1 sestavljeno število in trditev je dokazana. Fermatov rojak in sodobnik je bil Marin Mersenne (1588-1648). Njegova domneva se je glasila tako: Mersennova domneva: števila oblike Mn = 2" — 1 so praštevila za n = 2, 3, 5, 7, 13, 17, 19, 31, 67, 127 in 257 ter sestavljena števila za preostala pozitivna naravna števila, manjša od 257. Števila oblike 2" -1 imenujemo Mersen-nova števila, in če so hkrati praštevila, jim pravimo Mersennova praštevila. Kot bomo videli v naslednjem poglavju, jih pogosto srečamo. Hitro opazimo potrebni pogoj za Mersennova praštevila. Število oblike 2" -1 je praštevilo le, če je tudi število n praštevi-lo. V nasprotnem primeru, torej če je n sestavljeno število, lahko zapišemo n = k ■ m, kjer sta k in m neki naravni števili. Dobimo 094 Nekaj več o praštevilih 2" _i = 2mk -1 = (lm)k -1 = (2m -l)- ((2m)*_1 + (lm)k~2 +••• + 2m +l) Tako je v tem primeru tudi 2" —1 sestavljeno število. Mersennova števila nastopajo tudi drugje. Znano je, recimo, dejstvo, da je za optimalno rešitev matematičnega problema Hanojskih stolpičev z n diski potrebnih natanko Mn korakov. Najdemo jih tudi v računalništvu. Nepredznačeno n-bitno število lahko uporabimo za zapis števil do Mn . Vrnimo se k Mersennovi domnevi. Matematiki so potrebovali približno tri desetletja, da jim jo je uspelo preveriti. Ugotovljeno je bilo, da je Mersenne v svoji domnevi zagrešil pet napak. Med praštevila je uvrstil števili M67 in M257 , ki sta sestavljeni števili, in med praštevili izpustil M6l ,MS9 in Mm7. Sicer ni znano, kako je Mersenne prišel do svojih sklepov, vendar je moral biti, kljub petim napakam, zavidljivo spreten. Pri iskanju Mersennovih praštevil so matematiki veliko uspešnejši kot pri iskanju Fer-matovih praštevil. Znanih je že 46 Mersenno-vih praštevil. Na spletu poteka projekt iskanja teh praštevil, imenovan Great Internet Mer-senne Prime Search (GIMPS) [9]. V okviru tega projekta so našli tudi do zdaj največje Mersennovo praštevilo. Največje do danes poznano praštevilo je število 243112609 -1, ki ima v decimalnem zapisu natanko 12978189 števk. Odkrili so ga avgusta leta 2008, seveda s pomočjo računalnikov. Če bi ga hoteli zapisati, bi ob normalni pisavi v desetiškem sistemu potrebovali več kot petdeset velikih 60-listnih zvezkov! Iz njegovega zapisa vidimo tudi, da spada med Mersennova praštevi-la. Nasploh spadajo do danes največja znana praštevila med Mersennova praštevila. Kot vidimo, so Mersennova praštevila dokaj do- bro raziskana, toda kljub temu se je pojavilo enostavno vprašanje, na katerega še ni bilo odgovora: Ali je neskončno mnogo Mersen-novih praštevil? Z Praštevilski izrek Morda najpomembnejši, zagotovo pa z najbogatejšo in spletk polno zgodovino med vsemi tukaj omenjenimi izreki in domnevami, je praštevilski izrek. Ta nam odgovori na vprašanje, koliko je približno praštevil na nekem konkretnem delu naravnih števil. S pomočjo praštevilskega izreka lahko, recimo, odgovorimo na vprašanje, koliko je približno praštevil med prvimi milijontimi naravnimi števili. Poudarek je na besedi približno, število praštevil se v resnici asimptotično približuje vrednosti izraza v praštevilskem izreku. Asimptotično približevanje pa presega težavnost tega članka in zato se bomo zadovoljili z besedo približno. O asimptotičnem približevanju si lahko več preberemo v [3]. Seveda je bralcu poznan že vsaj en način, kako poiskati praštevila med prvimi nekaj naravnimi števili. Iz množice naravnih števil najprej izločimo število 1 in nato vsa soda naravna števila, razen števila 2. Nato izločimo vse večkratnike števila 3, razen samega števila 3. Nadaljujemo in izločamo vse večkratnike naravnih števil, ki še niso bila izločena. Ta postopek je poznan pod imenom Eratostenovo sito. Njegova glavna pomanjkljivost je, da je precej dolgotrajen, vendar nam po drugi strani najde prav vsa praštevila. Poglejmo zdaj, kaj pravi praštevilski izrek. Praštevilski izrek: Med prvimi n naravnimi števili je približno n/ log n praštevil. Z log n je označen naravni logaritem števila n. S pomočjo praštevilskega izreka izračunamo, da je med prvimi milijon naravnimi števili približno 72382 praštevil. V resnici jih je 78498. Pravo moč dobi praštevilski izrek šele za zelo velika števila. Prva matematika, ki sta to domnevala neodvisno drug od drugega, sta bila Adrien-Marie Legendre (1752-1833) in na začetku omenjeni Gauss. Izrek je bil z analitičnimi metodami pokazan konec devetnajstega stoletja. Pri tem sta Jacques Salomon Hada-mard (1865-1963) in de la Vallée Poussin (1866-1962) uporabila analitične metode, vključno z znano Riemannovo zeta funkcijo. O tem si lahko več preberemo v [3]. Zanimivejše je bilo dogajanje okrog elementarnega dokaza praštevilskega izreka. Elementarni dokaz pomeni, da v njem ne uporabljamo zahtevnih analitičnih metod in izrekov. Pred približno sto leti so imeli matematiki še zelo malo upanja, da bodo kmalu našli dokaz te vrste. Sredi prejšnjega stoletja sta se prej omenjeni Erdos in Atle Selberg (1917-2007), norveško-ameriški matematik, zelo približala dokazu. Nato sta se sprla in dokaz dokončala vsak zase [8]. Selberg je leta 1950 za svoje delo na elementarnem dokazu praštevilskega izreka dobil celo Fi-eldsovo medaljo, ki je najvišje matematično priznanje. Redakcijska opomba: Članek je objavljen izjemoma, kljub temu, da v njem ni pomembnejše navezave na pouk. Prepričani smo, da bodo učitelji iz vsebine prispevka znali izluščiti gradivo za preiskovalne naloge, za moti-viranje učencev, za delo z nadarjenimi itd____ Z Viri in literatura 1. T. M. Apostol, Introduction to Analytic Number theory, Springer (1976). 2. J. Bračič, Uvod v analitično teorijo števil. Podiplomski seminar iz matematike 26, DMFA - založništvo (2003). 3. A. Jurišic, J. Tonejc, Pametne kartice in varnost, Monitor, Letnik 11, št. 6 (2001), str. 66-75. 4. A. Jurišic, J. Tonejc, Pametne kartice. Del 2, Zasebno življenje javnih ključev, Monitor, Letnik 11, št. 7-8 (2001), str. 44-51. 5. Pridobljeno s spletnega mesta: http://www.math.dart-mouth.edu/_euler/correspondence/letters/000765.pdf. 6. Pridobljeno s spletnega mesta: http://www.nd.edu/_ dgalvin1/pdf/bertrand.pdf. 7. Pridobljeno s spletnega mesta: http://www.math.co-lumbia.edu/_goldfeld/ErdosSelbergDispute.pdf. 8. Spletno mesto: http://www.mersenne.org/. 096 Nekaj več o praštevilih Priprava bodočih učiteljev matematike na povezovanje znanj pri pouku matematike Preparation of future mathematics teachers for integration of knowledge at mathematical lessons I Povzetek Povezovanje znanj pri pouku matematike je lahko dragocena sestavina pouka matematike, ki pomaga osmišljati nekatere matematične vsebine, prispeva k poglobitvi konceptualnih znanj, spodbuja k bolj celostnemu pogledu na vsebine in dejavnosti pri pouku matematike ter ustvarja pogoje za razvoj procesnih znanj. Odnos do povezovanja znanj pri učiteljih je v določeni meri odvisen že od izhodiščnega razumevanja pomena le-teh ter osebnih stališč in izkušenj s tovrstnimi aktivnostmi, zato je pomembno, da se z njimi seznanijo že študenti, bodoči učitelji matematike. V prispevku poleg teoretičnih izhodišč povezovanja znanj predstavimo tudi nekatere načine priprave študentov na povezovanje znanj pri pouku matematike, ki jih izvajamo na Oddelku za matematiko in računalništvo na Fakulteti za naravoslovje in matematiko Univerze v Mariboru. Ključne besede: pouk matematike, povezovanje znanj, med-predmetne povezave, projektni dnevi I Abstract Integrating knowledge within mathematics can be a valuable component of mathematical instruction that helps making sense of some mathematical content, contributs to a deepening of conceptual knowledge, encourages a more integrated (holistic) view of the content and activities in mathematics and creates Samo Repolusk, Fakulteta za naravoslovje in matematiko, Univerza v Mariboru Matematika v šol i ~ XVIII. [2012] ~ 068-072 conditions for the development of some specific skills (eg. integrating knowledge, finding resources, generalising). Teachers' attitudes towards integration of knowledge depend on their basic understanding of the practice of integration, as well as their personal views and experiences with such activities, hence it is important to present the paradigm to students, future mathematics teachers. The paper presents some possible ways of preparing students for interdisciplinary approach to mathematics instruction at the Faculty of Natural Sciences and Mathematics, University of Maribor. Key words: mathematics instruction, integration of knowledge, interdisciplinary links, project days a Uvod Povezovanje znanj pri pouku matematike je eden izmed načinov celostnega (holističnega) pristopa k učenju in poučevanju. Zmožnost povezovanja matematičnih znanj znotraj matematike kot tudi različnih znanj med posameznimi disciplinami je sicer eden od splošnih ciljev pouka matematike in šolskega kurikula nasploh (prim. Žakelj et al., 2007), vendar je razvijanje te zmožnosti zahtevna naloga, saj zahteva tako od učitelja kot tudi od učencev široko in poglobljeno poznavanje obravnavanih konceptov, ob tem pa v prvi vrsti tudi poznavanje orodij in jezika, s katerim lahko pojave opisujemo. Prav zaradi tega moramo imeti na primarni in sekundarni stopnji izobraževanja realna pričakovanja o možnostih povezovanja znanj. Tako kot je za uspešno uporabo strategij in hevristik pri reševanju problemov najprej potrebno poznavanje dovolj širokega nabora temeljnih pojmov, konceptov in postopkov (prim. Ma-gajna, 2003), tako je za uspešno (razumljivo, učinkovito) povezovanje znanj potrebno najprej zelo dobro ali celo odlično poznavanje temeljnih pojmov, pojavov in orodij vsake discipline. Tega dejstva se lahko posebej za- vemo ob v današnjem času hitro napredujočih znanosti (medicina, farmacija, genetika, teoretična fizika, ekologija, nanotehnologije, tehnologije učinkovite izrabe novih energetskih virov itd.), kjer ključne preboje v razumevanju pojavov in novih odkritjih omogoča šele tesno sodelovanje med strokovnjaki različnih znanstvenih področij, mnogi raziskovalci pa celo sami izkazujejo odličnost na več področjih hkrati. V luči zavedanja zahtevnosti udejanjanja paradigme povezovanj znanj lahko trdimo, da se povezovanje znanj v šoli lahko najprej učinkovito uresničuje predvsem znotraj posameznih predmetnih področij, med sorodnimi ali različnimi predmeti pa predvsem na intuitivni ravni zavedanja medsebojne povezanosti orodij in pojavov, šele kasneje (z dovolj dobrim obvladovanjem kompleksnih orodij in jezika posameznih disciplin) pa tudi na ravni uporabe različnih znanj in kreativnega oblikovanja novih spoznanj. Kljub temu pa nas zahtevnost razvijanja zmožnosti povezovanja znanj ne sme odvrniti od iskanj in delovanja v tej smeri, saj je konec koncev to eden izmed načinov nekonvencionalnega in ustvarjalnega učenja, ki je lahko v tradicionalnem šolskem okolju tudi zanemarjeno. 098 Priprava bodočih učiteljev matematike na povezovanje znanjpri pouku matematiae b Nekatera teoretična izhodišča povezovanja znanj Odnos do povezovanja znanj je odvisen že od izhodiščnega razumevanja pomena le-teh ter osebnih stališč in izkušenj s tovrstnimi aktivnostmi, zato je smiselno, da z njimi seznanimo že študente, bodoče učitelje matematike. Kakšno dodano vrednost k pouku matematike lahko prispeva povezovanje znanj? Poglejmo nekatere vidike, kjer se lahko povezovanje znanj izkaže kot smiselno. • Osmišljanje matematičnih vsebin: Učitelji se pri pouku pogosto srečamo z vprašanji »Kje bomo pa to rabili v življenju?« ali pa »Kje je to uporabno?«. Nanje lahko odreagiramo na različne načine: lahko jih preslišimo, lahko jih razberemo kot znak nespoštovanja in provokacije, lahko pa v njih uzremo iskrico bolj ali manj resno izražene mladostniške radovednosti, ki nas vabi k premisleku o smislu tega, kar počnemo. Katero pot bomo izbrali, je seveda odvisno od nas samih, a z načinom reagiranja si pri učencih oblikujemo strokovno in moralno podobo. V tem kontekstu je lahko povezovanje znanj z namenom osmišljanja obravnavanih vsebin odlična priložnost za potešitev mladostniške radovednosti in oblikovanje ustvarjalnega in spoštljivega vzdušja v razredu. • Poglobitev konceptualnih znanj: Moč, lepota in globina nekaterih matematičnih konceptov včasih še posebej zasije v luči predstavitve uporabe teh konceptov na drugih predmetnih področjih (npr. pogled na fizikalne enačbe skozi oči matematičnih funkcij, uporaba vektorjev, odvoda, integrala, itd.), hkrati pa takšno povezovanje znanj omogoča prepoznavanje in posplošeno razumevanje neka- terih lastnosti matematičnih objektov (npr. osvoboditev od fiksacije uporabe standardnih črkovnih oznak za spremenljivke v predpisih). Zgodovinsko gledano so se mnogi matematični objekti razvili z abstrahiranjem lastnosti in zvez na konkretnih objektih in procesih v naravi, v današnji šoli pa smo ta uvid v naravno nastajanje novega znanja izgubili, saj abstraktnih konceptov ne znamo prepoznavati v konkretnem dogajanju okrog nas. Prav z umestitvijo matematičnih konceptov v njihovo izvorno okolje lahko poglobimo nekatera konceptualna znanja. Razvijanje procesnih znanj: Poleg usva-janja matematičnih vsebin pri pouku matematike razvijamo tudi procesna znanja, med katerimi so mnoga prenosljiva na ostala področja človekovega delovanja in so običajno trajnejša. Povezovanje znanj nam lahko pomaga razvijati nekatera procesna znanja, kot so zbiranje, urejanje, povezovanje, analiziranje, prikazovanje in kritično preverjanje podatkov in rezultatov, iskanje in uporaba virov, utemeljevanje, posploševanje, napovedovanje, učinkovita uporaba simbolnega jezika, modeliranje, logično sklepanje itd. Celostni (holistični) pogled na vsebine in dejavnosti pri pouku: Parcialni pristopi k stvarnosti (izraziteje od 19. stoletja z oblikovanjem ločenih znanstvenih disciplin) so človeštvu po eni strani omogočili silovit znanstveni razvoj in tehnični napredek, po drugi strani pa niso uspeli rešiti nekaterih temeljnih bivanjskih izzivov človeštva ali so celo ustvarili nove (izčrpavanje naravnih virov, lokalno in globalno uničevanje naravnega okolja, vzpodbujanje potrošništva ob nenehnem vojaške tehnologije, tehnični odpadki, prekomerno naraščanje prebivalstva, zlorabe v genetiki . ). Ko na prehodu v 3. tisočletje izraziteje čutimo tudi negativne posledice ekonomskega in znanstvenega napredka zaradi omejenega parcialnega gledanja na stvarnost in odsotnosti etične komponente nosilcev razvoja, se pred ljudi in znanost postavlja izziv popravnega izpita. Reševanje ekološke problematike, izraba novih virov energije in skrb za kakovost bivanja zahtevajo vse večje povezovanje različnih znanosti, saj vse bolj spoznavamo, da le celostni pogled na stvarnost omogoča iskanje odgovorov na današnje izzive. Zavedanje, da je stvarnost ena, različni pa so pristopi k njej in vidiki njene obravnave (kar počnemo z delitvijo znanosti na znanstvene discipline), nam omogoča celosten pogled na razvoj novih spoznanj (npr. povezovanje biologije, kemije in etike v medicini, ali fizike, tehnike, matematike, geografije in biologije pri razvoju »zelenih tehnologij« izrabe naravnih virov) in hkrati zmanjša možnost, da spregledamo kakšno ključno razvojno komponento (tehnološki odpadki, etična vprašanja ...). Povezovanje znanj v šoli je zato naraven korak v smeri razmišljanja, ki ga bo moralo človeštvo ponotranjiti, če bo želelo živeti kakovostno in pravično, oziroma sploh preživeti. V nadaljevanju bomo predstavili nekatere načine priprave študentov na povezovanje znanj pri pouku matematik, ki jih izvajamo na Oddelku za matematiko in računalništvo na Fakulteti za naravoslovje in matematiko Univerze v Mariboru. Y Primeri priprave študentov na povezovanje znanj Pri oblikovanju načinov priprave študentov na povezovanje znanj na naši fakulteti sem izhajal iz izkušenj lastne pedagoške prakse (pristopi, preizkušeni v razredu), ki sem jih v letih dela na fakulteti nadgrajeval z novimi teoretičnimi spoznanji in zgledi dobrih praks ostalih učiteljev in njihovih izobraževalcev. Nekaj možnosti je na kratko predstavljenih v nadaljevanju: • Predstavitev teoretičnih osnov: Če želimo, da študenti povezovanje znanj sprejmejo za naravno in smiselno, jim je treba predstaviti širši kontekst izgradnje znanja. Ta kontekst se oblikuje postopno preko predstavitev teorij učenja, kompetenc učiteljev matematike, kompetenc, ki jih razvijamo v šolskem prostoru in posebej matematične kompetence, taksonomije matematičnih znanj (npr. prirejena Ga-gnejeva taksonomija, ki jo uporabljamo pri pouku matematike), kot tudi obravnave umestitve znanja v današnji družbi. • Izdelava pisne priprave učne enote: V uvodu vsake učne ure (motivacija, mobilizacija znanja) od študentov zahtevamo pripravljenost na vprašanje učencev »Kje pa je to uporabno?«. Ob tem naj najprej poudarimo, da nikakor nismo zagovorniki opravičevanja obravnave vsebin zaradi njihove »uporabne vrednosti«, še posebej zato, ker matematika ni zgolj ali predvsem aplikativna znanost, ampak tudi umetnost - umetnost človeškega uma. Tako kot se Leonardu da Vinciju ni bilo treba nikomur opravičevati, zakaj je porabil dragoceni čas za ustvarjanje Zadnje večerje, ali pa se Enniu Morriconeju ni treba opravičevati za porabljeni čas skladanja 0100 Priprava bodočih učiteljev matematike na povezovanje znanjpri pouku matematiae nepozabne Gabriel's Oboe, tako tudi matematikom ni treba vedno opravičevati aktivnosti, s katerimi matematika bogati človeški um. Kljub temu pa sodi k najbolj notranji potrebi človeškega uma, da želimo poznati pomen stvari. Če učenec želi odgovor, lahko to spoštujemo in morda celo ugotovimo, da je vprašanje zanimivo tudi za nas. Načinov, kako poiskati odgovor na takšno vprašanje, je veliko: poleg osebnih izkušenj v času šolanja in študija imamo danes skoraj neomejen vir informacij v knjigah in na svetovnem spletu, seveda pa tudi pomoč drugih učiteljev. Pri odgovorih na takšno vprašanje pride zmožnost povezovanja znanj posebej do izraza. Študente tudi vzpodbujamo, da se pri reševanju nalog ne izogibajo primerom, ki se navezujejo na druga predmetna področja. Kjer takšnih primerov v matematičnih knjigah ni, lahko za pomoč zaprosijo tudi kolege učitelje drugih predmetov. • Nasveti za odprto učenje in iskanje primerov povezovanja znanj v času študija: Čas študija je lahko dobra priložnost, da študent po pridobljenih temeljih na primarni in sekundarni stopnji izobraževanja ta znanja nadgradi s primeri uporabe, ki jih sreča v času študija. Študente tako vzpodbujamo, da so pri predmetih, kot so fizika, numerične metode, matematično modeliranje ipd., posebej pozorni na zglede uporabe matematičnih konceptov, študenti dvopredmetnih smeri pa imajo poleg tega še možnost povezovanja znanj pri specifičnih predmetih druge stroke. • Učenje problemskih znanj: Strategije in hevristike reševanja problemov so poseben primer problemskih znanj pri pouku matematike, ki sodijo med procesna zna- nja. Razvijanje takšnih procesnih znanj je eden od načinov povezovanja znanj znotraj matematike. Študentom na zgledih osnovnošolske in srednješolske matematike predstavimo zglede uporabe strategij in hevristik reševanja matematičnih problemov (prim. Posamentier in Krulik, 1998). Zgledi povezovanja znanj ob konkretnih matematičnih vsebinah: Za študente je smiselno primere povezovanj znanj med matematiko in ostalimi predmeti prikazati tudi ob konkretnih matematičnih vsebinah. Tako s študenti skupaj predelamo nekatere vsebine, kjer lahko dobro prikažemo povezovanje znanj: obrestni račun (ekonomija), transformacije grafov funkcij (fizika, primeri matur oz. zunanjih preverjanj iz drugih držav z aplikativnimi zgledi), krivulje II. reda (fizika, astronomija), matematično modeliranje (različne discipline), vektorji (fizika), obdelava podatkov (računalništvo), ustni del poklicne mature pri matematiki (povezava matematike in strokovnega predmeta). Projektna naloga ali portfolio: Projektna naloga ali portfolio sta celoletni obveznosti študentov, ob kateri v miru premislijo specifične vsebine, s katerimi se bodo srečevali v neposredni učni praksi. Ta del študijskih obveznosti je najbolj avtentičen način učenja povezovanja znanj. V primeru izbire portfolia študenti izdelajo podrobne priprave učnih ur iz naslednjih sklopov: priprava tem za raziskovalne naloge pri matematiki, učenje z uporabo matematičnih računalniških programov ali e-gradiv, priprava ur z medpredmetni-mi povezavami (naravoslovni predmeti, tehnika, geografija ...), preiskovanje/reše- letnega načrta (koncepta) razrednih ur in podrobna priprava ene tematske ure. V vseh navedenih primerih morajo povezovati znanja tako znotraj matematike kot med različnimi predmeti. V primeru izbire projektne naloge pa študenti pripravijo podrobne priprave (do izvedbe natančno) za tri projektne dneve na temo medpredmetnih povezav. Vsak dan projektnega tedna predvidoma obsega 6 ur, vključijo pa lahko tudi terenske vaje in strokovno ekskurzijo. V vsakem primeru učenci izdelajo podrobne učne priprave, delovne liste z rešitvami, opis potrebnih materialov in morebitnih stroškov (materiali, prevoz, vstopnine). Projektna naloga za medpredmetni projektni teden nastaja praviloma v timu (dva oz. največ trije študenti), kar omogoča pridobivanje izkušenj s timskim načrtovanjem. V vseh primerih dobijo študenti natančna navodila za delo, pri čemer se seznanijo tudi s citiranjem virov, pravili za pisanje matematičnih besedil, kot tudi z vsem ostalim, kar sodi k projektnemu delu. Zadnji dve študijski leti smo izdelavo projektnih nalog na temo medpredmetnih povezav vključili tudi v projekt Naravoslovne kompetence, katerega nosilka je Fakulteta za naravoslovje in matematiko Univerze v Mae Viri in literatura riboru, pri čemer študenti v projektu sodelujejo prostovoljno, nastala gradiva pa ustrezajo tudi zahtevam projektnih smernic (npr. vključevanje in opis naravoslovnih kompe-tenc, ki jih ob tem razvijamo ter natančnih navodil za učence in učitelje). Naša skromna želja je, da bi najboljša med temi gradivi lahko kdaj postala dostopna tudi učiteljem v obliki publikacije. 5 Sklep Naštete možnosti lahko predstavljajo prve korake bodočih učiteljev v zavestno povezovanje znanj pri pouku matematike. Učitelj, ki ponotranji zavest o smiselnosti povezovanja znanj in pridobi tudi konkretne pozitivne izkušnje takšnega povezovanja, bo to tudi veliko suvereneje vpeljeval v učno prakso, pri čemer mu iskanje povezav ne bo predstavljalo bremena, ampak intelektualni izziv v vseživljenjski strokovni rasti. Proces je sicer dolgotrajen in za kakšnega učitelja tudi naporen, a po lastni izkušnji odzivov učencev v razredu in osebnega zadovoljstvo ob novih odkritjih odtehta vložen trud. In če nam na ta način uspe matematiko pri nekaterih učencih približati kot lepo in intelektualno izzivalno, smo naredili korak v dobri smeri. 1. Magajna, Z. (2003). 'Problemi, problemsko znanje in problemski pristop pri pouku matematike'. Matematika v šoli, 10, str. 129-138. 2. Posamentier, A. S., Krulik, S. (1998). Problem-Solving Strategies for Efficient and Elegant Solutions: A Resource for the Mathematics Teacher. Thousand Oaks: Corwin Press. 3. Zakelj, A., Bon-Klajnšček, M., Jerman, M., Kmetič, S., Repolusk, S., Ruter, A. (2007). Učni načrt, gimnazijski program - Matematika. Ljubljana: ZRSŠ. 0102 Priprava bodočih učiteljev matematike na povezovanje znanjpri pouku matematiae Najava Konference o učenju in poučevanju matematike 2012 (KUPM 2012) http://www.zrss.si/kupm2012/ Vse okoli nas se spreminja. V zadnjem desetletju so bili prenovljeni tudi vsi učni načrti in katalogi znanj ter prinesli nekaj ključnih novosti, od avtonomije učitelja do učenja z uporabo informacijsko-komunikacijske tehnologije. Kako se spreminjamo mi in pouk matematike? Odgovore bomo skupaj iskali na prvi mednarodni konferenci KUPM 2012. Vljudno vas vabimo na 1. mednarodno Konferenco o učenju in poučevanju matematike KUPM 2012, ki bo 23. in 24. avgusta 2012 v Mariboru. Konferenca je namenjena učiteljem matematike v osnovnih in srednjih šolah ter učiteljem razrednega pouka. Namen konference je predstaviti nove in izvirne pristope k učenju in poučevanju matematike po vsej vertikali in za vse izobraževalne programe. Delo konference bo potekalo v obliki: • plenarnih predavanj z vabljenimi predavatelji, • delavnic, • predavanj, • plakatnih predstavitev. Gradivo konference: zbornik povzetkov prispevkov in e-zbornik prispevkov Spremljevalni program: natečaj Odsev matematike v fotografskem objektivu (zavihek Fotografski natečaj na spletni strani) Matematika v šol i ~ XVIII. [2012] ~ 068-072 Vsebine: • Aktivna raba informacijsko-komunikacijske tehnologije pri učenju in poučevanju matematike - Aktivna raba različnih matematičnih programov in računal (numeričnih, grafičnih, simbolnih) - Učinki uporabe e-gradiv • Pristopi, strategije in oblike dela pri pouku matematike s preverjeno učinkovitostjo - Različne organizacijske tehnike pri pouku matematike - Diferenciacija in individualizacija pouka - Učne težave pri učenju matematike - Nevralgične točke pri učenju in poučevanju matematike • Ugotavljanje znanja pri matematiki - Spremljanje učenčevega napredka in vloga povratne informacije - Pisni preizkusi - Alternativne oblike preverjanja in ocenjevanja - Vpliv nacionalnih in mednarodnih preverjanj na učenje in poučevanje matematike • Uvajanje novosti iz učnih načrtov in katalogov znanja - Reševanje in raziskovanje matematičnih in realnih problemov - Matematično modeliranje - Vzorci - Nadpredmetno povezovanje znanja - Bralna pismenost - Učenje učenja - Od obdelave podatkov do statistike • Matematika in naravoslovni predmeti Pomembni datumi: Rok za oddajo povzetkov prispevkov: 15. 4. 2012 Rok za oddajo celotnega prispevka: 25. 5. 2012 Zaključek fotografskega natečaja: 20. 5. 2012 Rok za prijavo na konferenco (kotizacije ni): 15. 6. 2012 oz. do zapolnitve mest Oddaja povzetkov in prijava na konferenco poteka na spletni strani http://www.zrss.si/ kupm2012/ v zavihku Povzetki, prispevki in prijava. 0104 Matematika v šol i ~ XVIII. [2012] ~ 068-072 Matematika v šol i ~ XVIII. [2012] ~ 068-072 Matematika v šol i ~ XVIII. [2012] ~ 068-072 Matematika v šol i ~ XVIII. [2012] ~ 068-072 Matematika v šol i ~ XVIII. [2012] ~ 068-072 Matematika v šol i ~ XVIII. [2012] ~ 068-072 Matematika v šoli ~ XVIII. [2002] ~ 02-06 Matematika v šol i ~ XVIII. [2012] ~ 068-072 Matematika v šoli ~ XVIII. [2012] ~ 6-06 Matematika v šoli ~ XVIII. [2012] ~ 02-06